SlideShare a Scribd company logo
1 of 67
Download to read offline
TÀI LIỆU
LUYỆN THI TUYỂN SINH
LỚP 10 – MÔN TOÁN
[lưu hành nội bộ]
TP HCM – 2012
1
MỤC LỤC
I. MỘT SỐ THÔNG TIN CẦN THAM KHẢO....................................................................... 4
1. Điểm chuẩn năm học 2009-2010...................................................................................... 4
2. Điểm chuẩn năm học 2010-2011...................................................................................... 7
3. Điểm chuẩn năm học 2011-2012.................................................................................... 10
4. Bí quyết đạt "điểm rơi phong độ" ngay trong ngày thi.................................................... 13
II. Đề thi tuyển sinh lớp 10 thành phố Hồ Chí Minh .............................................................. 15
1. Đề thi tuyển sinh lớp 10 tp Hồ Chí Minh Năm học 2011-2012....................................... 15
2. Đề thi tuyển sinh lớp 10 tp Hồ Chí Minh Năm học 2010-2011....................................... 15
3. Đề thi tuyển sinh lớp 10 tp Hồ Chí Minh Năm học 2009-2010....................................... 16
4. Đề thi tuyển sinh lớp 10 tp Hồ Chí Minh Năm học 2008-2009....................................... 17
5. Đề thi tuyển sinh lớp 10 tp Hồ Chí Minh Năm học 2007-2008....................................... 18
6. Đề thi tuyển sinh lớp 10 tp Hồ Chí Minh Năm học 2006-2007....................................... 18
III. Đề thi tuyển sinh lớp 10 Chuyên thành phố Hồ Chí Minh................................................ 19
1. Chuyên tp Hồ Chí Minh, năm học 2011-2012(N/A)....................................................... 19
2. Chuyên tp Hồ Chí Minh, năm học 2010-2011(N/A)....................................................... 19
3. Chuyên tp Hồ Chí Minh, năm học 2009-2010................................................................ 19
4. Chuyên tp Hồ Chí Minh, năm học 2008-2009(N/A)....................................................... 21
5. Chuyên tp Hồ Chí Minh, năm học 2007-2008................................................................ 21
6. Chuyên tp Hồ Chí Minh, năm học 2006-2007................................................................ 21
7. Chuyên Lê Hồng Phong tp Hồ Chí Minh, năm học 2001-2002....................................... 22
8. Chuyên Lê Hồng Phong tp Hồ Chí Minh, năm học 2002-2003....................................... 23
9. Chuyên Lê Hồng Phong tp Hồ Chí Minh, năm học 2003-2004....................................... 24
10. Chuyên Lê Hồng Phong tp Hồ Chí Minh, năm học 2004-2005..................................... 25
11. Chuyên Đại học Sư Phạm tp Hồ Chí Minh, năm học 2006-2007(N/A)......................... 26
12. Chuyên Trần Đại Nghĩa tp Hồ Chí Minh, năm học....................................................... 26
2
IV. Đề thi tuyển sinh lớp 10 trường Phổ Thông Năng Khiếu.................................................. 26
1. Toán không chuyên trường Phổ Thông Năng Khiếu, năm học 2011-2012...................... 26
2. Toán không chuyên trường Phổ Thông Năng Khiếu, năm học 2010-2011...................... 27
3. Toán không chuyên trường Phổ Thông Năng Khiếu, năm học 2009-2010...................... 28
4. Toán AB trường Phổ Thông Năng Khiếu, năm học 2008-2009 ...................................... 29
5. Toán CD trường Phổ Thông Năng Khiếu, năm học 2008-2009 ...................................... 30
6. Toán AB trường Phổ Thông Năng Khiếu, năm học 2007-2008 ...................................... 31
7. Toán AB trường Phổ Thông Năng Khiếu, năm học 2006-2007(N/A)............................. 32
8. Toán CD trường Phổ Thông Năng Khiếu, năm học 2006-2007(N/A)............................. 32
9. Toán AB trường Phổ Thông Năng Khiếu, năm học 2005-2006(N/A)............................. 32
10. Toán CD trường Phổ Thông Năng Khiếu, năm học 2005-2006(N/A)........................... 32
11. Toán AB trường Phổ Thông Năng Khiếu, năm học 2004-2005 .................................... 33
12. Toán CD trường Phổ Thông Năng Khiếu, năm học 2004-2005 .................................... 34
V. Đề thi tuyển sinh lớp 10 trên toàn quốc............................................................................. 35
1. Tuyển sinh lớp 10 thành phố Đà Nẵng năm học 2011-2012 ........................................... 35
2. Tuyển sinh lớp 10 thành phố Đà Nẵng năm học 2010-2011 ........................................... 36
3. Tuyển sinh lớp 10 thành phố Đà Nẵng năm học 2009-2010 ........................................... 36
4. Tuyển sinh lớp 10 thành phố Hà Nội năm học 2010-2011.............................................. 37
5. Tuyển sinh lớp 10 thành phố Hà Nội năm học 2011-2012.............................................. 38
6. Tuyển sinh lớp 10 tỉnh Hải Dương năm học 2011-2012 ................................................. 39
7. Tuyển sinh lớp 10 tỉnh Hà Tĩnh năm học 2010-2011...................................................... 40
8. Tuyển sinh lớp 10 tỉnh Nghệ An năm học 2011-2012 .................................................... 40
9. Tuyển sinh lớp 10 tỉnh Hà Tĩnh năm học 2011-2012...................................................... 41
10. Tuyển sinh lớp 10 tỉnh Bình Định năm học 2011-2012 ................................................ 42
11. Tuyển sinh lớp 10 tỉnh Quảng Nam năm học 2011-2012.............................................. 42
12. Tuyển sinh lớp 10 tỉnh Quảng Ngãi năm học 2011-2012.............................................. 43
3
13. Tuyển sinh lớp 10 tỉnh Thanh Hoá năm học 2011-2012 ............................................... 44
14. Tuyển sinh lớp 10 tỉnh Khánh Hoà năm học 2011-2012............................................... 44
15. Tuyển sinh lớp 10 tỉnh Bình Dương năm học 2010-2011 ............................................. 45
16. Tuyển sinh lớp 10 tỉnh Bến Tre năm học 2011-2012.................................................... 46
VI. Đề tham khảo ôn tập tuyển sinh....................................................................................... 48
1. ĐỀ SỐ 01....................................................................................................................... 48
2. ĐỀ SỐ 02....................................................................................................................... 48
3. ĐỀ SỐ 03....................................................................................................................... 49
4. ĐỀ SỐ 04....................................................................................................................... 50
5. ĐỀ SỐ 05....................................................................................................................... 51
6. ĐỀ SỐ 06....................................................................................................................... 52
7. ĐỀ SỐ 07....................................................................................................................... 53
8. ĐỀ SỐ 08....................................................................................................................... 54
9. ĐỀ SỐ 09....................................................................................................................... 55
10. ĐỀ SỐ 10..................................................................................................................... 56
11. ĐỀ SỐ 11..................................................................................................................... 57
12. ĐỀ SỐ 12..................................................................................................................... 57
13. ĐỀ SỐ 13..................................................................................................................... 58
14. ĐỀ SỐ 14..................................................................................................................... 59
15. ĐỀ SỐ 15..................................................................................................................... 60
16. ĐỀ SỐ 16..................................................................................................................... 61
17. ĐỀ SỐ 17..................................................................................................................... 62
18. ĐỀ SỐ 18..................................................................................................................... 63
19. ĐỀ SỐ 19..................................................................................................................... 63
20. ĐỀ SỐ 20..................................................................................................................... 64
21. ĐỀ SỐ 21..................................................................................................................... 65
4
I. MỘT SỐ THÔNG TIN CẦN THAM KHẢO
1. Điểm chuẩn năm học 2009-2010
Tên trường
Điểm chuẩn
NV1 NV2 NV3
THPT Trưng Vương 36.75 37.25 37.25
THPT Bùi Thị Xuân 40.00 41.00 42.00
THPT Năng khiếu TDTT 21.00 21.00 21.50
THPT Ten Lơ Man 26.25 26.75 27.50
THPT Lương Thế Vinh Q1 32.25 32.25 32.25
THPT Giồng Ông Tố 21.25 21.50 22.50
THPT Thủ Thiêm 17.00 17.50 17.75
THPT Lê Quý Đôn 37.50 38.00 38.00
THPT Nguyễn Thị Minh Khai 40.50 41.50 42.50
THPT Lê Thị Hồng Gấm 29.75 30.50 30.50
THPT Marie Curie 32.25 32.25 32.50
THPT Nguyễn Thị Diệu 29.50 29.50 29.50
THPT Nguyễn Trãi 32.00 32.25 32.25
THPT Nguyễn Hữu Thọ 24.50 25.00 25.25
THPT Hùng Vương 36.50 37.00 37.00
TH Thực Hành ĐHSP 38.50 38.75 39.00
THPT Trần Khai Nguyên 31.25 31.25 31.25
THPT Trần Hữu Trang 25.75 26.25 26.50
5
THPT Mạc Đĩnh Chi 35.75 35.75 35.75
THPT Bình Phú 33.00 33.00 33.00
THPT Lê Thánh Tôn 25.50 25.75 26.00
THPT Ngô Quyền 24.00 24.50 24.75
THPT Tân Phong 20.00 20.50 20.50
THPT Nam Sài Gòn 18.50 19.50 20.50
THPT Lương Văn Can 28.25 28.50 29.50
THPT Ngô Gia Tự 17.50 18.50 19.25
THPT Tạ Quang Bửu 25.75 26.00 26.00
THPT Chuyên Năng khiếu TDTT Nguyễn Thị Định 18.75 19.75 20.25
THPT Nguyễn Khuyến 35.00 35.00 35.00
THPT Nguyễn Du 36.00 36.00 36.00
THPT Nguyễn An Ninh 23.00 24.00 24.25
THPT Diên Hồng 21.25 22.25 23.00
THPT Sương Nguyệt Anh 23.75 24.50 24.50
THPT Nguyễn Hiền 34.00 34.75 35.75
THPT Trần Quang Khải Q11 30.00 30.00 30.00
THPT Nam Kỳ Khởi Nghĩa 26.00 26.00 26.00
THPT Võ Trường Toản 30.50 30.75 30.75
THPT Trường Chinh 28.25 28.50 28.50
THPT Thạnh Lộc 22.50 22.50 22.50
THPT Thanh Đa 26.50 26.75 27.00
6
THPT Võ Thị Sáu 35.75 36.00 36.00
THPT Gia Định 37.75 37.75 37.75
THPT Phan Đăng Lưu 30.50 30.75 30.75
THPT Hoàng Hoa Thám 31.00 31.50 31.50
THPT Gò Vấp 33.25 34.00 34.75
THPT Nguyễn Công Trứ 38.00 39.00 39.00
THPT Trần Hưng Đạo 33.50 33.50 33.50
THPT Nguyễn Trung Trực 27.50 27.50 28.25
THPT Phú Nhuận 38.50 38.50 39.00
THPT Hàn Thuyên 21.00 21.50 22.00
THPT Nguyễn Chí Thanh 33.50 34.50 34.75
THPT Nguyễn Thượng Hiền 41.75 42.75 43.75
THPT Lý Tự Trọng 25.00 25.50 26.25
THPT Nguyễn Thái Bình 29.75 30.00 30.00
THPT Nguyễn Hữu Huân 36.00 36.00 36.00
THPT Thủ Đức 32.50 33.50 33.50
THPT Tam Phú 29.75 30.00 30.00
THPT Hiệp Bình 24.75 25.25 25.25
THPT Long Thới 13.00 13.00 13.00
THPT Tân Bình 32.00 32.00 32.00
THPT Trần Phú 36.75 37.25 37.50
THPT Tây Thạnh 27.25 27.25 27.50
7
THPT Vĩnh Lộc 23.25 24.00 24.50
THPT An Lạc 26.25 26.50 26.50
2. Điểm chuẩn năm học 2010-2011
Teân Tröôøng NV 1 NV 2 NV 3
THPT Tröng Vöông 36.25 37.25 38.25
THPT Buøi Thò Xuaân 40.25 41.25 42.25
THPT Naêng Khieáu TDTT 15.00 15.00 15.00
THPT Ten Lô Man 26.50 27.00 27.50
THPT Löông Theá Vinh 32.75 33.50 34.50
THPT Leâ Quyù Ñoân 37.50 38.50 38.50
THPT Nguyeãn Thò Minh Khai 40.00 41.00 42.00
THPT Leâ Thò Hoàng Gaám 28.75 29.75 30.00
THPT Marie Curie 31.00 31.75 32.00
THPT Nguyeãn Thò Dieäu 27.75 28.50 28.75
THPT Nguyeãn Traõi 32.25 33.00 33.00
THPT Nguyeãn Höõu Thoï 22.50 22.75 23.75
THPT Huøng Vöông 35.50 36.50 36.50
TH Thöïc Haønh ÑHSP 40.50 41.00 42.00
TH Thöïc Haønh Saøi Goøn 34.75 35.25 35.50
THPT Traàn Khai Nguyeân 32.00 32.25 33.00
8
THPT Traàn Höõu Trang 25.75 26.00 27.00
THPT Maïc Ñónh Chi 36.25 37.25 38.25
THPT Bình Phuù 32.50 33.25 33.75
THPT Leâ Thaùnh Toân 26.75 27.25 27.25
THPT Ngoâ Quyeàn 25.50 26.50 26.50
THPT Taân Phong 20.00 20.25 20.75
THPT Nam Saøi Goøn 24.00 24.75 25.25
THPT Löông Vaên Can 26.50 27.00 27.00
THPT Ngoâ Gia Töï 19.00 19.50 20.50
THPT Taï Quang Böûu 23.50 24.25 25.00
THPT Phöôøng 7 15.00 15.00 15.50
THPT Chuyeân TDTT Ng.Thò Ñònh 25.00 25.50 25.75
THPT Nguyeãn Khuyeán 34.25 34.50 34.50
THPT Nguyeãn Du 36.25 36.50 36.50
THPT Nguyeãn An Ninh 25.75 26.75 27.00
THPT Dieân Hoàng 26.00 26.25 26.25
THPT Söông Nguyeät Anh 24.00 24.50 25.50
THPT Nguyeãn Hieàn 33.75 34.25 34.25
THPT Traàn Quang Khaûi 31.00 31.75 31.75
THPT Nam Kyø Khôûi Nghóa 29.25 29.75 29.75
THPT Voõ Tröôøng Toaûn 32.25 32.50 32.50
9
THPT Tröôøng Chinh 28.50 29.50 30.00
THPT Thaïnh Loäc 25.00 25.50 25.75
THPT Thanh Ña 25.75 26.00 27.00
THPT Voõ Thò Saùu 34.50 35.50 35.50
THPT Gia Ñònh 37.75 38.50 39.50
THPT Phan Ñaêng Löu 29.75 30.50 30.75
THPT Hoaøng Hoa Thaùm 31.00 31.50 31.50
THPT Goø Vaáp 32.50 32.75 32.75
THPT Nguyeãn Coâng Tröù 36.75 36.75 36.75
THPT Traàn Höng Ñaïo 33.00 33.50 33.50
THPT Nguyeãn Trung Tröïc 26.50 27.00 27.00
THPT Phuù Nhuaän 38.75 39.25 40.25
THPT Haøn Thuyeân 23.50 24.25 24.50
THPT Nguyeãn Chí Thanh 33.50 34.50 34.50
THPT Nguyeãn Thöôïng Hieàn 41.75 42.75 43.75
THPT Lyù Töï Troïng 26.00 26.50 27.50
THPT Nguyeãn Thaùi Bình 29.50 30.00 30.00
THPT Long Thôùi 15.00 15.00 15.00
THPT Phöôùc Kieån 15.00 15.00 15.00
THPT Taân Bình 31.50 31.75 31.75
THPT Traàn Phuù 37.00 38.00 38.00
10
THPT Taây Thaïnh 27.50 28.50 28.75
THPT Vónh Loäc 23.25 23.75 24.50
THPT Bình Trò Ñoâng A 21.50 22.00 22.00
THPT An Laïc 28.00 28.50 29.00
3. Điểm chuẩn năm học 2011-2012
Tên Trường Q/H NV 1 NV 2 NV 3
THPT Trưng Vương 01 33.50 34.50 35.50
THPT Bùi Thị Xuân 01 37.00 38.00 39.00
THPT Năng Khiếu TDTT 01 14.00 14.50 15.00
THPT Ten Lơ Man 01 24.75 25.50 26.50
THPT Lương Thế Vinh 01 31.25 31.50 32.00
THPT Lê Quý Đôn 03 35.00 35.25 35.25
THPT Nguyễn Thị Minh Khai 03 37.25 37.25 38.25
THPT Lê Thị Hồng Gấm 03 25.50 26.00 26.50
THPT Marie Curie 03 29.25 30.25 31.00
THPT Nguyễn Thị Diệu 03 26.25 26.50 27.50
THPT Nguyễn Trãi 04 28.50 29.25 30.00
THPT Nguyễn Hữu Thọ 04 20.00 20.50 21.25
THPT Hùng Vương 05 31.75 32.75 33.75
TH Thực Hành ĐHSP 05 37.25 38.25 39.25
TH Thực Hành Sài Gòn 05 33.00 34.00 34.00
11
THPT Trần Khai Nguyên 05 30.00 30.25 31.00
THPT Trần Hữu Trang 05 24.25 25.25 26.25
THPT Lê Thánh Tôn 07 22.50 23.00 23.00
THPT Ngô Quyền 07 24.50 24.50 25.00
THPT Tân Phong 07 18.25 19.00 19.75
THPT Nam Sài Gòn 07 23.75 24.50 25.00
THPT Lương Văn Can 08 25.00 25.50 26.00
THPT Ngô Gia Tự 08 19.00 20.00 21.00
THPT Tạ Quang Bửu 08 23.00 24.00 24.50
THPT Nguyễn Văn Linh 08 15.50 16.25 17.25
THPT chuyên NKTDTT Nguyễn Thị Định 08 13.00 13.00 13.00
THPT Nguyễn Khuyến 10 32.50 33.25 33.25
THPT Nguyễn Du 10 35.50 36.25 37.25
THPT Nguyễn An Ninh 10 23.75 24.75 25.25
THPT Diên Hồng 10 23.50 24.00 25.00
THPT Sương Nguyệt Anh 10 22.75 23.75 24.50
THPT Nguyễn Hiền 11 32.00 32.75 32.75
THPT Trần Quang Khải 11 27.75 28.75 28.75
THPT Nam Kỳ Khởi Nghĩa 11 25.75 26.75 26.75
THPT Võ Trường Toản 12 28.75 29.75 30.75
THPT Trường Chinh 12 26.75 27.50 28.00
THPT Thạnh Lộc 12 24.25 24.50 24.50
12
THPT Thanh Đa Bình Thạnh 23.50 24.50 25.50
THPT Võ Thị Sáu Bình Thạnh 31.75 32.50 32.50
THPT Gia Định Bình Thạnh 35.00 35.25 35.75
THPT Phan Đăng Lưu Bình Thạnh 26.50 27.25 27.50
THPT Hoàng Hoa Thám Bình Thạnh 28.00 29.00 29.50
THPT Gò Vấp Gò Vấp 29.25 30.00 30.00
THPT Nguyễn Công Trứ Gò Vấp 35.25 36.25 37.25
THPT Trần Hưng Đạo Gò Vấp 30.75 31.50 32.25
THPT Nguyễn Trung Trực Gò Vấp 24.25 25.25 26.00
THPT Phú Nhuận Phú Nhuận 35.75 36.00 37.00
THPT Hàn Thuyên Phú Nhuận 23.25 24.25 24.75
THPT Nguyễn Chí Thanh Tân Bình 32.00 32.25 32.25
THPT Nguyễn Thượng Hiền Tân Bình 39.00 40.00 41.00
THPT Lý Tự Trọng Tân Bình 25.25 25.75 26.50
THPT Nguyễn Thái Bình Tân Bình 27.25 28.00 28.00
THPT Long Thới Nhà Bè 13.00 13.00 13.00
THPT Phước Kiển Nhà Bè 13.00 13.00 13.00
THPT Tân Bình Tân Phú 29.75 30.50 30.50
THPT Trần Phú Tân Phú 35.25 35.25 36.25
THPT Tây Thạnh Tân Phú 26.75 27.50 28.50
13
4. Bí quyết đạt "điểm rơi phong độ" ngay trong ngày thi
TTO - Làm thế nào để đạt kết quả tốt nhất có thể tương ứng với khả năng của mình khi tham gia
những kỳ thi quan trọng? Để làm được điều đó, các bạn cần quan tâm đến một khái niệm được gọi là “điểm
rơi phong độ”.
Phong độ học tập của các bạn sẽ có lúc lên “cao chót vót” (những lúc tập trung cao độ hoặc những lúc
các bạn dành nhiều thời gian đầu tư học tập) nhưng cũng có lúc phong độ “rơi tự do” (vì bận xem những bộ
phim hay, những chuyến đi chơi...).
"Điểm rơi phong độ" là thời điểm “phong độ” của mình đạt ở đỉnh cao nhất, điều đó có nghĩa là các
bạn cần “canh” làm sao mà ngay ngày thi, “phong độ” học tập của các bạn đạt ở mức “thượng thừa”. Chắc
hẳn các bạn tự hỏi làm sao để phong độ đạt ở đỉnh điểm cao nhất khi kỳ thi diễn ra. Các bạn cần quan
tâm các yếu tố sau:
Tâm lý phải vững vàng
Trong kết quả khảo sát bỏ túi gần đây do Trung tâm đào tạo kỹ năng sống Ý Tưởng Việt thực hiện với
hơn 2.000 học sinh lớp 12 tại TP.HCM với câu hỏi: “Hai tuần trước kỳ thi đại học, bạn làm gì?” thì có hơn
90% học sinh trả lời: “Em sẽ nghỉ ngơi, thư giãn”. Nhưng khi khảo sát thực tế lại có hơn 85% trong số các
bạn đó “cắm đầu cắm cổ” học ngày học đêm đến nỗi mắt thâm quầng, mặt mày xanh như... tàu là chuối và
không còn chút tâm lý nào để bước vào kỳ thi.
Hai yếu tố chính ảnh hưởng đến tâm lý thí sinh trước kỳ thi.
Thứ nhất: áp lực từ gia đình, bắt buộc thí sinh phải thi đậu để nở mày nở mặt với gia đình, với bà con
chòm xóm.
Thứ hai: áp lực do chính thí sinh tạo ra vì ganh đua bạn bè, với suy nghĩ: "Bạn mình đậu mà mình
không đậu thì quê lắm".
Chính những áp lực này khiến thí sinh như mang trên vai mình một tảng đá lớn và mang tâm lý nặng
nề vào phòng thi, khiến chất lượng bài thi thấp hơn rất nhiều so với khả năng thực tế.
Bí quyết có tâm lý thoải mái trước ngày "xung trận"
Thứ nhất: tư duy tích cực. Thay vì lo sợ: "Chết rồi, mai là ngày thi, phải làm sao đây?” thì hãy nghĩ:
“Mình rất tự tin chào đón kỳ thi! Vũ khí của mình là kiến thức. Đề có thế nào thì mình cũng có thể đối phó
được”.
Thứ hai: thí sinh hãy dẹp bỏ tư tưởng: "Đã thi là phải đậu, không đậu thì mình sẽ... chết". Thay vào đó,
hãy suy nghĩ sẽ cố gắng hết khả năng. Như vậy, tâm lý bạn sẽ nhẹ nhàng và thoải mái hơn khi đối mặt với kỳ
thi.
Những bí quyết nho nhỏ cho sức khỏe ngày thi
Thứ nhất: để đảm bảo dạ dày và hệ tiêu hóa hoạt động tốt, ba ngày trước ngày thi thí sinh cần chọn ăn
đúng một món ăn sáng bạn thích nhất như: cơm chiên, phở, hủ tiếu… nhưng chỉ nên ăn đúng một loại thức
ăn và ở đúng một quán để cơ thể thích nghi tốt nhất.
Vào ngày thi, nhiều thí sinh ra quán ăn gần điểm thi ăn đại món nào đó lót bụng, khiến không ít bạn
vào phòng thi mà mặt mày nhăn nhó, tay ôm bụng vì cơn đau quằn quại.
14
Thứ hai: thí sinh cần tập thói quen dậy sớm trước ngày thi ít nhất ba ngày để đồng hồ sinh học tự điều
chỉnh thích nghi với thời gian đó. Có những trường hợp các bạn thí sinh đành ngậm ngùi tiếc nuối chỉ vì đến
hội đồng thi trễ 15 phút.
Các bí quyết hữu ích trong phòng thi
Khi đi thi, ngoài những vật dụng cần thiết phục vụ việc làm bài thì thí sinh còn cần mang theo
những vật gì?
Thứ nhất: thời gian làm bài thi khá dài dễ gây mất năng lượng, chính vì vậy việc mang theo một
thanh chocolate để bổ sung năng lượng có thể là ý tưởng nghe khá lạ nhưng rất thực tế. Chocolate có
nhiều chất đường, cung cấp năng lượng cấp thời cho não, bên cạnh đó còn có chất cafein giúp bạn giữ
tỉnh táo. Xem xét kỹ quy chế thi hiện nay thì không có khoản nào cấm thí sinh mang kẹo vào phòng thi.
Thứ hai: thí sinh cần mang theo đủ nước để uống. Theo các chuyên gia tâm lý, rất có thể nước làm
tăng tốc độ truyền thông tin giữa các tế bào não. Một khả năng khác là những người uống đủ nước sẽ
không bị phân tán tư tưởng bởi cảm giác khát nước mà nhờ đó có thể tập trung giải quyết bài thi. Nhiều
nghiên cứu cũng chứng minh rằng não người trưởng thành hoạt động hiệu quả hơn sau khi họ uống
nước đầy đủ.
Thứ ba: khi nào nên xin tiếp giấy thi? Hầu hết thí sinh trả lời đó là khi viết hết tờ giấy thi hiện có.
Nếu làm vậy thì dòng suy nghĩ sẽ bị cắt đứt trong khi chờ giám thị ký tên rồi mang giấy thi đến. Thay
vào đó, khi viết đến trang thứ 4 của tờ giấy thi, bạn hãy nhanh chóng giơ tay xin tờ khác để có thể sử
dụng ngay khi vừa viết xong và dòng suy nghĩ không bị ngắt quãng.
Thứ tư: bạn nhớ mang theo đồng hồ để luôn kiểm soát sát sao lượng thời gian đã trôi qua, đồng
thời tránh quay qua hỏi thí sinh khác hay hỏi giám thị. Một ít khăn giấy cũng sẽ rất cần thiết nếu bạn ra
mồ hôi nhiều hay nhằm lúc trời mưa, làm đổ nước khi uống...
Chuyên viên tâm lý ĐÀO LÊ HÒA AN (theo www.tuoitre.vn)
Moïi yù kieán ñoùng goùp xin vui loøng lieân heä ban bieân taäp:
DÑ: 098 9829820 – 0953 474 474
Email: chuyenluyenthi@yahoo.com
15
Đề thi tuyển sinh lớp 10 thành phố Hồ Chí Minh
5. Đề thi tuyển sinh lớp 10 tp Hồ Chí Minh Năm học 2011-2012
Bài 1: (2 điểm) Giải các phương trình và hệ phương trình sau:
a) 2
3 2 1 0
x x
− − = b)
5 7 3
5 4 8
x y
x y
+ =


− = −

c) 4 2
5 36 0
x x
+ − = d) 2
3 5 3 3 0
x x
+ + − =
Bài 2: (1,5 điểm)
a) Vẽ đồ thị (P) của hàm số 2
y x
= − và đường thẳng (D): 2 3
y x
= − − trên cùng một hệ trục toạ độ.
b) Tìm toạ độ các giao điểm của (P) và (D) ở câu trên bằng phép tính.
Bài 3: (1,5 điểm) Thu gọn các biểu thức sau:
3 3 4 3 4
2 3 1 5 2 3
A
− +
= −
+ −
2 28 4 8
3 4 1 4
x x x x x
B
x x x x
− + − +
= − +
− − + −
( 0, 16)
x x
≥ ≠
Bài 4: (1,5 điểm) Cho phương trình 2
2 4 5 0
x mx m
− − − = (x là ẩn số)
a) Chứng minh rằng phương trình luôn luôn có nghiệm với mọi m.
b) Gọi x1, x2 là các nghiệm của phương trình. Tìm m để A = 2 2
1 2 1 2
x x x x
+ − đạt giá trị nhỏ nhất.
Bài 5: (3,5 điểm) Cho đường tròn (O) có tâm O, đường kính BC. Lấy một điểm A trên đường tròn (O)
sao cho AB > AC. Từ A, vẽ AH vuông góc với BC (H thuộc BC). Từ H, vẽ HE vuông góc với AB và HF
vuông góc với AC (E thuộc AB, F thuộc AC).
a) Chứng minh rằng AEHF là hình chữ nhật và OA vuông góc với EF.
b) Đường thẳng EF cắt đường tròn (O) tại P và Q (E nằm giữa P và F). Chứng minh AP2
= AE.AB.
Suy ra APH là tam giác cân.
c) Gọi D là giao điểm của PQ và BC; K là giao điểm cùa AD và đường tròn (O) (K khác A). Chứng
minh AEFK là một tứ giác nội tiếp. Gọi I là giao điểm của KF và BC. Chứng minh IH2
= IC.ID.
6. Đề thi tuyển sinh lớp 10 tp Hồ Chí Minh Năm học 2010-2011
Bài 1: (2 điểm) Giải các phương trình và hệ phương trình sau:
a) 2
2 3 2 0
x x
− − = b)
4 1
6 2 9
x y
x y
+ = −


− =

c) 4 2
4 13 3 0
x x
− + = d) 2
2 2 2 1 0
x x
− − =
Bài 2: (1,5 điểm)
a) Vẽ đồ thị (P) của hàm số
2
2
x
y = − và đường thẳng (D):
1
1
2
y x
= − trên cùng một hệ trục toạ độ.
b) Tìm toạ độ các giao điểm của (P) và (D) bằng phép tính.
16
Bài 3: (1,5 điểm) Thu gọn các biểu thức sau:
12 6 3 21 12 3
A = − + −
2 2
5 3
5 2 3 3 5 2 3 3 5
2 2
B
   
= + + − − + − + + −
   
   
   
Bài 4: (1,5 điểm) Cho phương trình 2 2
(3 1) 2 1 0
x m x m m
− + + + − = (x là ẩn số)
a) Chứng minh rằng phương trình luôn luôn có 2 nghiệm phân biệt với mọi giá trị của m.
b) Gọi x1, x2 là các nghiệm của phương trình. Tìm m để biểu thức sau đạt giá trị lớn nhất:
A = 2 2
1 2 1 2
3
x x x x
+ − .
Bài 5: (3,5 điểm) Cho đường tròn tâm O đường kính AB=2R. Gọi M là một điểm bất kỳ thuộc đường
tròn (O) khác Avà B.Các tiếp tuyến của (O) tại A và M cắt nhau tại E. Vẽ MP vuông góc với AB (P thuộc
AB), vẽ MQ vuông góc với AE (Q thuộc AE).
a) Chứng minh rằng AEMO là tứ giác nội tiếp đường tròn và APMQ là hình chữ nhật.
b) Gọi I là trung điểm của PQ. Chứng minh O, I, E thẳng hàng.
c) Gọi K là giao điểm của EB và MP. Chứng minh hai tam giác EAO và MPB đồng dạng.
Suy ra K là trung điểm của MP.
d) Đặt AP = x. Tính MP theo R và x. Tìm vị trí của M trên (O) để hình chữ nhật APMQ có diện tích
lớn nhất.
7. Đề thi tuyển sinh lớp 10 tp Hồ Chí Minh Năm học 2009-2010
Câu 1: Giải các phương trình và hệ phương trình sau:
a) 2
8 2 1 0
x x
− − = ; b)
2 3 3
5 6 12
x y
x y
+ =


− =

; c) 4 2
2 3 0
x x
− − = ; d) 2
3 2 6 2 0
x x
− + = .
Câu 2:
a) Vẽ đồ thị (P) của hàm số
2
2
x
y = và đường thẳng (D): y = x + 4 trên cùng một hệ trục toạ độ.
b) Tìm toạ độ giao điểm của (P) và (D) bằng phép tính.
Câu 3: Thu gọn biểu thức sau:
4 8 15
3 5 1 5 5
A = − +
+ +
:
1
1 1
x y x y x xy
B
xy
xy xy
 
+ −  
+
= +
   
  −
− +  
 
Câu 4: Cho phương trình ( )
2 2
5 1 6 2 0
x m x m m
− − + − = (m là tham số)
a) Chứng minh phương trình luôn có nghiệm với mọi m;
17
b) Gọi 1 2
;
x x là nghiệm của phương trình. Tìm m để 2 2
1 2 1
x x
+ = .
Câu 5: Cho tam giác ABC (AB < AC) có ba góc nhọn nội tiếp đường tròn (O) có tâm O, bán kính R.
Gọi H là giao điểm của ba đường cao AD, BE, CF của tam giác ABC. Gọi S là diện tích tam giác ABC.
a) Chứng minh rằng AEHF và AEDB là các tứ giác nội tiếp đường tròn.
b) Vẽ đường kính AK của đường tròn (O). Chứng minh tam giác ABD và tam giác AKC đồng dạng
với nhau. Suy ra AB.AC = 2R.AD và S =
AB.BC.CA
4R
.
c) Gọi M là trung điểm của BC. Chứng minh EFDM là tứ giác nội tiếp đường tròn.
d) Chứng minh rằng OC vuông góc với DE và (DE + EF + FD).R = 2S.
8. Đề thi tuyển sinh lớp 10 tp Hồ Chí Minh Năm học 2008-2009
Bài 1: (2 điểm) Giải các phương trình và hệ phương trình sau:
a) 2
2 3 5 0
x x
+ − = b) 4 2
3 4 0
x x
− − = c)
2 1
3 4 1
x y
x y
+ =


+ = −

Bài 2: (2 điểm)
a) Vẽ đồ thị (P) của hàm số 2
y x
= − và đường thẳng (D): 2
y x
= − trên cùng một hệ trục.
b) Tìm tọa độ các giao điểm của (P) và (D) ở câu trên bằng phép tính.
Bài 3: (1 điểm) Thu gọn các biểu thức sau:
a) 7 4 3 7 4 3
A = − + +
b)
1 1 2 4 8
.
4 4 4
x x x x x x
B
x x x x
 
+ − + − −
= −
 
 
− + +
 
với 0; 4
x x
> ≠
Bài 4: (1, 5 điểm) Cho phương trình 2
2 1 0
x mx
− − =
a) Chứng minh phương trình trên luôn có 2 nghiệm phân biệt.
b) Gọi 1 2
,
x x là hai nghiệm của phương trình trên. Tìm m để 2 2
1 2 1 2 7
x x x x
+ − = .
Bài 5: (3,5 điểm) Từ điểm M bên ngoài đường tròn (O) vẽ cát tuyến MCD không đi qua tâm O
và hai tiếp tuyến MA, MB đến đường tròn (O), A, B là các tiếp điểm và C nằm giữa M, D.
a) Chứng minh 2
.
MA MC MD
=
b) Gọi I là trung điểm của CD. Chứng minh rằng 5 điểm M, A, O, I, B cùng nằm trên một
đường tròn.
18
c) Gọi H là giao điểm của AB và MO. Chứng minh tứ giác CHOD nội tiếp được đường tròn.
Suy ra AB là đường phân giác của góc CHD.
d) Gọi K là giao điểm của các tiếp tuyến tại C và D của đường tròn (O). Chứng minh A, B, K
thẳng hàng.
9. Đề thi tuyển sinh lớp 10 tp Hồ Chí Minh Năm học 2007-2008
Bài 1.(1.5 điểm) Giải các phương trình và hệ phương trình sau:
a) 2
2 5 4 0
x x
− + = b) 4 2
29 100 0
x x
− + = c)
5 6 17
9 7
x y
x y
+ =


− =

Bài 2.(1.5 điểm) Thu gọn các biểu thức sau
a)
4 2 3
6 2
A
−
=
−
b) ( )
3 2 6 6 3 3
B = + −
Bài 3.(1 điểm) Một khu vườn hình chữ nhật có diện tích bằng 675m2
và có chu vi bằng 120m.
Tìm chiều dài và chiều rộng của khu vườn đó.
Bài 4.(2 điểm) Cho phương trình 2 2
2 1 0
x mx m m
− + − + = với m là tham số và x là ẩn số.
a)Giải phương trình với m = 1.
b)Tìm m để phương trình có hai nghiệm phân biệt 1 2
,
x x .
c)Với điều kiện của câu b) hãy tìm m để biểu thức 1 2 1 2
A x x x x
= − − đạt giá trị nhỏ nhất.
Bài 5.(4 điểm) Cho tam giác ABC có ba góc nhọn (AB < AC). Đường tròn đường kính BC cắt
AB, AC theo thứ tự tại E và F. Biết EF cắt CE tại H và AH cắt BC tại D.
a)Chứng minh tứ giác BEFC nội tiếp và AH vuông góc với BC.
b)Chứng minh AE.AB = AF.AC
c)Gọi O là tâm đường tròn ngoại tiếp tam giác ABC và K là trung điểm của BC. Tính tỉ số
OK
BC
khi tứ giác BHOC nội tiếp.
d) Cho HF = 3cm, HB = 4cm, CE = 8cm và HC > HE. Tính HC.
10. Đề thi tuyển sinh lớp 10 tp Hồ Chí Minh Năm học 2006-2007
Bài 1. Giải phương trình và hệ phương trình sau:
19
a)
3 2 1
5 3 4
x y
x y
+ =


+ = −

b) 2
2 2 3 3 0
x x
+ − = c) 4 2
9 8 1 0
x x
+ − =
Bài 2. Thu gọn biểu thức
15 12 1
5 2 2 3
A
−
= −
− −
;
2 2 4
.
2 2
a a
B a
a a a
 
− +  
= − −
   
 
+ −  
 
, với 0; 4
a a
> ≠ .
Bài 3. Cho mãnh đất hình chữ nhật có diện tích 360m2
. Nếu tăng chiều rộng 2m và giảm chiều
dài đi 6m thì diện tích mãnh đất không đổi. Tính chu vi mãnh đất lúc ban đầu.
Bài 4.
a)Viết phương trình đường thẳng (d) song song với đường thẳng y = 3x + 1 và cắt trục tung
tại điểm có tung độ bằng 4.
b) Vẽ đồ thị hàm số y = 3x + 4 và
2
2
x
y = − trên cùng một hệ trục toạ độ. Tìm toạ độ giao
điểm của hai đồ thị ấy bằng phép tính.
Bài 5. Cho tam giác ABC có ba góc nhọn và AB < AC. Đường tròn tâm O đường kính BC cắt
cạnh AB, AC theo thứ tự tại E và D.
a)Chứng minh rằng AD. AC = AE. AB.
b)Gọi H là giao điểm của BD và CE, gọi K là giao điểm của AH và BC. Chứng minh rằng AH
vuông góc với BC.
c)Từ A kẻ tiếp tuyến AM, AN đến (O) với M, N là các tiếp điểm. Chứng minh  
ANM AKN
= .
d)Chứng minh rằng M, H, N thẳng hàng.
II. Đề thi tuyển sinh lớp 10 Chuyên thành phố Hồ Chí Minh
1. Chuyên tp Hồ Chí Minh, năm học 2011-2012(N/A)
2. Chuyên tp Hồ Chí Minh, năm học 2010-2011(N/A)
3. Chuyên tp Hồ Chí Minh, năm học 2009-2010
Bài 1:(4 điểm)
1)Giải hệ phương trình 2 2
1
2
x y xy
x y xy
− − = −


− =

.
20
2)Cho phương trình 2 2
2 16 5 0
x mx m
− − + = ( x là ẩn số).
a)Tìm m để phương trình có nghiệm.
b)Gọi 1 2
,
x x là các nghiệm của phương trình. Tìm giá trị lớn nhất và giá trị nhỏ nhất
của biểu thức ( ) ( )
1 1 2 2 2 1
5 3 17 5 3 17
A x x x x x x
= + − + + − .
Bài 2:(4 điểm)
1)Thu gọn biểu thức
45 27 2 45 27 2 3 2 3 2
5 3 2 5 3 2 3 2 3 2
A
+ + − + + −
= −
+ − − + − −
2)Cho , ,
x y z là ba số dương thỏa điều kiện 2
xyz = . Tính giá trị biểu thức
2
2 1 2 2
x y x
B
xy x yz y zx z
= + +
+ + + + + +
Câu 3:(2 điểm)
1) Cho ba số thực , ,
a b c . Chứng minh rằng
( ) ( ) ( )
2 2 2
2 2 2
26 6 2009
a b b c c a
a b c ab bc ca
− − −
+ + ≥ + + + − +
2)Cho 0
a  và 0
b  . Chứng minh
1 2 8
2
a b a b
≥ +
−
.
Câu 4:(2 điểm)
1)Cho hệ phương trình
5
5
ax by
bx ay
+ =


+ =

(a,b là nguyên dương và a khác b).
Tìm a, b để hệ có nghiệm (x; y) với x, y là các số nguyên dương.
2)Chứng minh rằng không tồn tại các số nguyên x, y, z thỏa hệ:
2 2 2
2 2
3 3 31
8 100
x xy y z
x xy z
 − + − =


+ + =


.
Câu 5:(3 điểm) Cho ABCD là hình thoi có cạnh bằng 1. Giả sử tồn tại điểm M thuộc cạnh BC
và N thuộc cạnh CD sao cho tam giác CMN có chu vi bằng 2 và  
2
BAD MAN
= . Tính các góc của
hình thoi ABCD.
Câu 7:(2 điểm) Cho a, b là các số dương thỏa
2
1
1 1
a b
a b
+ =
+ +
. Chứng minh 2 1
8
ab ≤ .
21
4. Chuyên tp Hồ Chí Minh, năm học 2008-2009(N/A)
5. Chuyên tp Hồ Chí Minh, năm học 2007-2008
Câu 1:(4 điểm)
a)Chứng minh với mọi số thực x, y, z, t ta luôn có bất đẳng thức sau:
( )
2 2 2 2
x y z t x y z t
+ + + ≥ + + . Đẳng thức xảy ra khi nào?
b)Chứng minh với mọi số thực a, b khác không ta luôn có bất đẳng thức sau:
2 2
2 2
4 3
a b a b
b a b a
 
+ + ≥ +
 
 
Câu 2:(2 điểm) Tìm nghiệm nguyên của phương trình sau: 2
6 5 8
x xy x y
− = − −
Câu 3:(4 điểm) Cho hệ phương trình
( )( )
2 2
2 2 11
2 2
x y x y
xy x y m
 + + + =


+ + =


a)Giải hệ phương trình khi 24
m = .
b)Tìm m để hệ phương trình có nghiệm.
Câu 4: (2điểm) Cho ( )( )
2 2
2007 2007 2007
x x y y
+ + + + = . Tính S x y
= + .
Câu 5:(2 điểm) Cho a, b là các số nguyên dương sao cho
1 1
a b
b b
+ +
+ cũng là số nguyên.
Gọi d là ước số chung của a và b. Chứng minh d a b
≤ + .
Câu 6:(6 điểm) Cho tam giác ABC có ba góc nhọn nội tiếp trong đường tròn (O) (AB  AC).
Các tiếp tuyến với (O) tại B và C cắt nhau tại N. Vẽ dây AM song song với BC. Đường thẳng
MN cắt đường tròn (O) tại M và P.
a)Cho biết 2 2
1 1 1
16
OB NC
+ = , tính độ dài đoạn BC.
b)Chứng minh
BP CP
AC AB
= .
c)Chứng minh BC, ON và AP đồng qui tại một điểm.
6. Chuyên tp Hồ Chí Minh, năm học 2006-2007
Câu 1:(2 điểm) Thu gọn các biểu thức sau:
22
( )
2 4 6 2 5 10 2
A
 
= + − −
 
 
2
1 1 2
1 , 0, 1
1
1 1
a a
B a a
a
a a
 
− +  
= + −  ≠
  
  +
+ −  
 
Câu 2:(1 điểm) Với giá trị nào của m thì đường thẳng ( )
3
: 2
2
d y x m
= + cắt Parabol
( ) 2
3
:
4
P y x
= − tại hai điểm phân biệt?
Câu 3:(1,5 điểm) Giải các phương trình và hệ phương trình:
a) 2
5 1
x x
− = −
b)
3 4
2
4 5
3
x y
x y

− =



 − =


c) 2
4 2 2 8 5 2 3
x x x x
− + − + − + − = +
Câu 5:(4 điểm)
a)Cho hai số dương x, y thoả 3
x y xy
+ = . Tính
x
y
.
b)Tìm các số nguyên dương ,
x y thoả
1 1 1
2
x y
+ = .
Câu 5:(4 điểm) Cho tam giác ABC có ba góc nhọn (AB  AC), có đường cao AH.
Gọi D, E lần lượt là trung điểm AB và AC.
a)Chứng minh DE là tiếp tuyến chung của hai đường tròn ngoại tiếp hai tam giác DBH và
ECH.
b)Gọi F là giao điểm thứ nhì của hai đường tròn ngoại tiếp hai tam giác DBH và ECH.
Chứng minh HF đi qua trung điểm của DE.
c)Chứng minh rằng đường tròn ngoại tiếp tam giác ADE đi qua điểm F.
7. Chuyên Lê Hồng Phong tp Hồ Chí Minh, năm học 2001-2002
Bài 1: Cho phương trình ( ) ( )
2
1 2 2 3 0
m x m x m
+ − + + − = .
23
a)Định m để phương trình có nghiệm.
b)Định m để phương trình có hai nghiệm 1 2
,
x x thỏa mãn ( )( )
1 2
4 1 4 1 18
x x
+ + = .
Bài 2: Chứng minh các bất đẳng thức sau
a) 2 2 2
a b c ab bc ca
+ + ≥ + + , với mọi , ,
a b c .
b)
8 8 8
3 3 3
1 1 1
a b c
a b c a b c
+ +
≥ + + , với ( )
0, 0, 0
a b c
   .
c) ( )
2 2 2 2 2
a b c d e a b c d e
+ + + + ≥ + + + , với mọi , , , ,
a b c d e.
Bài 3: Giải các phương trình sau
a) 2 2
8
1
x
x
x
+ =
−
b) 2 2
4 5
1
8 7 10 7
x x
x x x x
+ = −
− + − +
Bài 4: Cho tam giác ABC có ba góc nhọn nội tiếp trong đường tròn tâm O và có trực tâm H.
Lấy M thuộc cung nhỏ BC.
a)Xác định vị trí điểm M sao cho BHCM là một hình bình hành.
b)Với M lấy bất kỳ thuộc cung nhỏ BC, gọi N, E lần lượt là các điểm đối xứng của M qua
AB, AC. Chứng minh rằng N, H, E thẳng hàng.
c)Xác định vị trí của M thuộc cung nhỏ BC sao cho NE có độ dài lớn nhất.
Bài 5: cho đường tròn cố định tâm O, bán kính bằng 1. Tam giác ABC thay đổi luôn ngoại tiếp
đường tròn (O). Một đường thẳng đi qua tâm O và cắt các cạnh AB, AC lần lượt tại M, N. Xác định
giá trị nhỏ nhất của diện tích tam giác AMN.
8. Chuyên Lê Hồng Phong tp Hồ Chí Minh, năm học 2002-2003
Bài 1: Rút gọn biểu thức
a) 5 3 29 12 5
A = − − −
b)
8 4
4 2
3 4
2
x x
B
x x
+ +
=
+ +
Bài 2: Cho phương trình ( )
2
2 1 2 4 0
x m x m
− − + − = .
24
a)Chứng minh rằng phương trình có hai nghiệm phân biệt.
b)Gọi 1 2
,
x x là hai nghiệm của phương trình. Tìm giá trị nhỏ nhất của biểu thức 2 2
1 2
y x x
= + .
Bài 3:
a)Chứng minh:
( )
2
2 2
2
x y
x y
+
+ ≥
b)Chứng minh:
( )
4
4 4
8
x y
x y
+
+ ≥
c)Cho x, y  0 và x + y =1. Chứng minh rằng ( )
4 4 1
8 5
x y
xy
+ + ≥ .
Bài 4: Giải các phương trình sau
a) 3 4 1 8 6 1 5
x x x x
+ + − + + − − =
b) ( )( )( )( )
4 1 12 1 3 2 1 4
x x x x
+ − + + =
Bài 5: Cho đường tròn (O; R) và đường thẳng (d) không qua O cắt đường tròn (O) tại hai điểm
A, B. Từ một điểm di động trên đường thẳng (d) và ở ngoài (O), ta vẽ hai tiếp tuyến MN, MP với
đường tròn (O) (N, P là hai tiếp điểm)
a)Chứng minh rằng  
NMO NPO
= .
b)Chứng minh rằng đường tròn ngoại tiếp tam giác MNP đi qua một điểm cố định khi M lưu
động trên đường thẳng (d).
c)Xác định vị trí điểm M trên đường thẳng (d) sao cho tứ giác MNOP là hình vuông.
d)Chứng minh rằng tâm I của đường tròn nội tiếp tam giác MNP lưu động trên một đường cố
định khi M lưu động trên (d).
9. Chuyên Lê Hồng Phong tp Hồ Chí Minh, năm học 2003-2004
Bài 1: Cho phương trình 2
2 6 9 0
x mx m
− − − = .
a)Tìm m để phương trình có hai nghiệm phân biệt đều âm.
b)Gọi 1 2
,
x x là hai nghiệm của phương trình. Tìm m để có 2 2
1 2 13
x x
+ = .
Bài 2:
25
a)Cho 0, 0
x y
  và 1
x y
+ ≤ . Chứng minh rằng 2 2
1 1
4
x xy y xy
+ ≥
+ +
.
b)Tìm giá trị nhỏ nhất của biểu thức
2
3
2 2 7
A
x x
=
+ − +
.
Bài 3: Giải các hệ phương trình sau
a) 2 2
11
30
x y xy
x y xy
+ + =


+ =

b)
64
1 1 1
4
xy
x y
= −



− =


Bài 4: Chứng minh rằng nếu 2
a b
+  thì ít nhất một trong hai phương trình sau có nghiệm
2 2
2 0; 2 0
x ax b x bx a
+ + = + + =
Bài 5: Cho đường tròn tâm O đường kính AB. Gọi K là trung điểm của cung AB, M là điểm
lưu động trên cung nhỏ AK (M khác A và K). Lấy điểm N trên đoạn BM sao cho BM = AM.
a)Chứng minh rằng  
AMK BNK
= .
b)Chứng minh rằng tam giác MNK vuông cân.
c)Hai đường thẳng AM và OK cắt nhau tại D. Chứng minh MK là đường phân giác 
DMN .
Bài 6: Cho tam giác ABC có BC = a, CA = b, AB = c và có R là bán kính đường tròn ngoại
tiếp thoả mãn hệ thức ( )
R b c a bc
+ = . Hãy định dạng tam giác ABC.
10. Chuyên Lê Hồng Phong tp Hồ Chí Minh, năm học 2004-2005
I.Phần tự chọn: học sinh chọn một trong hai bài sau đây
Bài 1a: Cho phương trình ( )
2
3 1 2 18 0
x m x m
− + + − = .
a)Tìm m để phương trình có hai nghiệm phân biệt đều âm.
b)Gọi 1 2
,
x x là hai nghiệm của phương trình. Tìm m để có 1 2 5
x x
− ≤ .
Bài 1b: Rút gọn các biểu thức sau:
a)
2 2
1
1 1
x x x x
A x
x x x x
− +
= − + +
+ + − +
b)
2 2 1
1
2 1
x x x x x x
B
x
x x x
  
+ − + − −
= −
  
  
−
+ +
  
II.Phần bắt buộc:
Bài 2: Giải các phương trình sau
26
a) 2
3 4 2 2
x x x
+ − = − b)
( )
2
2
2
9
3 9 2
x
x
x
= +
− +
Bài 3:
a)Cho 1, 1
x y
≥ ≥ . Chứng minh rằng 1 1
x y y x xy
− + − ≤ .
b) 0, 0
x y
  và 1
x y
+ = . Tìm giá trị nhỏ nhất của biểu thức 2 2
1 1
1 1
A
x y
 
 
= − −
 
 
  
.
Bài 4: Tìm các số nguyên x, y thoả hệ
2
1 0
2 1 1 0
y x x
y x
 − − − ≥


− + + − ≤


.
Bài 5: Cho đường tròn tâm O. Từ M nằm ngoài đường tròn (O) vẽ các tiếp tuyến MC, MD với
(O) (C, D là các tiếp điểm). Vẽ các tiếp tuyến MAB không đi qua tâm O, A nằm giữa M và B. Tia
phân giác của góc 
ACB cắt AB tại E.
a)Chứng minh rằng MC = ME.
b)Chứng minh DE là phân giác của góc 
ABD.
c)Gọi I là trung điểm của đoạn AB. Chứng minh 5 điểm O, I, C, M, D cùng nằm trên một
đường tròn.
d)Chứng minh IM là phân giác của 
CID .
Bài 6: Cho hình thang ABCD có hai cạnh đáy là BC và AD (BC  AD). Trên tia đối của tia
CA lấy điểm P tùy ý. Đường thẳng qua P và trung điểm I của BC cắt AB tại M, đường thẳng qua P
và trung điểm J của AD cắt CD tại N. Chứng minh MN song song với AD.
11. Chuyên Đại học Sư Phạm tp Hồ Chí Minh, năm học 2006-2007(N/A)
12. Chuyên Trần Đại Nghĩa tp Hồ Chí Minh, năm học
III. Đề thi tuyển sinh lớp 10 trường Phổ Thông Năng Khiếu
1. Toán không chuyên trường Phổ Thông Năng Khiếu, năm học 2011-2012
Bài 1:(2,5 điểm) Cho phương trình ( ) ( )
2 2
2 3 0 1
x mx m x
− − − = .
a)Giải phương trình (1) khi m = 2.
b)Tìm m để 2 2
2 0
x mx m
− − = có hai nghiệm phân biệt 1 2
,
x x thoả 2 2 2
1 2
2 7 2
x x m
+ = + .
27
c)Chứng minh phương trình (1) luông có không quá hai nghiệm phân biệt.
Bài 2:(2 điểm)
a)Giải phương trình 2 5 2 1 6
x x x
+ + − = + − .
b)Giải hệ phương trình
2 2
2 1
1
x y y
xy x
 + = +

= +

.
Bài 3:(1,5 điểm)
a)Rút gọn biểu thức
( )
2
3 1
1 1
:
1
1 1
x x
x x
R
x
x x
 
− +
 
− −  
= +
   
 
−
− +
 
   
với 0
x ≥ và 1
x ≠ .
b)Chứng minh R  1.
Bài 4:(1 điểm) Một tổ mua nguyên liệu để tổ chức thuyết trình tại lớp hết 72.000 đồng, chi phí
được chia đều cho mỗi thành viên của tổ. Nếu tổ giảm bớt 2 người thì mỗi người phải đóng thêm
3.000 đồng. Hỏi số người của tổ?
Bài 5:(3 điểm) Cho tam giác ABC có  75o
BAC = ,  45o
BCA = , 2
AC a
= , AK vuông góc với
BC (K thuộc BC).
a)Tính độ dài các đoạn KC và AB theo a.
b)Gọi H là trực tâm và O là tâm đường tròn ngoại tiếp tam giác ABC. Tính góc 
HOC .
c)Đường tròn tâm I nội tiếp tam giác ABC. Tính bán kính đường tròn ngoại tiếp tam giác
HIO theo a.
2. Toán không chuyên trường Phổ Thông Năng Khiếu, năm học 2010-2011
Bài 1.(2,5 điểm)
a)Tìm m để phương trình 2
2 3 0
x x m
+ + − = có hai nghiệm phân biệt 1 2
,
x x thoả
( )
2
2 2
1 2 1 2 1 2
2 7
x x x x x x
+ + =
b)Giải phương trình ( )( )
2
9 4 6 9 2 5 9 4
x x x x x
− − + = − + −
Bài 2.(2 điểm)
a)Giải hệ phương trình
2 2
2 0
1
x y xy xy
xy x y
 + + =

+ − =

28
b)Rút gọn biểu thức ( )
1 2 1
: 1 , 1
1
1 1
a a a a a
A a a
a
a a
 
+ + +
= − + − 
 
  −
+ +
 
.
Bài 3.(1,5 điểm) Cho tam giác ABC vuông tại A, có chu vi bằng 30cm và diện tích bằng
30cm2
. Tính độ dài các cạnh của tam giác.
Bài 4.(1 điểm) Cho số tự nhiên n có hai chữ số, chữ số hàng chục là x, chữ số hàng đơn vị là
y (nghĩa là 0
x ≠ và 10
n x y
= + . Gọi
n
M
x y
=
+
.
a)Tìm n để M = 2.
b)Tìm n để M đạt giá trị nhỏ nhất.
Bài 5.(3 điểm)Cho hình chữ nhật ABCD có tâm O, cạnh AB = 3a,  30o
ABD = . Gọi G là trọng
tâm của tam giác AOD, AG cắt CD tại E.
a)Chứng minh tứ giác AOED nội tiếp được trong một đường tròn.
b)Cho DG cắt AB tại F. Tính diện tích tứ giác AFOE.
c)Đường tròn tâm J nội tiếp trong tam giác BCD tiếp với DB, CD tại I và K. Gọi H là giao
điểm của IK và AC. Tính 
IOJ và độ dài đoạn HE.
3. Toán không chuyên trường Phổ Thông Năng Khiếu, năm học 2009-2010
Bài 1.(2 điểm)
a)Giải phương trình bằng cách đặt ẩn số
5 4
t
x x
 
= −
 
 
: 2
2
400 5
35 24
4
x
x
x x
 
+ = + −
 
 
b)Cho phương trình ( )
2
3 1 2 3 0
mx m x m
+ + − + = . Tìm m để phương trình có hai nghiệm phân
biệt 1 2
,
x x thoả mãn 2 2
1 2 34
x x
+ = .
Bài 2.(2.5 điểm) Xét biểu thức
2 3 3 4 5
1 5 4 5
x x x x
R
x x x x
+ + + −
= − −
+ − − −
.
a)Rút gọn R.
b)Tìm số thực x để R  -2. Tìm số tự nhiên x là số chính phương sao cho R là số nguyên.
Bài 3.(2 điểm)
29
a)Giải hệ phương trình 2 2
0
8
x xy y
x y
+ + =


+ =

b)Cho a, b, c là độ dài ba cạnh của tam giác ABC. Giả sử phương trình
( )( ) ( )( ) ( )( ) 0
x a x b x b x c x c x a
− − + − − + − − = có nghiệm kép.
Tính số đo các góc tam giác ABC.
Bài 4.(1 điểm) cho tam giác ABC có  60o
ABC = ,  45o
ACB = . Dựng AH vuông góc với BC (H
thuộc BC), và dựng HK vuông góc với AB (K thuộc AB). Gọi M là trung điểm của AC. Biết
3
AH = , tính BC. Chứng minh rằng BKMC là tứ giác nội tiếp.
Bài 5.(1 điểm) Trong kỳ kiểm tra môn Toán một lớp gồm 3 tổ A, B, C điểm trung bình của
học sinh các tổ được thống kê ở bảng sau:
Tổ A B C A và B B và C
Điểm trung bình 9.0 8.8 7.8 8.9 8.2
Biết tổ A gồm 10 học sinh, hãy xác định số học sinh và điểm trung bình của toàn lớp.
Bài 6.(1 điểm) Cho tứ giác lồi ABCD nội tiếp trong (O), có đỉnh A cố định và các đỉnh B, C,
D di chuyển trên (O) sao cho  90o
BAD  . Kẻ tia Ax vuông góc với AD cắt BC tại E, kẻ tia Ay vuông
góc với AB cắt CD tại F. Gọi K là điểm đối xứng của A qua EF. Chứng minh tứ giác EFCK nội tiếp
được và đường thẳng EF luôn đi qua một điểm cố định.
4. Toán AB trường Phổ Thông Năng Khiếu, năm học 2008-2009
Bài 1: Cho phương trình ( )
2 2
2
2 1 6
2
x mx m
m x
x m
+ −
= − +
+
(1)
a)Giải phương trình khi m = 1.
b)Tìm tất cả các giá trị của m để phương trình (1) có nghiệm.
Bài 2:
a)Giải phương trình 2 1 2 1 1
x x
− − − = −
b)Giải hệ phương trình
2
2
2 2 4
2 4
x x y xy
x xy
 − + =


+ =


30
Bài 3:
a)Chứng minh rằng biểu thức sau không phụ thuộc vào biến x (x  1)
( )( )
( )( )( )
4 3 1
1 3
x x x x x x
A
x x x x x x
+ + −
=
− + + +
.
b)Cho a, b, c là các số thực thoả mãn a + 2b -3c = 0 và bc + 2ac – 3ab =0.
Chứng minh rằng a = b = c.
Bài 4: Cho tứ giác ABCD nội tiếp có góc A nhọn và hai đường chéo AC và BD vuông góc với
nhau tại M. P là trung điểm của CD, H là trực tâm của tam giác ABD.
a)Tính tỷ số
PM
DH
.
b)Gọi N, K lần lượt là chân đường cao hạ từ B và D của tam giác ABD, Q là giao điểm của
MK và BC. Chứng minh MN = MQ.
c)Chứng minh tứ giác BQNK nội tiếp.
Bài 5: Một nhóm học sinh định chia một số kẹo thành các phần quà cho các em nhỏ tại một
đơn vị trẻ em mồ côi. Nếu mỗi phần quà giảm đi 6 viên thì các em có thêm 5 phần quà, nếu giảm đi
10 viên thì các em có thêm 10 quà. Hỏi số kẹo mà nhóm học sinh này có.
5. Toán CD trường Phổ Thông Năng Khiếu, năm học 2008-2009
Bài 1:(2 điểm) Giả sử 1 2
,
x x là hai nghiệm của phương trình ( )
2
3 2 1 0
mx m x m
− + + + = .
a)Tính tổng S và tích P của hai nghiệm. Biết 1
S P
− = , tính 1 2
, ,
m x x .
b)Trong trường hợp tổng quát, không tính 1 2
,
x x hãy tìm một hệ thức liên hệ giữa S và P độc
lập với m.
Bài 2:(2 điểm)
a)Giải hệ phương trình
2 2
2 2
2 3 0,19
5 7 0,83
x y
x y
 − = −


+ =


, với 0
xy  .
b)Giải phương trình ( ) 2
2 1 1
x x x
+ − = .
31
Bài 3:(1 điểm) Một phân số P có dạng
3
a
P
a
=
+
với a là số nguyên dương. Nếu mẫu số tăng
thêm một thì phân số giảm đi
11
325
. Tính P.
Bài 4:(3 điểm) Cho tam giác ABC vuông tại A có AB = 6, AC =8.
a)Đường tròn nội tiếp tam giác ABC có tâm I, bán kính r. Tính IA.
b)Đường tròn ngoài tiếp tam giác ABC có tâm O, bán kính R. Tính
2
2
OI
R Rr
−
.
c)Dựng AH vuông góc với BC tại H và HK vuông góc với AC tại K. Đường tròn ngoại tiếp
tam giác BKC cắt AB tại E (E khác B). Tính 
AEH .
Bài 5:(2 điểm) Một sinh viên suýt bị tai nạn tàu hoả kể chuyệ với bạn: “Khi đang chạy trên cầu
sắt (sử dụng cho tàu hoả) dài 120m thì tôi thấy một chiếc xe lửa tốc hành lao tới.
Thay vì chạy cùng chiều với xe lửa tôi quyết định chạy nước rút nhược chiều xe lửa. Dù là vô
chạy nước rút của trường tôi, tôi cũng chỉ chạy được với vận tốc 18km/h vì đang mang ba lô trên
lưng. May mắn thay, khi tôi chạy đến đầu cầu bên kia thì 2 giây sau xe lửa mới đến nên tôi đã kịp
tránh sang một bên.
Khi hoàn hồn lại, tôi tính được rằng nếu tôi chạy cùng chiều với xe lửa thì ngay khi đến đầu
cầu bên kia thì tôi sẽ bị xe lửa đụng. May thật!”
Biết xe lửa chạy với vận tốc 72km/h, em hãy tính khi thấy xe lửa, anh sinh viên đang ở cách
đầu cầu bên này bao nhiêu mét?
6. Toán AB trường Phổ Thông Năng Khiếu, năm học 2007-2008
Bài 1. Cho phương trình
( ) ( )
2
2 2 1 3
0 1
1
x x m m m
x
− + + −
=
−
a)Tìm m để x = -1 là một nghiệm của phương trình (1).
b)Tìm m để phương trình (1) vô nghiệm.
Bài 2.
a)Giải bất phương trình ( )( ) 2
3 1 2 1 7
x x x x
+ − − −  −
32
b)Giải hệ phương trình
2 3 2 1
2 3 2 1
x y y x x x
y x x y y y
 + = −


+ = −


Bài 3.
a)Cho a, b là hai số thực thoả mãn điều kiện 2 2 2 2
3 2 2 5 7 0
a ab b a b a ab b a b
− − + − = − + − + = .
Chứng tỏ rằng 12 15 0
ab a b
− + = .
b)Cho
( )( )( )
( )
2 2
4 2 1 4 2 2 1
1
x x x x x x
A
x x x
+ − + + + + − +
=
−
.
Hãy tìm tất cả các giá trị của x để 0
A ≥ .
Bài 4. Cho tam giác ABC nhọn có trực tâm H và  60o
BAC = . Gọi M, N, P lần lượt là chân các
đường cao kẻ từ A, B, C của tam giác ABC và I là trung điểm của BC.
a)Chứng minh rằng tam giác INP đều.
b)Gọi E và K lần lượt là trung điểm PB và NC. Chứng minh rằng các điểm I, M, E, K cùng
thuộc một đường tròn.
c)Giả sử IA là phân giác của 
NIP . Hãy tính số đo của góc 
BCP .
Bài 5.Một công ty may giao cho tổ may A may 16800 sản phẩm, tổ B may 16500 sản phẩm và
bắt đầu thực hiện công việc cùng một lúc. Nếu sau 6 ngày, tổ A được hỗ trợ thêm 10 công
nhân may thì họ hoàn thành công việc cùng lúc với tổ B. Nếu tổ A được hỗ trợ thêm 10 công
nhân ngay từ đầu thì họ sẽ hoàn thành công việc sớm hơn tổ B một ngày. Hãy xác định số công
nhân ban đầu của mỗi tổ. Biết rằng mỗi công nhân mỗi ngày may được 20 sản phẩm.
7. Toán AB trường Phổ Thông Năng Khiếu, năm học 2006-2007(N/A)
8. Toán CD trường Phổ Thông Năng Khiếu, năm học 2006-2007(N/A)
9. Toán AB trường Phổ Thông Năng Khiếu, năm học 2005-2006(N/A)
10. Toán CD trường Phổ Thông Năng Khiếu, năm học 2005-2006(N/A)
Bài 1(2 điểm)
a)Gọi (d) là đường thẳng qua hai điểm A(0; -1) và B(1; -m-1). Tìm giá trị m để Parabol (P)
2
4
y mx mx
= + − tiếp xúc với đường thẳng (d).
33
b)Giải sử 1 2
,
x x là nghiệm cuả phương trình 2
2 3 0
mx mx
+ − = , tính 2 2
1 2
A x x
= + theo m.
Bài 2(2 điểm)
a)Với điều kiện xy  0, giải hệ phương trình
2 2
2 2
3 4 0,11
2 3 0,02
x y
x y
 − =


− =


b)Rút gọn biểu thức
3 5 3 5
2 3 5 2 3 5
R
+ −
= +
+ + − −
Bài 3(2 điểm)
a)Giải phương trình 2 2 15
4 4 6 9
2
x x x x x
− + + + + =
b)Tìm bảy số nguyên liên tiếp sao cho tổng bình phương bốn số đầu bằng tổng bình phương
ba số sau.
Bài 4(2 điểm)
Cho tam giác ABC có  45o
ACB = và   
2
ACB BAC ABC
+ = . Trung trực của AB cắt BC tại M.
a)Tính 
MAC .
b)Gọi I là tâm đường tròn ngoại tiếp tam giác AMC. Chứng minh tứ giác ABCI nội tiếp.
Bài 5 (2 điểm)
Một cuộc đua thuyền được tổ chức trên tuyến đường hình tam giác đều ABC (chạy từ A đến B,
từ B đến C rồi từ C về A). Chiếc thuyền “Bảy cây sứ trắng” tham dự cuộc đua và được ghi
nhận các thông tin sau đây: thuyền chạy từ
2
3
đoạn đường AB cho đến đích thì mất 3 giờ 15
phút; thuyền vượt đoạn BC nhanh hơn khi vượt đoạn CA 25 phút; thuyền chạy từ A đến
1
4
đoạn CA hết 2 giờ 40 phút. Giả sử rằng khi di chuyển trên mỗi cạnh, tốc độ của thuyền là
không đổi và thuyền đi rất thẳng; ngoài ra, thời gian để thuyền đổi hướng là không đáng kể
(khi vượt xong một cạnh nhưng chưa tới đích, thuyền đổi hướng chuyển sang cạnh kế). Tính
thời gian thuyền vượt toàn bộ đường đua.
11. Toán AB trường Phổ Thông Năng Khiếu, năm học 2004-2005
Bài 1.
a)Giải phương trình 4 3 2
x x
− − =
34
b)Định m để phương trình ( )
2
1 2 0
x m x m
− + + = có hai nghiệm phân biệt 1 2
,
x x sao cho 1 2
,
x x
là độ dài cạnh góc vuông của một tam giác vuông có cạnh huyền bằng 5.
Bài 2. Cho a, b là các số thực dương thoả mãn điều kiện ( ) ( ) ( )
2 2 2
2 2 2
a b c a b b c c a
+ + = − + − + −
a)Tính a b c
+ + biết rằng 9
ab bc ca
+ + = .
b)Chứng minh rằng nếu ,
c a c b
≥ ≥ thì c a b
≥ + .
Bài 3.Cùng một thời điểm, một chiếc ôtô A
X xuất phát từ thành phố A hướng về thành phố B
và một chiếc khác B
X xuất phát từ thành phố B hướng về thành phố A. Chúng chuyển động
với vận tốc riêng không đổi và gặp nhau lần đầu tại một điểm cách A là 20km. Cả hai chiếc xe,
sau khi đến B và A tương ứng, lập tức quay trở lại và chúng gặp nhau lần thứ hai tại một điểm
C. Biết thời gian xe B
X đi từ C đến B là 10 phút và thời gian giữa hai lần gặp nhau là 1 giờ,
hãy tính vận tốc của từng chiếc ôtô.
Bài 4.Gọi I, O lần lượt là tâm đường tròn nội tiếp và ngoại tiếp (C) của tam giác nhọn ABC.
Tia AI cắt đường tròn (C) tại K (K khác A) và J là điểm đối xứng của I qua K. Gọi P và Q lần
lượt là các điểm đối xứng của I và O qua BC.
a)Chứng minh rằng tam giác IBJ vuông tại B.
b)Tính góc BAC nếu Q thuộc đường tròn (C).
c)Chứng minh rằng nếu Q thuộc (C) thì P cũng thuộc (C).
Bài 5. Chứng minh rằng từ 8 số nguyên dương tuỳ ý không lớn hơn 20, luôn luôn chọn được
ba số x, y, z là độ dài ba cạnh của một tam giác.
12. Toán CD trường Phổ Thông Năng Khiếu, năm học 2004-2005
Bài 1.
a)Tìm m để Parabol (P) 2
2 2
y x mx m
= + − + tiếp xúc với đường thẳng (d) y x m
= + .
b)Giả sử phương trình ( )
2 2
2 1 1 0
mx m x m
+ + + − = có hai nghiệm phân biệt 1 2
,
x x . Hãy tính
tổng S và tích P của hai nghiệm, tìm một hệ thức liên hệ giữa S và P độc lập với m.
Bài 2.
35
a)Giải hệ phương trình 3 3
1
21
x y
x y
+ = −


+ = −

b)Giải phương trình 20 3 2 2 3
x x
− − = −
Bài 3.
a)Tìm k để đa thức ( ) 4 2
22 51 2
f x x x x k
= − + + chia hết cho đa thức ( ) 2
3 2
g x x x
= − + (nghĩa
là có đa thức ( )
h x sao cho ( ) ( ) ( )
f x h x g x
= . Giải phương trình ( ) 0
f x = với k vừa tìm
được.
b)Rút gọn biểu thức
2 2 2 2
2 2 2 2
3 2 3 4
:
2 3 2
a ab b a ab b
R
a ab b a ab b
− − − +
=
+ − + −
.
Bài 4. Cho tam giác ABC vuông ở đỉnh A và  75o
ABC = . Đường trung trực của BC cắt các
đường thẳng BC, AC và AB lần lượt tại các điểm M, N và P.
a)Tính
AN
NC
.
b)Gọi I là giao điểm của các đường thẳng BN và PC. So sánh MA và MI.
c)Lấy điểm Q trên đường thẳng vuông góc với mặt phẳng (ABC) tại B sao cho BQ = BI, QJ
vuông góc với PC, J nằm trên PC. Tính
QJ
AB
.
IV. Đề thi tuyển sinh lớp 10 trên toàn quốc
1. Tuyển sinh lớp 10 thành phố Đà Nẵng năm học 2011-2012
Bài 1: (2,0 điểm)
a)Giải phương trình: (2x + 1)(3-x) + 4 = 0 b)Giải hệ phương trình:
3 | | 1
5 3 11
x y
x y
− =


+ =

Bài 2: (1,0 điểm) Rút gọn biểu thức
6 3 5 5 2
( ): .
2 1 5 1 5 3
Q
− −
= +
− − −
Bài 3: (2,0 điểm) Cho phương trình x2
– 2x – 2m2
= 0 (m là tham số).
a) Giải phương trình khi m = 0
b) Tìm m để phương trình có hai nghiệm x1, x2 khác 0 và thỏa điều kiện 2 2
1 2
4
x x
= .
Bài 4: (1,5 điểm) Một hình chữ nhật có chu vi bằng 28 cm và mỗi đường chéo của nó có độ dài 10 cm. Tìm
độ dài các cạnh của hình chữ nhật đó.
Bài 5: (3,5 điểm)
Cho tam giác đều ABC nội tiếp đường tròn đường kính AD. Gọi M là một điểm di động trên cung
nhỏ AB ( M không trùng với các điểm A và B).
a) Chứng minh rằng MD là đường phân giác của góc BMC.
36
b) Cho AD = 2R. Tính diện tích của tứ giác ABDC theo R
c) Gọi K là giao điểm của AB và MD, H là giao điểm của AD và MC. Chứng minh rằng ba đường
thẳng AM, BD, HK đồng quy.
2. Tuyển sinh lớp 10 thành phố Đà Nẵng năm học 2010-2011
Bài 1 (2,0 điểm)
a) Rút gọn biểu thức A ( 20 45 3 5). 5
= − + b) Tính 2
B ( 3 1) 3
= − −
Bài 2 (2,0 điểm)
a) Giải phương trình 4 2
x 13x 30 0
− − = b) Giải hệ phương trình
3 1
7
x y
2 1
8
x y

− =



 − =


Bài 3 (2,5 điểm) Cho hai hàm số y = 2x2
có đồ thị (P) và y = x + 3 có đồ thị (d).
a) Vẽ các đồ thị (P) và (d) trên cùng một mặt phẳng tọa độ Oxy.
b) Gọi A là giao điểm của hai đồ thị (P) và (d) có hoành độ âm.
Viết phương trình của đường thẳng (∆) đi qua A và có hệ số góc bằng - 1.
c) Đường thẳng (∆) cắt trục tung tại C, cắt trục hoành tại D. Đường thẳng (d) cắt trục hoành tại B.
Tính tỉ số diện tích của hai tam giác ABC và tam giác ABD.
Bài 4 (3,5 điểm) Cho hai đường tròn (C) tâm O, bán kính R và đường tròn (C') tâm O', bán kính R'
(R  R') cắt nhau tại hai điểm A và B. Vẽ tiếp tuyến chung MN của hai đường tròn (M ∈ (C), N ∈
(C')). Đường thẳng AB cắt MN tại I (B nằm giữa A và I).
a) Chứng minh rằng  
BMN MAB
=
b) Chứng minh rằng IN2
= IA.IB
c) Đường thẳng MA cắt đường thẳng NB tại Q; đường thẳng NA cắt đường thẳng MB tại P.
Chứng minh rằng MN song song với QP.
3. Tuyển sinh lớp 10 thành phố Đà Nẵng năm học 2009-2010
Bài 1. ( 3 điểm ) Cho biểu thức
a 1 1 2
K :
a 1
a 1 a a a 1
   
= − +
   
−
− − +
 
 
a) Rút gọn biểu thức K. b) Tính giá trị của K khi a = 3 + 2 2
37
c) Tìm các giá trị của a sao cho K  0.
Bài 2. ( 2 điểm ) Cho hệ phương trình:
mx y 1
x y
334
2 3
− =



− =


a) Giải hệ phương trình khi cho m = 1.
b) Tìm giá trị của m để phương trình vô nghiệm.
Bài 3. ( 3,5 điểm ) Cho đường tròn (O), đường kính AB cố định, điểm I nằm giữa A và O sao
cho AI =
2
3
AO. Kẻ dây MN vuông góc với AB tại I. Gọi C là điểm tùy ý thuộc cung lớn MN
sao cho C không trùng với M, N và B. Nối AC cắt MN tại E.
a) Chứng minh tứ giác IECB nội tiếp được trong một đường tròn.
b) Chứng minh ∆AME ∆ACM và AM2
= AE.AC.
c) Chứng minh AE.AC - AI.IB = AI2
.
d) Hãy xác định vị trí của điểm C sao cho khoảng cách từ N đến tâm đường tròn ngoại tiếp
tam giác CME là nhỏ nhất.
Bài 4. ( 1,5 điểm ) Người ta rót đầy nước vào một chiếc ly hình nón thì được 8 cm3
. Sau đó
người ta rót nước từ ly ra để chiều cao mực nước chỉ còn lại một nửa. Hãy tính thể tích lượng
nước còn lại trong ly.
4. Tuyển sinh lớp 10 thành phố Hà Nội năm học 2010-2011
Bài I (2,5 điểm) Cho biểu thức : A =
2 3 9
9
3 3
x x x
x
x x
+
+ −
−
+ −
, với x≥ 0 và x≠ 9.
1) Rút gọn biểu thức A. 2) Tìm giá trị của x để A = 1/3
3) Tìm giá trị lớn nhất của biểu thức A.
Bài II (2,5 điểm)Giải bài toán sau bằng cách lập phương trình:
Một mảnh đất hình chữ nhật có độ dài đường chéo là 13 m và chiều dài lớn hơn chiều rộng 7 m.
Tính chiều dài và chiều rộng của mảnh đất đó.
Bài III (1,0 điểm) Cho parabol (P): y = -x2
và đường thẳng (d): y = mx – 1.
1) Chứng minh với mọi giá trị của m thì đường thẳng (d) luôn cắt parabol (P) tại hai điểm phân biệt.
38
2) Gọi x1, x2 lần lượt là hoành độ các giao điểm của đường thẳng (d) và parabol (P).
Tìm giá trị của m để: x1
2
x2 + x2
2
x1 – x1x2 = 3.
Bài IV (3,5 điểm)
Cho đường tròn (O) có đường kính AB = 2R và điểm C thuộc đường tròn đó (C khác A, B).
Lấy điểm D thuộc dây BC (D khác B, C). Tia AD cắt cung nhỏ BC tại điểm E, tia AC cắt tia BE tại
điểm F.
1) Chứng minh FCDE là tứ giác nội tiếp.
2) Chứng minh DA.DE = DB.DC.
3) Chứng minh 
CFD = 
OCB . Gọi I là tâm đường tròn ngoại tiếp tứ giác FCDE, chứng minh IC là
tiếp tuyến của đường tròn (O).
4) Cho biết DF = R, chứng minh tg 
AFB = 2.
Bài V ( 0,5 điểm) Giải phương trình: x2
+ 4x + 7 = (x + 4) 2
7
x +
5. Tuyển sinh lớp 10 thành phố Hà Nội năm học 2011-2012
Bài I (2,5 điểm) Cho
x 10 x 5
A
x 25
x 5 x 5
= − −
−
− +
Với x 0,x 25
≥ ≠ .
1) Rút gọn biểu thức A.
2) Tính giá trị của A khi x = 9.
3) Tìm x để
1
A
3
 .
Bài II (2,5 điểm) Giải bài toán sau bằng cách lập phương trình hoặc hệ phương trình:
Một đội xe theo kế hoạch chở hết 140 tấn hàng trong một số ngày quy định. Do mỗi ngày đội đó chở
vượt mức 5 tấn nên đội đã hoàn thành kế hoạch sớm hơn thời gian quy định 1 ngày và chở thêm được
10 tấn. Hỏi theo kế hoạch đội xe chở hàng hết bao nhiêu ngày?
Bài III (1,0 điểm) Cho Parabol (P):
2
y x
= và đường thẳng (d):
2
y 2x m 9
= − + .
1) Tìm toạ độ các giao điểm của Parabol (P) và đường thẳng (d) khi m = 1.
2) Tìm m để đường thẳng (d) cắt Parabol (P) tại hai điểm nằm về hai phía của trục tung.
Bài IV (3,5 điểm) Cho đường tròn tâm O, đường kính AB = 2R. Gọi d1 và d2 là hai tiếp tuyến của
đường tròn (O) tại hai điểm A và B.Gọi I là trung điểm của OA và E là điểm thuộc đường tròn (O) (E
39
không trùng với A và B). Đường thẳng d đi qua điểm E và vuông góc với EI cắt hai đường thẳng d1
và d2 lần lượt tại M, N.
1) Chứng minh AMEI là tứ giác nội tiếp.
2) Chứng minh ENI EBI
∠ = ∠ và
0
MIN 90
∠ = .
3) Chứng minh AM.BN = AI.BI .
4) Gọi F là điểm chính giữa của cung AB không chứa E của đường tròn (O).
Hãy tính diện tích của tam giác MIN theo R khi ba điểm E, I, F thẳng hàng.
Bài V (0,5 điểm) Với x  0, tìm giá trị nhỏ nhất của biểu thức:
2 1
M 4x 3x 2011
4x
= − + + .
6. Tuyển sinh lớp 10 tỉnh Hải Dương năm học 2011-2012
Câu 1 (3,0 điểm).
1)Giải các phương trình:
a)5( 1) 3 7
+ = +
x x b)
4 2 3 4
1 ( 1)
+
+ =
− −
x
x x x x
2)Cho hai đường thẳng (d1): 2 5
y x
= + ; (d2): 4 1
y x
= − − cắt nhau tại I. Tìm m để đường thẳng (d3):
( 1) 2 1
y m x m
= + + − đi qua điểm I.
Câu 2 (2,0 điểm). Cho phương trình: 2
2( 1) 2 0
x m x m
− + + = (1) (với ẩn là x ).
1)Giải phương trình (1) khi m=1.
2)Chứng minh phương trình (1) luôn có hai nghiệm phân biệt với mọi m.
3)Gọi hai nghiệm của phương trình (1) là 1
x ; 2
x . Tìm giá trị của m để 1
x ; 2
x là độ dài hai cạnh của
một tam giác vuông có cạnh huyền bằng 12 .
Câu 3 (1,0 điểm). Một hình chữ nhật có chu vi là 52 m. Nếu giảm mỗi cạnh đi 4 m thì được một hình
chữ nhật mới có diện tích 77 m2
. Tính các kích thước của hình chữ nhật ban đầu?
Câu 4 (3,0 điểm). Cho tam giác ABC có Â  900
. Vẽ đường tròn (O) đường kính AB và đường tròn
(O’) đường kính AC. Đường thẳng AB cắt đường tròn (O’) tại điểm thứ hai là D, đường thẳng AC
cắt đường tròn (O) tại điểm thứ hai là E.
1)Chứng minh bốn điểm B, C, D, E cùng nằm trên một đường tròn.
2)Gọi F là giao điểm của hai đường tròn (O) và (O’) (F khác A). Chứng minh ba điểm B, F, C thẳng
hàng và FA là phân giác của góc EFD.
3)Gọi H là giao điểm của AB và EF. Chứng minh BH.AD = AH.BD.
40
Câu 5 (1,0 điểm). Cho x, y, z là ba số dương thoả mãn x + y + z =3. Chứng minh rằng:
1
3 3 3
+ + ≤
+ + + + + +
x y z
x x yz y y zx z z xy
.
7. Tuyển sinh lớp 10 tỉnh Hà Tĩnh năm học 2010-2011
Câu I (3 điểm). Cho biểu thức A =
2 2
1
1 1
x
x
x x
− −
−
− +
.
1. Nêu điều kiện xác định và rút gọn biểu thức A.
2. Tính giá trị của biểu thức A khi x = 9.
3. Khi x thoả mãn điều kiện xác định. Hãy tìm giá trị nhỏ nhất của biểu thức B, với B = A(x – 1).
Câu II (2 điểm). Cho phương trình bậc hai sau, với tham số m: x2
– (m + 1)x + 2m – 2 = 0 (1)
1. Giải phương trình (1) khi m = 2.
2. Tìm giá trị của tham số m để x = -2 là một nghiệm của phương trình (1).
Câu III (1,5 điểm). Hai người cùng làm chung một công việc thì sau 4 giờ 30 phút họ làm xong. Nếu
một mình người thứ nhất làm trong 4 giờ, sau đó một mình người thứ hai làm trong 3 giờ thì cả hai
người làm được 75% công việc. Hỏi nếu mỗi người làm một mình thì sau bao lâu sẽ xong công việc?
(Biết rằng năng suất làm việc của mỗi người là không thay đổi).
Câu IV (3,5 điểm). Cho nửa đường tròn tâm O đường kính AB. Điểm H cố định thuộc đoạn thẳng AO
(H khác A và O). Đường thẳng đi qua điểm H và vuông góc với AO cắt nửa đường tròn (O) tại C.
Trên cung BC lấy điểm D bất kỳ (D khác B và C). Tiếp tuyến của nửa đường tròn (O) tại D cắt
đường thẳng HC tại E. Gọi I là giao điểm của AD và HC.
1. Chứng minh tứ giác HBDI nội tiếp đường tròn.
2. Chứng minh tam giác DEI là tam giác cân.
3. Gọi F là tâm đường tròn ngoại tiếp tam giác ICD. Chứng minh góc ABF có số đo không đổi khi
D thay đổi trên cung BC (D khác B và C).
8. Tuyển sinh lớp 10 tỉnh Nghệ An năm học 2011-2012
Câu I (3,0 điểm) Cho biểu thức A =
( )
2
1 1 1
:
1 1
x
x x x x
+
 
+
 
− −
  −
a) Nêu ĐKXĐ và rút gọn A.
b) Tìm giá trị của x để A =
1
3
.
41
c) Tìm giá trị lớn nhất của biểu thức P = A - 9 x .
Câu 2. (2,0 điểm) Cho phương trình bậc hai: x2
– 2(m + 2)x + m2
+ 7 = 0 (1), (m là tham số)
a) Giải phương trình (1) khi m = 1.
b) Tìm m để phương trình (1) có hai nghiệm x1, x2 thỏa mãn: x1x2 – 2(x1 + x2) = 4.
Câu 3(1,5 điểm) Quãng đường AB dài 120 km. Hai xe máy khởi hành cùng một lúc đi từ A đến B.
Vận tốc của xe thứ nhất lớn hơn vận tốc của xe thứ hai là 10 km/h nên xe máy thứ nhất đến B trước
xe thứ hai 1 giờ. Tính vận tốc của mỗi xe.
Câu 4. (3,5 điểm) Cho A nằm ngoài đường tròn (O). Từ A kẻ hai tiếp tuyến AB, AC và cát tuyến ADE
tới đường tròn đó (B, C là hai tiếp điểm; D nằm giữa A và E). Gọi H là giao điểm của AO và BC.
a) Chứng minh rằng ABOC là tứ giác nội tiếp.
b) Chứng minh rằng: AH. AO = AD. AE
c) Tiếp tuyến tại D của đường tròn (O) cắt AB, AC theo thứ tự tại I và K. Qua điểm O kẻ đường
thẳng vuông góc với OA cắt AB tại P và cắt AC tại Q. Chứng minh rằng: IP + KQ ≥ PQ
9. Tuyển sinh lớp 10 tỉnh Hà Tĩnh năm học 2011-2012
Câu 1
a)Tìm m để đường thẳng y = (2m – 1)x + 3 song song với đường thẳng y = 5x – 1.
b)Giải hệ phương trình:
2 5
3 2 4
x y
x y
+ =


− =

Câu 2 Cho biểu thức:
1 1 1
1
1 1
P
a a a
  
= − +
  
− +
  
với a 0 và 1
a ≠
a)Rút gọn biểu thức P. b)Với những giá trị nào của a thì P 
1
2
.
Câu 3
a)Tìm tọa độ giao điểm của đồ thị các hàm số: y = x2
và y = - x + 2.
b)Xác định các giá trị của m để phương trình x2
– x + 1 – m = 0 có 2 nghiệm x1, x2 thỏa mãn đẳng
thức: 1 2
1 2
1 1
5 4 0
x x
x x
 
+ − + =
 
 
.
Câu 4 Trên nửa đường tròn đường kính AB, lấy hai điểm P, Q sao cho P thuộc cung AQ.
Gọi C là giao điểm của tia AP và tia BQ; H là giao điểm của hai dây cung AQ và BP.
a)Chứng minh tứ giác CPHQ nội tiếp đường tròn.
b)Chứng minh CBP
∆ HAP
∆ .
c)Biết AB = 2R, tính theo R giá trị của biểu thức: S = AP.AC + BQ.BC.
Câu 5 Cho các số a, b, c đều lớn hơn
25
4
. Tìm giá trị nhỏ nhất của biểu thức:
2 5 2 5 2 5
a b c
Q
b c a
= + +
− − −
.
10. Tuyển sinh lớp 10 tỉnh Bình Định năm học 2011-2012
Bài 1: (2,0 điểm)
a)Giải hệ phương trình
 −


3x y = 7
2x + y = 8
.
b) Cho hàm số y = ax + b . Tìm a và b biết rằng đồ thị của hàm số đã cho song song với đường thẳng
2 3
y x
= − + và đi qua điểm M(2; 5).
Bài 2: (2,0 điểm) Cho phương trình ( )
2
2 1 4 0
x m x m
+ + + − = , với m là tham số.
a) Giải phương trình đã cho khi .
b) Chứng tỏ phương trình đã cho luôn có hai nghiệm phân biệt với mọi giá trị của tham số m.
c) Tìm m để phương trình đã cho có nghiệm x1, x2 thõa mãn hệ thức 2 2
1 2 1 2
3 0
x x x x
+ + = .
Bài 3: (2,0 điểm) Một mảnh đất hình chữ nhật có chiều dài hơn chiều rộng 6m và bình phương của số
đo độ dài đường chéo gấp 5 lần số đo của chu vi. Tính diện tích của mảnh đất hình chữ nhật đã cho.
Bài 4: (3,0 điểm) Cho đường tròn tâm O và BC là dây cung không đi qua tâm. Trên tia đối của tia BC
lấy điểm M sao cho M không trùng với B. Đường thẳng đi qua M cắt đường tròn (O) đã cho tại N và
P (N nằm giữa M và P) sao cho O nằm bên trong . Gọi A là điểm chính giữa của cung nhỏ NP.
Các dây AB và AC lần lượt cắt NP tại D và E .
a) Chứng minh tứ giác BDEC nội tiếp.
b) Chứng tỏ MB.MC = MN.MP .
c) OA cắt NP tại K. Chứng minh MK2
 MB.MC .
Bài 5: (1,0 điểm) Tìm giá trị nhỏ nhất của biểu thức
2
2
2 2011
x x
A
x
− +
= (với )
11. Tuyển sinh lớp 10 tỉnh Quảng Nam năm học 2011-2012
Bài 1 (2,0 điểm) Rút gọn các biểu thức sau:
a)A 2 5 3 45 500
= + − b)
1 15 12
B
5 2
3 2
−
= −
−
+
Bài 2 (2,5 điểm)
1) Giải hệ phương trình:
3x y 1
3x 8y 19



− =
+ =
2) Cho phương trình bậc hai: 2
x mx +m 1= 0 (1)
− −
a) Giải phương trình (1) khi m = 4.
b) Tìm các giá trị của m để phương trình (1) có hai nghiệm 1 2
x ;x thỏa 1 2
1 2
x x
1 1
x x 2011
+
+ = .
m 5
= −

PMC
x 0
≠
43
Bài 3 (1,5 điểm) Cho hàm số y = 2
1
x
4
.
1) Vẽ đồ thị (P) của hàm số đó.
2) Xác định a, b để đường thẳng (d): y = ax + b cắt trục tung tại điểm có tung độ bằng –2 và cắt đồ
thị (P) nói trên tại điểm có hoành độ bằng 2.
Bài 4 (4,0 điểm) Cho nửa đường tròn (O; R) đường kính AB. Gọi C là điểm chính giữa của cung AB.
Trên tia đối của tia CB lấy điểm D sao cho CD = CB. OD cắt AC tại M. Từ A, kẻ AH vuông góc với
OD (H thuộc OD). AH cắt DB tại N và cắt nửa đường tròn (O; R) tại E.
1) Chứng minh MCNH là tứ giác nội tiếp và OD song song với EB.
2) Gọi K là giao điểm của EC và OD. Chứng minh rằng ∆CKD = ∆CEB. Suy ra C là trung điểm
của KE.
3) Chứng minh tam giác EHK vuông cân và MN song song với AB.
4) Tính theo R diện tích hình tròn ngoại tiếp tứ giác MCNH.
12. Tuyển sinh lớp 10 tỉnh Quảng Ngãi năm học 2011-2012
Bài 1: (1.5 điểm)
1) Thực hiện phép tính: 2 9 3 16
+
2) Giải phương trình và hệ phương trình sau:
a) x2
– 20x + 96 = 0 b)
4023
1
x y
x y
+ =


− =

Bài 2: (2.5điểm)
1) Cho hàm số y = x2
có đồ thị là (P) và đường thẳng (d): y = x + 2
a) Vẽ ( P ) và ( d ) trên cùng một hệ toạ độ Oxy
b) Bằng phép tính hãy tìm toạ độ giao điểm của ( P ) và ( d )
2) Trong cùng một hệ toạ độ Oxy cho 3 điểm: A(2;4); B(-3;-1) và C(-2;1).
Chứng minh 3 điểm A, B, C không thẳng hàng.
3) Rút gọn biểu thức:
2
1
x x x
M
x x x
−
= +
− −
với 0; 1
x x
 ≠
Bài 3: (1.5điểm) Hai bến sông cách nhau 15 km. Thơì gian một ca nô xuôi dòng từ bến A đến bến B,
tại bến B nghỉ 20 phút rồi ngược dòng từ bến B trở về bến A tổng cộng là 3 giờ. Tính vận tốc của
ca nô khi nước yên lặng, biết vận tốc của dòng nước là 3 km/h.
Bài 4: (3.5 điểm) Cho nửa đường tròn tâm O đường kính AB. Một điểm C cố định thuộc đoạn thẳng
AO ( C khác A và C khác O ). Đường thẳng đi qua điểm C và vuông góc với AO cắt nửa đường
tròn đã cho tại D. Trên cung BD lấy điểm M ( với M khác B và M khác D). Tiếp tuyến của nửa
đường tròn đã cho tại M cắt đường thẳng CD tại E. Gọi F là giao điểm của AM và CD.
1)Chứng minh : BCFM là tứ giác nội tiếp đường tròn.
2)Chứng minh EM = EF
3)Gọi I là tâm đường tròn ngoại tiếp tam giác FDM. Chứng minh D, I, B thẳng hàng; từ đó suy ra
góc ABI có số đo không đổi khi M thay đổi trên cung BD.
Bài 5:(1.0 điểm) Cho phương trình ( ẩn x ): ( )
2
2 3 0
x m x m
− + + = . Gọi x1 và x2 là hai nghiệm của
phương trình đã cho. Tìm giá trị của m để biểu thức 2 2
1 2
x x
+ có giá trị nhỏ nhất.
13. Tuyển sinh lớp 10 tỉnh Thanh Hoá năm học 2011-2012
Bài 1: ( 1,5 điểm )
1. Cho hai số : b1 = 1 + ; b2 = 1 - . Tính b1 + b2
2. Giải hệ phương trình
Bài 2: ( 1,5 điểm ) Cho biểu thức B = với b và b 4
1. Rút gọn biểu thức B
2. Tính giá trị của B tại b = 6 + 4
Bài 3: ( 2,5 điểm ) Cho phương trình : x2
- ( 2n -1 )x + n (n - 1) = 0 ( 1 ) với n là tham số
1. Giải phương trình (1) với n = 2
2. Chứng minh rằng phương trình (1) luôn có hai nghiệm phân biệt với mọi n
3. Gọi x1, x2 là hai nghiệm của phương trình (1) (vơí x1x2). Chứng minh x1
2
- 2x2 + 3 0.
Bài 4: ( 3 điểm ) Cho tam giác BCD có 3 góc nhọn. Các đường cao CE và DF cắt nhau tại H .
1. CM: Tứ giác BFHE nội tiếp được trong một đường tròn
2. Chứng minh BFE và BDC đồng dạng
3. Kẻ tiếp tuyến Ey của đường tròn tâm O đường kính CD cắt BH tại N.
CMR: N là trung điểm của BH .
Bài 5: ( 1 điểm ) Cho các số dương x, y, z . Chứng minh rằng
14. Tuyển sinh lớp 10 tỉnh Khánh Hoà năm học 2011-2012
Bài 1( 2 điểm)
1)Đơn giản biểu thức: A
2 3 6 8 4
2 3 4
+ + + +
=
+ +
2) Cho biểu thức:
1 1
;( 1)
1 1
P a a
a a a a
 
= − − ≥
 
− − + −
 
. Rút gọn P và chứng tỏ P ≥ 0.
Bài 2( 2 điểm)
1)Cho phương trình bậc hai x2
+ 5x + 3 = 0 có hai nghiệm x1; x2. Hãy lập một phương trình bậc hai
có hai nghiệm (x1
2
+ 1 ) và ( x2
2
+ 1).
2)Giải hệ phương trình
2 3
4
2
4 1
1
2
x y
x y

+ =
 −


 − =
 −

2 2



−
=
−
=
+
3
2
1
2
n
m
n
m
2
1
:
)
4
1
4
2
2
(
+
−
−
+
−
−
+ b
b
b
b
b
b
b
0
≥ ≠
2
≥
∆
∆ ∆
2

+
+
+
+
+ y
x
z
z
x
y
z
y
x
45
Bài 3( 2 điểm) Quãng đường từ A đến B dài 50km. Một người dự định đi xe đạp từ A đến B với vận
tốc không đổi. Khi đi được 2 giờ, người ấy dừng lại 30 phút để nghỉ. Muốn đến B đúng thời gian đã
định, người đó phải tăng vận tốc thêm 2 km/h trên quãng đường còn lại. Tính vận tốc ban đầu của
người đi xe đạp.
Bài 4( 4 điểm) Cho tam giác ABC có ba góc nhọn và H là trực tâm. Vẽ hình bình hành BHCD. Đường
thẳng đi qua D và song song BC cắt đường thẳng AH tại E.
1) Chứng minh A,B,C,D,E cùng thuộc một đường tròn
2) Chứng minh BAE DAC
∠ = ∠
3) Gọi O là tâm đường tròn ngoại tiếp tam giác ABC và M là trung điểm của BC,đường thẳng AM
cắt OH tại G.Chứng minh G là trọng tâm của tam giácABC.
4) Giả sử OD = a.Hãy tính độ dài đường tròn ngoại tiếp tam giác BHC theo a
15. Tuyển sinh lớp 10 tỉnh Bình Dương năm học 2010-2011
Bài 1 (1đ) Rút gọn 2
16 8 1
M x x
= + + . Tính giá trị của M tại x = 2.
Bài 2 (1đ5)
1) Vẽ đồ thị của các hàm số sau trên cùng một mặt phẳng tọa độ 2
( ) :
P y x
= ;
( ) : 2 3
d y x
= +
2) Tìm tọa độ giao điểm (nếu có) của (d) và (P).
Bài 3(2đ)
1) Giải phương trình 2
5 6 0
x x
+ + =
2) Giải hệ phương trình
3 4
2 5 7
x y
x y
+ =


+ =

Bài 4 (2đ)
1) Một người dự định đi xe gắn máy từ địa điểm A đến địa điểm B cách nhau 90km. Vì
có việc gấp phải đến B trước giờ dự định là 45 phút nên người ấy phải tăng vận tốc
lên mỗi giờ 10 km . Hãy tính vận tốc mà người đó dự định đi .
2) Chứng minh rằng phương trình ( )
2
2 2 1 4 8 0
x m x m
− − + − = (m là tham số) luôn có 2
nghiệm phân biệt và khác 1 với mọi m ∈ R .
Bài 5 (3đ5) Một hình vuông ABCD nội tiếp trong đường tròn Tâm O bán kính R . Một điểm
M di động trên cung ABC , M không trùng với A,B và C, MD cắt AC tại H.
1) Chứng minh tứ giác MBOH nội tiếp được trong đường tròn và DH.DM = 2R2
.
2) Chứng minh tam giác MDC đồng dạng với tam giác MAH .
3) Hai tam giác MDC và MAH bằng nhau khi M ở một vị trí đặc biệt M’. Xác định điểm
M’. Khi đó M’D cắt AC tại H’. Đường thẳng qua M’ và vuông góc với AC cắt AC tại
I. Chứng minh rằng I là trung điểm của H’C .
46
16. Tuyển sinh lớp 10 tỉnh Bến Tre năm học 2011-2012
I. PHẦN TRẮC NGHIỆM: Thời gian làm bài 20 phút / 3,0 điểm
(Chọn phương án đúng cho mỗi câu và ghi vào giấy làm bài . Ví dụ: câu 1 chọn A thì ghi 1.A)
Câu 1. Biểu thức M = 4 2 3 3
+ − có giá trị bằng:
A. 2 3 1
− B. 1 2 3
− C. 1 D. -1
Câu 2. Với giá trị nào của m thì đường thẳng (d1): mx – 2y = 2 cắt đường thẳng (d2): x + y = 3?
A. 2
m ≠ − B. 2
m ≠ C. 2
m = − D. 2
m =
Câu 3. Hệ phương trình
2 4
2
x y
x y



+ =
− =
có nghiệm (x;y). Tổng x + y bằng:
A.0 B. 2 C. 4 D. 6
Câu 4. Đồ thị hàm số y = f(x) = ax2
đi qua điểm A(-2; 4) có hệ số a bằng:
A. -1 B. 1 C.
1
8
D.
1
8
−
Câu 5. Cho hàm số y = f(x) = ax2
. Nếu f(2) = 1 th ì f(-2) + 2 bằng:
A. 1 B. 2 C. 3 D. 4
Câu 6. Nếu 0 1 3
x = − là nghiệm của phương trình 2
1
x x m
− + = thì m bằng:
A. 4 3
− B. 4 3
+ C.
4 3
12
−
D.
4 3
2
+
Câu 7. Với giá trị nào của m thì phương trình ( )
2
2 1 2 0
mx m x m
+ − + + = có nghiệm?
A.
1
12
m ≥ B.
1
12
m ≤ C.
1
12
m ≥ và 0
m ≠ D.
1
12
m  và 0
m ≠
Câu 8. Phương trình nào sau đây nhận 1 2
2 3; 2 3
x x
= − = + là nghiệm?
A. 2
4 0
x x
+ + = B. 2
4 0
x x
− − = C. 2
4 1 0
x x
+ + = D. 2
4 1 0
x x
− + =
Câu 9. Tam giác ABC cân tại A nội tiếp đường tròn (O) có  0
A 60
= , số đo của 
AOB bằng:
A.
0
65 B.
0
120 C.
0
130 D.
0
135
Câu 10. Cho tam giác ABC cân tại B có 6
AC cm
= ,  0
120
B = .
Độ dài đường tròn ngoại tiếp tam giác ABC tính bằng cm là:
47
A. 3
π B. 2 3
π C.4 3
π D. 5 3
π
Câu 11. Một ngọn tháp cao 50, có bóng trên mặt đất dài 15m.
Góc mà tia sáng mặt trời tạo với mặt đất (làm tròn đến độ) là:
A.
0
71 B.
0
73 C.
0
75 D.
0
80
Câu 12. Cho tam giác ABC vuông tại A. Biết rằng
5
6
AB
AC
= , đường cao 30 .
AH cm
=
Độ dài BH tính bằng cm là:
A.18 B.20 C.25 D.36
II. PHẦN TỰ LUẬN: Thời gian làm bài 100 phút/7 điểm.
Bài 1. (1,0 điểm)
Cho biểu thức
1 1 1 2
:
1 2 1
x x
A
x x x x
+ +
   
= − −
   
− − −
   
.
1. Rút gọn A khi 0; 1; 2
x x x
≠ ≠ ≠
2. Tìm x để giá trị của
3
3
A = − .
Bài 2. (2,0 điểm)
Cho hệ phương trình
2
3 5 2
x y m
x y m
+ = +


+ =

với m là tham số.
1. Giải hệ phương trình khi 1
m = − .
2. Xác định giá trị của m để hệ phương trình có nghiệm ( )
;
x y thoả mãn điều kiện: 1
x y
+ =
Bài 3. (1,5 điểm)
Cho phương trình ( )
2
2 1 3 0
x m x m
− + − − = với m là tham số.
1. Chứng minh rằng phương trình luôn có hai nghiệm phân biệt.
2. Gọi 1 2
,
x x là hai nghiệm của phương trình. Tìm m để ( )
2
1 2
x x
− đạt giá trị nhỏ nhất.
Bài 4. (2,5 điểm)
Cho góc xOy và điểm P nằm trong góc đó. Gọi H và K lần lượt là hình chiếu của P lên Ox và Oy.
Đường thẳng PK cắt Ox tại A, đường thẳng PH cắt Oy tại B.
1. Chứng minh tứ giác OKPH và tứ giác KHAB nội tiếp đường tròn.
2. Cho  0
60
xOy = và OP a
= . Tính độ dài HK và AB theo a.
3. Gọi M và N lần lượt là trung điểm OP và AB. Chứng minh tứ giác MKNH nội tiếp đường tròn.
48
V. Đề tham khảo ôn tập tuyển sinh
1. ĐỀ SỐ 01
Bài 1. Giải phương trình và hệ phương trình:
a) 4 2
4 2 1 0
x x
− − = b)
3 2 0
2 5 5
x y
x y
− =


+ = −

Bài 2. Rút gọn biểu thức:
a)
2 2
3 5 3 5
+
+ −
b)
17 12 2 2 2 2 1
3 2 2 2 2 3 2
− +
− +
− + −
Bài 3. Cho hàm số ( )
2
2
x
y P
= − và ( )
2
y x d
= .
a)Vẽ (P) và (d) trên cùng một hệ trục toạ độ.
b)Tìm toạ độ giao điểm của (P) và (d) bằng phép toán.
c)Viết phương trình đường thẳng ( )
1
d // ( )
d và tiếp xúc với ( )
P .
Bài 4. Cho phương trình 2
2 2 1 0
x mx m
− + − = .
a)Tìm m để phương trình có 2 nghiệm cùng dương.
b)Với m tìm được ở câu a hãy tính 1 2
x x
+ theo m.
c)Với giá trị nào của m thì phương trình có 2 nghiệm đối nhau.
Bài 5. Cho nửa đường tròn (O), đường kính AD. Trên nửa đường tròn lấy 2 điểm B và C sao cho
cung AB nhỏ hơn cung AC, AC cắt BD tại E. Kẻ EF vuông góc với AD tại F.
a)Chứng minh rằng ADEF nội tiếp.
b)Chứng minh rằng BD là phân giác của 
CEF .
c)Gọi M là trung điểm của ED. Chứng minh rằng BCMF nội tiếp.
d)CF cắt BD tại I. Chứng minh rằng 2
. .
BI BF BC IF IC
= − .
2. ĐỀ SỐ 02
Bài 1. Rút gọn biểu thức
49
a) ( )( )
4 7 2 14 4 7
A = − + + b)
2 2
2 2 2 2 2 2
B = +
+ + − +
Bài 2. Giải các phương trình và hệ phương trình:
a) ( )
4 2
2 5 2 5 0
x x
− − − = b)
3 4 2 3
2 3 3
x y
x y
 − =


+ = −


Bài 3. Cho hàm số ( ) 2
1
2
P y x
= − và ( ) 2
D y x
= .
a)Vẽ (P) và (D) trên cùng một hệ trục toạ độ Oxy.
b)Bằng phép toán, tìm toạ độ giao điểm (P) và (D).
c)Viết phương trình đường thẳng (d) đi qua A(-3; -4) và tiếp xúc với (P).
Bài 4. Cho phương trình ( )
2
2 1 4 0
x m x m
− + + − = .
a)Chứng minh rằng phương trình có hai nghiệm phân biệt 1 2
,
x x với mọi m.
b)Tìm m để phương trình có hai nghiệm trái dấu.
c)Chứng minh rằng biểu thức ( ) ( )
1 2 2 1
1 1
M x x x x
= − + − không phụ thuộc vào m.
Bài 5. Cho tam giác ABC vuông tại A (AB  AC). D là điểm thuộc AC. Vẽ DE vuông góc BC tại E.
a)Chứng minh rằng ADEB nội tiếp. Xác định tâm O của đường tròn ngoại tiếp này.
b)Vẽ đường tròn tâm D bán kính DE cắt (O) tại F, BF cắt AD tại I, BD cắt AE tại K.
Chứng minh rằng ABKI nội tiếp.
c)Chứng minh rằng BI. BF = BK. BD
d)Trung tuyến AM của tam giác ABC cắt tại N. Chứng minh rằng NA = NF.
3. ĐỀ SỐ 03
Bài 1. Cho biểu thức
2 2 2
: ; 0, 1
1 1
2 1
x x x
A x x
x x
x x
 
− +
= −  ≠
 
 
− −
+ +
 
.
a)Rút gọn biểu thức A.
b)Tìm x để A có giá trị nguyên.
Bài 2. Giải phương trình và hệ phương trình
50
a) 4 2
25 144 0
x x
− + = b) ( )
2
2 2
6 0
x x
− − = c)
2 0
4 3 20
x y
x y
− =


− =

d) 2
2 3 2 0
x x
− − =
Bài 3.
a)Vẽ đồ thị hàm số ( )
2
y x P
= − và ( ) 6
D y x
= − trên cùng một hệ trục toạ độ.
b)Tìm toạ độ giao điểm của (P) và (D) bằng phép toán.
c)Viết phương trình đường thẳng (D’) vuông góc với (D) và tiếp xúc với (P).
Bài 4. Cho phương trình ( )
2 2
2 4 8 0
x m x m
− + + − = .
a)Định m để phương trình có nghiệm.
b)Gọi 1 2
,
x x là 2 nghiệm của. Lập hệ thức liên hệ giữa 1 2
,
x x không phụ thuộc vào m.
c)Tìm m để 2 2
1 2 1 2
A x x x x
= − − đạt giá trị nhỏ nhất. Tìm giá trị đó.
Bài 5. Từ đỉnh A của hình vuông ABCD, ta kẻ hai tia tạo với nhau một góc 45o
. Một tia cắt BC tại
E và cắt BD tại P, tia kia cắt CD tại F và cắt BD tại Q.
a)Chứng minh rằng APED nội tiếp.
b)Chứng minh rằng E, P, Q, F, C cùng thuộc một đường tròn.
c)Chứng minh rằng 2
AEF APQ
S S
= .
d)Kẻ đường trung trực của cạnh CD cắt AE tại M. Tính 
MAB khi tứ giác CPMD nội tiếp.
4. ĐỀ SỐ 04
Bài 1. Giải phương trình và hệ phương trình
a) 4 2
3 10 8 0
x x
+ + = b) ( )
2
2 3 2 3 0
x x
− − − = c)
3 2 0
2 3 5
x y
x y
− =


+ = −

Bài 2. Cho hai hàm số ( )
2
4
x
y P
= và ( )
1
y x d
= − − .
a)Vẽ (P) và (d) trên cùng một hệ trục toạ.
b)Tìm toạ độ giao điểm của (P) và (d) bằng phép toán.
c)Viết phương trình đường thẳng ( ) ( )
1 / /
d d và cắt (P) tại điểm có hoành độ bằng O.
Bài 3. Cho phương trình ( )
2
2 2 0
x m x m
+ − − = .
51
a)Chứng minh rằng phương trình có nghiệm với mọi m.
b)Tìm m để phương trình có nghiệm số kép và tính nghiệm đó.
c)Với giá trị nào của m thì phương trình có 2 nghiệm phân biệt là hai số nghịch đảo của nhau.
Bài 4. Rút gọn
a) 13 160 53 4 90
− − + b)
23 2
2 14 5 3
+ +
Bài 5. Cho tam giác ABC nhọn (AB  AC) nội tiếp trong (O; R) có BE và CF là các đường cao cắt
nhau tại H.
a)Chứng minh rằng BFEC nội tiếp, xác định tâm I.
b)Tia AH cắt BC tại D. Chứng minh rằng EB là phân giác 
DEF .
c)Chứng minh rằng OA vuông góc với EF.
d)Đường thẳng EF cắt (O) tại N và M (F nằm giữa N và E). Chứng minh rằng AN là tiếp
tuyến của đường tròn ngoại tiếp tam giác NHD.
e)Vẽ (H, HA) cắt AB và AC lần lượt tại P và Q. Chứng minh rằng đường thẳng kẻ từ A
vuông góc với PQ đi qua điểm P cố định.
5. ĐỀ SỐ 05
Bài 1. Tính
a) ( )
3 5 10 2
A = − + b)
3 8 15
1
2 30 2
B
−
= − +
−
Bài 2. Giải phương trình và hệ phương trình
a) 4 2
4 5 9 0
x x
− − = b) ( )
2
1 3 3 0
x x
− + + = c)
2 2 3 5
9
3 2 3
2
x y
x y
 + =


− =


d)
4
4 20 3 5 9 45 6
3
x x x
+ − + + + =
Bài 3. Cho hàm số ( )
2
2
x
y P
= và ( )
2
2
x
y D
= + .
a)Vẽ (P) và (D) trên cùng một hệ trục toạ độ.
52
b)Tìm toạ độ giao điểm của (P) và (D).
c)Viết phương trình đường thẳng (D’), biết (D’) // (D) và đi qua B(2; -3).
Bài 4. Cho phương trình ( )
2
2 1 2 1 0
x m x m
− + + + = .
a)Tìm điều kiện m để phương trình có nghiệm.
b)Gọi 1 2
,
x x là nghiệm của phương trình. Tìm giá trị nhỏ nhất của ( )
2
1 2 1 2
8
M x x x x
= + + .
c)Tìm m để phương trình có hai nghiệm thoả 1 2
2 2
x x
+ = .
Bài 5. Cho (O; R) lấy P ở ngoài đường tròn sao cho OP = 2R. Vẽ cát tuyến PAB, từ A và B vẽ 2 tiếp
tuyến của (O) cắt nhau tại M, hạ MH vuông góc với OP tại H.
Chứng minh rằng MO vuông góc với AB tại I và tứ giác MIHP nội tiếp.
Chứng minh rằng OH luôn không đổi khi cát tuyến PAB quay quanh P.
Cho
3
R
OI = . Tính PHA
S theo R.
6. ĐỀ SỐ 06
Bài 1. Rút gọn biểu thức
a)
5 2 5 3 3
5 3
5 3
+ +
+ − − b) ( )
9 9 6
6 0; 9
3 3
x x x
x x
x x
− − +
− − ≥ ≠
+ −
Bài 2. Giải các phương trình và hệ phương trình sau:
a) 2
2 7 3 0
x x
− + = b) 4 2
5 4 0
x x
− + = c)
2 5
3 2 8
x y
x y
− =


− =

Bài 3. Cho (P)
2
2
x
y = và (D) 1
2
x
y = + .
Vẽ (P) và (D) trên cùng một hệ trục toạ độ.
Xác định toạ độ giao điểm của (P) và (D) bằng đồ thị và kiểm tra lại bằng phép toán.
Bài 4. Cho phương trình bậc hai 2
3 2 0
x x m
− + − = , trong đó m là tham số, x là ẩn số.
a)Tìm điều kiện của m để phương trình có nghiệm.
b)Gọi 1 2
,
x x là nghiệm của phương trình. Tính 2 3
1 2 1 2
3 3 2
A x x x x
= + − theo m.
53
Bài 5. Cho tam giác nhọn ABC (AB  AC) nội tiếp trong (O; R) có BE và CF là các đường cao cắt
nhau tại H (E thuộc AC, F thuộc AB).
a)Chứng minh rằng BFEC nội tiếp trong đường tròn có tâm là M, xác định vị trí điểm M.
b)Tia AH cắt BC tại D. Chứng minh rằng EB là tia phân giác góc 
DEF .
c)Vẽ tiếp tuyến xAy của đường tròn (O). Chứng minh rằng OA vuông góc với OF.
d)Đường thẳng EF cắt đường tròn tại M, N (F nằm giữa N và E).
Chứng minh rằng AN là tiếp tuyến của đường tròn ngoại tiếp tam giác NHD.
7. ĐỀ SỐ 07
Bài 1. Giải phương trình và hệ phương trình
a) ( )
4 2
2 2 2 2 0
x x
− − − = b)
2 3
1
3 2
1 1 4
2 3 3
x y
x y

− =



 + =


Bài 2. Cho (P)
2
2
x
y = − và (D) 2
y x
= .
a)Vẽ (P) và (D) trên cùng một hệ trục toạ độ.
b)Tìm toạ độ giao điểm của (P) và (D) bằng phép toán.
c)Viết phương trình đường thẳng (D’) // (D) và (D’) cắt (P) tại điểm A có hoành độ bằng -2.
Bài 3. Cho phương trình 2 2
3 1 0
x x m
− − − = .
a)Chứng minh rằng phương trình luôn có nghiệm với mọi x.
b)Tìm m để phương trình có hai nghiệm 1 2
,
x x thoả 2 2
1 2 29
x x
+ =
c)Tìm m để phương trình có hai nghiệm 1 2
,
x x thoả 1 2 5
x x
− =
Bài 4. Cho
2 2 1
:
1
2 1
x x x
Q
x
x x x
 
+ − +
= −
 
 
−
+ +
 
a)Tìm điều kiện để Q có nghĩa và rút gọn Q.
b)Tìm các giá trị nguyên của x để Q có giá trị nguyên.
Bài 5. Cho (O; R) đường kinh AB. Tiếp tuyến tại điểm M bất kỳ thuộc (O) (M khác A và B) cắt tiếp
tuyến tại A và B của đường tròn này lần lượt tại C và D.
54
a)Chứng minh rằng ACMO và DBOM là các tứ giác nội tiếp.
b)Chứng minh rằng AC + BD = CD và tam giác COD vuông.
c)Gọi I và K lần lượt là tâm đường tròn nội tiếp của tam giác DBM và tam giác CAM.
Chứng minh rằng các điểm I, K thuộc một đường tròn cố định và độ dài IK không đổi khi M
di động trên (O).
d)Tìm vị trí của điểm M trên (O) để tứ giác CKID có diện tích nhỏ nhất.
8. ĐỀ SỐ 08
Bài 1. Tính và rút gọn các biểu thức sau:
a) ( )
2 4 6 2 5 10 2
A
 
= + + −
 
 
b) ( )
2
1 1 2
1 0; 1
1
1 1
a a
B a a
a
a a
 
− +  
= + −  ≠
  
  +
+ −  
 
Bài 2. Giải phương trình và hệ phương trình sau:
a)
4
2 2
2 3
2
4 2 1
2 3
x y
x y
x y
x y

+ − =
 + +


 − + =
 + +

b) 2 2
4 2 2 8 5 2 3
x x x x
− + − + − + − = +
Bài 3. Một khu vườn hình chữ nhật, có chiều dài hơn chiều rộng 5m và diện tích bằng 300 m2. Tính
kích thước và chu vi khu vườn.
Bài 4. Cho phương trình 2 2
4 3 0
x x m
− − = (m là tham số khác 0)
Chứng minh rằng phương trình có 2 nghiệm trái dấu với mọi giá trị m.
Định m để phương trình có hai nghiệm 1 2
,
x x thoả 1 2
3 4 26
x x
− = .
Bài 5. Cho tam giác ABC vuông tại A nội tiếp trong (O; R) có đường cao AH. Gọi M, N, P lần lượt
là tâm đường tròn nội tiếp các tam giác AHB, AHC, ABC và I là điểm chính giữa của cung BC
không chứa A.
a)HM cắt AB tại E, HN cắt AC tại F. Chứng minh rằng tam giác AEF vuông cân.
b)Chứng minh rằng MN // EF.
c)Chứng minh rằng BM vuông góc với AN và AI vuông góc với MN.
d)Khi A di động trên nửa (O) thì P đi động trên đường cố định nào.
BỘ ĐỀ TUYỂN SINH VÀO 10 THÀNH PHỐ HỒ CHÍ MINH
BỘ ĐỀ TUYỂN SINH VÀO 10 THÀNH PHỐ HỒ CHÍ MINH
BỘ ĐỀ TUYỂN SINH VÀO 10 THÀNH PHỐ HỒ CHÍ MINH
BỘ ĐỀ TUYỂN SINH VÀO 10 THÀNH PHỐ HỒ CHÍ MINH
BỘ ĐỀ TUYỂN SINH VÀO 10 THÀNH PHỐ HỒ CHÍ MINH
BỘ ĐỀ TUYỂN SINH VÀO 10 THÀNH PHỐ HỒ CHÍ MINH
BỘ ĐỀ TUYỂN SINH VÀO 10 THÀNH PHỐ HỒ CHÍ MINH
BỘ ĐỀ TUYỂN SINH VÀO 10 THÀNH PHỐ HỒ CHÍ MINH
BỘ ĐỀ TUYỂN SINH VÀO 10 THÀNH PHỐ HỒ CHÍ MINH
BỘ ĐỀ TUYỂN SINH VÀO 10 THÀNH PHỐ HỒ CHÍ MINH
BỘ ĐỀ TUYỂN SINH VÀO 10 THÀNH PHỐ HỒ CHÍ MINH
BỘ ĐỀ TUYỂN SINH VÀO 10 THÀNH PHỐ HỒ CHÍ MINH

More Related Content

Similar to BỘ ĐỀ TUYỂN SINH VÀO 10 THÀNH PHỐ HỒ CHÍ MINH

Giải pháp hoàn thiện quản trị nguồn nhân lực tại Công ty Dịch vụ Bảo Vệ An Ninh
Giải pháp hoàn thiện quản trị nguồn nhân lực tại Công ty Dịch vụ Bảo Vệ An NinhGiải pháp hoàn thiện quản trị nguồn nhân lực tại Công ty Dịch vụ Bảo Vệ An Ninh
Giải pháp hoàn thiện quản trị nguồn nhân lực tại Công ty Dịch vụ Bảo Vệ An NinhDịch vụ viết thuê Luận Văn - ZALO 0932091562
 
Luận Văn Tác Động Của Yếu Tố Đạo Đức Đến Ý Định Vi Phạm Bản Quyền Của Người ...
Luận Văn  Tác Động Của Yếu Tố Đạo Đức Đến Ý Định Vi Phạm Bản Quyền Của Người ...Luận Văn  Tác Động Của Yếu Tố Đạo Đức Đến Ý Định Vi Phạm Bản Quyền Của Người ...
Luận Văn Tác Động Của Yếu Tố Đạo Đức Đến Ý Định Vi Phạm Bản Quyền Của Người ...Hỗ Trợ Viết Đề Tài luanvanpanda.com
 
Nghiên cứu tổng hợp vật liệu mof 199 và khảo sát hoạt tính xúc tác trên phản ...
Nghiên cứu tổng hợp vật liệu mof 199 và khảo sát hoạt tính xúc tác trên phản ...Nghiên cứu tổng hợp vật liệu mof 199 và khảo sát hoạt tính xúc tác trên phản ...
Nghiên cứu tổng hợp vật liệu mof 199 và khảo sát hoạt tính xúc tác trên phản ...https://www.facebook.com/garmentspace
 
Nghiên cứu tổng hợp vật liệu mof 199 và khảo sát hoạt tính xúc tác trên phản ...
Nghiên cứu tổng hợp vật liệu mof 199 và khảo sát hoạt tính xúc tác trên phản ...Nghiên cứu tổng hợp vật liệu mof 199 và khảo sát hoạt tính xúc tác trên phản ...
Nghiên cứu tổng hợp vật liệu mof 199 và khảo sát hoạt tính xúc tác trên phản ...https://www.facebook.com/garmentspace
 
Nghiên cứu tổng hợp vật liệu mof 199 và khảo sát hoạt tính xúc tác trên phản ...
Nghiên cứu tổng hợp vật liệu mof 199 và khảo sát hoạt tính xúc tác trên phản ...Nghiên cứu tổng hợp vật liệu mof 199 và khảo sát hoạt tính xúc tác trên phản ...
Nghiên cứu tổng hợp vật liệu mof 199 và khảo sát hoạt tính xúc tác trên phản ...TÀI LIỆU NGÀNH MAY
 
Thiet ke he_thong_tinh_huong_trong_day_hoc_hoa_hoc_lop_10_trung_hoc_pho_thong...
Thiet ke he_thong_tinh_huong_trong_day_hoc_hoa_hoc_lop_10_trung_hoc_pho_thong...Thiet ke he_thong_tinh_huong_trong_day_hoc_hoa_hoc_lop_10_trung_hoc_pho_thong...
Thiet ke he_thong_tinh_huong_trong_day_hoc_hoa_hoc_lop_10_trung_hoc_pho_thong...Garment Space Blog0
 
Luận văn Thạc sĩ Nghiên cứu ảnh hưởng của một số yếu tố đến quá trình xử lý ...
 Luận văn Thạc sĩ Nghiên cứu ảnh hưởng của một số yếu tố đến quá trình xử lý ... Luận văn Thạc sĩ Nghiên cứu ảnh hưởng của một số yếu tố đến quá trình xử lý ...
Luận văn Thạc sĩ Nghiên cứu ảnh hưởng của một số yếu tố đến quá trình xử lý ...Dịch vụ viết thuê Luận Văn - ZALO 0932091562
 
đáNh giá hiện trạng môi trường không khí và đề xuất biện pháp giảm thiểu ô nh...
đáNh giá hiện trạng môi trường không khí và đề xuất biện pháp giảm thiểu ô nh...đáNh giá hiện trạng môi trường không khí và đề xuất biện pháp giảm thiểu ô nh...
đáNh giá hiện trạng môi trường không khí và đề xuất biện pháp giảm thiểu ô nh...TÀI LIỆU NGÀNH MAY
 
THU HÚT VỐN ĐẦU TƯ TRỰC TIẾP NƯỚC NGOÀI VÀO TỈNH ĐỒNG NAI GIAI ĐOẠN 2021 - 2025
THU HÚT VỐN ĐẦU TƯ TRỰC TIẾP NƯỚC NGOÀI VÀO TỈNH ĐỒNG NAI GIAI ĐOẠN 2021 - 2025THU HÚT VỐN ĐẦU TƯ TRỰC TIẾP NƯỚC NGOÀI VÀO TỈNH ĐỒNG NAI GIAI ĐOẠN 2021 - 2025
THU HÚT VỐN ĐẦU TƯ TRỰC TIẾP NƯỚC NGOÀI VÀO TỈNH ĐỒNG NAI GIAI ĐOẠN 2021 - 2025lamluanvan.net Viết thuê luận văn
 
Luận Văn Động Lực Làm Việc Của Nhân Viên tạii Bảo Hiểm Xã Hội
Luận Văn Động Lực Làm Việc Của Nhân Viên tạii Bảo Hiểm Xã HộiLuận Văn Động Lực Làm Việc Của Nhân Viên tạii Bảo Hiểm Xã Hội
Luận Văn Động Lực Làm Việc Của Nhân Viên tạii Bảo Hiểm Xã HộiViết Thuê Luận Văn Luanvanpanda.com
 
Luận văn: Các nhân tố của chất lượng dịch vụ đào tạo ảnh hưởng đến sự hài lòn...
Luận văn: Các nhân tố của chất lượng dịch vụ đào tạo ảnh hưởng đến sự hài lòn...Luận văn: Các nhân tố của chất lượng dịch vụ đào tạo ảnh hưởng đến sự hài lòn...
Luận văn: Các nhân tố của chất lượng dịch vụ đào tạo ảnh hưởng đến sự hài lòn...Viết thuê trọn gói ZALO 0934573149
 
Nghiên cứu thực trạng sinh viên tốt nghiệp có việc làm
Nghiên cứu thực trạng sinh viên tốt nghiệp có việc làmNghiên cứu thực trạng sinh viên tốt nghiệp có việc làm
Nghiên cứu thực trạng sinh viên tốt nghiệp có việc làmTBiAnh7
 
ĐỀ TÀI : Nhân tố ảnh hưởng đến dự định khởi sự kinh doanh của sinh viên
ĐỀ TÀI : Nhân tố ảnh hưởng đến dự định khởi sự kinh doanh của sinh viênĐỀ TÀI : Nhân tố ảnh hưởng đến dự định khởi sự kinh doanh của sinh viên
ĐỀ TÀI : Nhân tố ảnh hưởng đến dự định khởi sự kinh doanh của sinh viênLuận Văn 1800
 
Nhân tố ảnh hưởng đến dự định khởi sự kinh doanh của sinh viên - Gửi miễn phí...
Nhân tố ảnh hưởng đến dự định khởi sự kinh doanh của sinh viên - Gửi miễn phí...Nhân tố ảnh hưởng đến dự định khởi sự kinh doanh của sinh viên - Gửi miễn phí...
Nhân tố ảnh hưởng đến dự định khởi sự kinh doanh của sinh viên - Gửi miễn phí...Dịch vụ viết bài trọn gói ZALO: 0909232620
 

Similar to BỘ ĐỀ TUYỂN SINH VÀO 10 THÀNH PHỐ HỒ CHÍ MINH (20)

Giải pháp hoàn thiện quản trị nguồn nhân lực tại Công ty Dịch vụ Bảo Vệ An Ninh
Giải pháp hoàn thiện quản trị nguồn nhân lực tại Công ty Dịch vụ Bảo Vệ An NinhGiải pháp hoàn thiện quản trị nguồn nhân lực tại Công ty Dịch vụ Bảo Vệ An Ninh
Giải pháp hoàn thiện quản trị nguồn nhân lực tại Công ty Dịch vụ Bảo Vệ An Ninh
 
Luận văn: Phát triển làng nghề truyền thống ở tỉnh Trà Vinh, HOT
Luận văn: Phát triển làng nghề truyền thống ở tỉnh Trà Vinh, HOTLuận văn: Phát triển làng nghề truyền thống ở tỉnh Trà Vinh, HOT
Luận văn: Phát triển làng nghề truyền thống ở tỉnh Trà Vinh, HOT
 
Luận Văn Tác Động Của Yếu Tố Đạo Đức Đến Ý Định Vi Phạm Bản Quyền Của Người ...
Luận Văn  Tác Động Của Yếu Tố Đạo Đức Đến Ý Định Vi Phạm Bản Quyền Của Người ...Luận Văn  Tác Động Của Yếu Tố Đạo Đức Đến Ý Định Vi Phạm Bản Quyền Của Người ...
Luận Văn Tác Động Của Yếu Tố Đạo Đức Đến Ý Định Vi Phạm Bản Quyền Của Người ...
 
Hoàn thiện công tác quản trị nguồn nhân lực tại Công ty Bảo Vệ An Ninh Toàn Việt
Hoàn thiện công tác quản trị nguồn nhân lực tại Công ty Bảo Vệ An Ninh Toàn ViệtHoàn thiện công tác quản trị nguồn nhân lực tại Công ty Bảo Vệ An Ninh Toàn Việt
Hoàn thiện công tác quản trị nguồn nhân lực tại Công ty Bảo Vệ An Ninh Toàn Việt
 
Nghiên cứu tổng hợp vật liệu mof 199 và khảo sát hoạt tính xúc tác trên phản ...
Nghiên cứu tổng hợp vật liệu mof 199 và khảo sát hoạt tính xúc tác trên phản ...Nghiên cứu tổng hợp vật liệu mof 199 và khảo sát hoạt tính xúc tác trên phản ...
Nghiên cứu tổng hợp vật liệu mof 199 và khảo sát hoạt tính xúc tác trên phản ...
 
Nghiên cứu tổng hợp vật liệu mof 199 và khảo sát hoạt tính xúc tác trên phản ...
Nghiên cứu tổng hợp vật liệu mof 199 và khảo sát hoạt tính xúc tác trên phản ...Nghiên cứu tổng hợp vật liệu mof 199 và khảo sát hoạt tính xúc tác trên phản ...
Nghiên cứu tổng hợp vật liệu mof 199 và khảo sát hoạt tính xúc tác trên phản ...
 
Nghiên cứu tổng hợp vật liệu mof 199 và khảo sát hoạt tính xúc tác trên phản ...
Nghiên cứu tổng hợp vật liệu mof 199 và khảo sát hoạt tính xúc tác trên phản ...Nghiên cứu tổng hợp vật liệu mof 199 và khảo sát hoạt tính xúc tác trên phản ...
Nghiên cứu tổng hợp vật liệu mof 199 và khảo sát hoạt tính xúc tác trên phản ...
 
Thiet ke he_thong_tinh_huong_trong_day_hoc_hoa_hoc_lop_10_trung_hoc_pho_thong...
Thiet ke he_thong_tinh_huong_trong_day_hoc_hoa_hoc_lop_10_trung_hoc_pho_thong...Thiet ke he_thong_tinh_huong_trong_day_hoc_hoa_hoc_lop_10_trung_hoc_pho_thong...
Thiet ke he_thong_tinh_huong_trong_day_hoc_hoa_hoc_lop_10_trung_hoc_pho_thong...
 
Luận văn Thạc sĩ Nghiên cứu ảnh hưởng của một số yếu tố đến quá trình xử lý ...
 Luận văn Thạc sĩ Nghiên cứu ảnh hưởng của một số yếu tố đến quá trình xử lý ... Luận văn Thạc sĩ Nghiên cứu ảnh hưởng của một số yếu tố đến quá trình xử lý ...
Luận văn Thạc sĩ Nghiên cứu ảnh hưởng của một số yếu tố đến quá trình xử lý ...
 
Luận Văn Các Yếu Tố Tác Động Đến Thu Hút Đầu Tư Trực Tiếp Nước Ngoài.doc
Luận Văn Các Yếu Tố Tác Động Đến Thu Hút Đầu Tư Trực Tiếp Nước Ngoài.docLuận Văn Các Yếu Tố Tác Động Đến Thu Hút Đầu Tư Trực Tiếp Nước Ngoài.doc
Luận Văn Các Yếu Tố Tác Động Đến Thu Hút Đầu Tư Trực Tiếp Nước Ngoài.doc
 
đáNh giá hiện trạng môi trường không khí và đề xuất biện pháp giảm thiểu ô nh...
đáNh giá hiện trạng môi trường không khí và đề xuất biện pháp giảm thiểu ô nh...đáNh giá hiện trạng môi trường không khí và đề xuất biện pháp giảm thiểu ô nh...
đáNh giá hiện trạng môi trường không khí và đề xuất biện pháp giảm thiểu ô nh...
 
Luận văn: Quản lý hoạt động học tập của học sinh tỉnh Đồng Nai
Luận văn: Quản lý hoạt động học tập của học sinh tỉnh Đồng NaiLuận văn: Quản lý hoạt động học tập của học sinh tỉnh Đồng Nai
Luận văn: Quản lý hoạt động học tập của học sinh tỉnh Đồng Nai
 
Luận văn: Xử lý nước thải sinh hoạt bằng cây rau ngổ dại, HAY
Luận văn: Xử lý nước thải sinh hoạt bằng cây rau ngổ dại, HAYLuận văn: Xử lý nước thải sinh hoạt bằng cây rau ngổ dại, HAY
Luận văn: Xử lý nước thải sinh hoạt bằng cây rau ngổ dại, HAY
 
THU HÚT VỐN ĐẦU TƯ TRỰC TIẾP NƯỚC NGOÀI VÀO TỈNH ĐỒNG NAI GIAI ĐOẠN 2021 - 2025
THU HÚT VỐN ĐẦU TƯ TRỰC TIẾP NƯỚC NGOÀI VÀO TỈNH ĐỒNG NAI GIAI ĐOẠN 2021 - 2025THU HÚT VỐN ĐẦU TƯ TRỰC TIẾP NƯỚC NGOÀI VÀO TỈNH ĐỒNG NAI GIAI ĐOẠN 2021 - 2025
THU HÚT VỐN ĐẦU TƯ TRỰC TIẾP NƯỚC NGOÀI VÀO TỈNH ĐỒNG NAI GIAI ĐOẠN 2021 - 2025
 
Luận Văn Động Lực Làm Việc Của Nhân Viên tạii Bảo Hiểm Xã Hội
Luận Văn Động Lực Làm Việc Của Nhân Viên tạii Bảo Hiểm Xã HộiLuận Văn Động Lực Làm Việc Của Nhân Viên tạii Bảo Hiểm Xã Hội
Luận Văn Động Lực Làm Việc Của Nhân Viên tạii Bảo Hiểm Xã Hội
 
Luận văn Thạc sĩ Ký túc xá trường chuyên Bắc Ninh
Luận văn Thạc sĩ Ký túc xá trường chuyên Bắc NinhLuận văn Thạc sĩ Ký túc xá trường chuyên Bắc Ninh
Luận văn Thạc sĩ Ký túc xá trường chuyên Bắc Ninh
 
Luận văn: Các nhân tố của chất lượng dịch vụ đào tạo ảnh hưởng đến sự hài lòn...
Luận văn: Các nhân tố của chất lượng dịch vụ đào tạo ảnh hưởng đến sự hài lòn...Luận văn: Các nhân tố của chất lượng dịch vụ đào tạo ảnh hưởng đến sự hài lòn...
Luận văn: Các nhân tố của chất lượng dịch vụ đào tạo ảnh hưởng đến sự hài lòn...
 
Nghiên cứu thực trạng sinh viên tốt nghiệp có việc làm
Nghiên cứu thực trạng sinh viên tốt nghiệp có việc làmNghiên cứu thực trạng sinh viên tốt nghiệp có việc làm
Nghiên cứu thực trạng sinh viên tốt nghiệp có việc làm
 
ĐỀ TÀI : Nhân tố ảnh hưởng đến dự định khởi sự kinh doanh của sinh viên
ĐỀ TÀI : Nhân tố ảnh hưởng đến dự định khởi sự kinh doanh của sinh viênĐỀ TÀI : Nhân tố ảnh hưởng đến dự định khởi sự kinh doanh của sinh viên
ĐỀ TÀI : Nhân tố ảnh hưởng đến dự định khởi sự kinh doanh của sinh viên
 
Nhân tố ảnh hưởng đến dự định khởi sự kinh doanh của sinh viên - Gửi miễn phí...
Nhân tố ảnh hưởng đến dự định khởi sự kinh doanh của sinh viên - Gửi miễn phí...Nhân tố ảnh hưởng đến dự định khởi sự kinh doanh của sinh viên - Gửi miễn phí...
Nhân tố ảnh hưởng đến dự định khởi sự kinh doanh của sinh viên - Gửi miễn phí...
 

More from Blue.Sky Blue.Sky

NẮM TRỌN CHUYÊN ĐỀ HÌNH HỌC TUYỂN SINH 9 VÀO 10
NẮM TRỌN CHUYÊN ĐỀ HÌNH HỌC TUYỂN SINH 9 VÀO 10NẮM TRỌN CHUYÊN ĐỀ HÌNH HỌC TUYỂN SINH 9 VÀO 10
NẮM TRỌN CHUYÊN ĐỀ HÌNH HỌC TUYỂN SINH 9 VÀO 10Blue.Sky Blue.Sky
 
TOÁN 9-CHUYÊN ĐỀ HÀM SỐ BẬC NHẤT VÀ PARABOL.pdf
TOÁN 9-CHUYÊN ĐỀ HÀM SỐ BẬC NHẤT VÀ PARABOL.pdfTOÁN 9-CHUYÊN ĐỀ HÀM SỐ BẬC NHẤT VÀ PARABOL.pdf
TOÁN 9-CHUYÊN ĐỀ HÀM SỐ BẬC NHẤT VÀ PARABOL.pdfBlue.Sky Blue.Sky
 
TOÁN 9-CHUYÊN ĐỀ HỆ THỨC VIET VÀ ỨNG DỤNG
TOÁN 9-CHUYÊN ĐỀ HỆ THỨC VIET VÀ ỨNG DỤNGTOÁN 9-CHUYÊN ĐỀ HỆ THỨC VIET VÀ ỨNG DỤNG
TOÁN 9-CHUYÊN ĐỀ HỆ THỨC VIET VÀ ỨNG DỤNGBlue.Sky Blue.Sky
 
ĐỀ THI THỬ MỚI NHẤT CÁC TRƯỜNG HÀ NỘI 2021-2022.
ĐỀ THI THỬ MỚI NHẤT CÁC TRƯỜNG HÀ NỘI 2021-2022.ĐỀ THI THỬ MỚI NHẤT CÁC TRƯỜNG HÀ NỘI 2021-2022.
ĐỀ THI THỬ MỚI NHẤT CÁC TRƯỜNG HÀ NỘI 2021-2022.Blue.Sky Blue.Sky
 
đề toán 6 học kì 2 các trường Hà Nội
đề toán 6 học kì 2 các trường Hà Nộiđề toán 6 học kì 2 các trường Hà Nội
đề toán 6 học kì 2 các trường Hà NộiBlue.Sky Blue.Sky
 
BỘ đề kiểm tra học kỳ 2 môn toán lớp 5 năm học 2021-2022.
BỘ đề kiểm tra học kỳ 2 môn toán lớp 5 năm học 2021-2022.BỘ đề kiểm tra học kỳ 2 môn toán lớp 5 năm học 2021-2022.
BỘ đề kiểm tra học kỳ 2 môn toán lớp 5 năm học 2021-2022.Blue.Sky Blue.Sky
 
chuyên đề hàm số và parabol toán 9 ôn thi vào 10.
chuyên đề hàm số và parabol toán 9 ôn thi vào 10.chuyên đề hàm số và parabol toán 9 ôn thi vào 10.
chuyên đề hàm số và parabol toán 9 ôn thi vào 10.Blue.Sky Blue.Sky
 
de-cuong-on-thi-hoc-ki-2-mon-toan-lop-1.
de-cuong-on-thi-hoc-ki-2-mon-toan-lop-1.de-cuong-on-thi-hoc-ki-2-mon-toan-lop-1.
de-cuong-on-thi-hoc-ki-2-mon-toan-lop-1.Blue.Sky Blue.Sky
 
de-cuong-on-tap-hoc-ki-2-mon-toan-lop-4.
de-cuong-on-tap-hoc-ki-2-mon-toan-lop-4.de-cuong-on-tap-hoc-ki-2-mon-toan-lop-4.
de-cuong-on-tap-hoc-ki-2-mon-toan-lop-4.Blue.Sky Blue.Sky
 
De-thi-hoc-ki-2-toan-6-sach-chan-troi-sang-tao.
De-thi-hoc-ki-2-toan-6-sach-chan-troi-sang-tao.De-thi-hoc-ki-2-toan-6-sach-chan-troi-sang-tao.
De-thi-hoc-ki-2-toan-6-sach-chan-troi-sang-tao.Blue.Sky Blue.Sky
 
ĐỀ THI TOÁN 6 HỌC KỲ 2
ĐỀ THI TOÁN 6 HỌC KỲ 2ĐỀ THI TOÁN 6 HỌC KỲ 2
ĐỀ THI TOÁN 6 HỌC KỲ 2Blue.Sky Blue.Sky
 
TÀI LIỆU ÔN THI VÀO LỚP 10 MÔN TIẾNG ANH 2022-2023
TÀI LIỆU ÔN THI VÀO LỚP 10 MÔN TIẾNG ANH 2022-2023TÀI LIỆU ÔN THI VÀO LỚP 10 MÔN TIẾNG ANH 2022-2023
TÀI LIỆU ÔN THI VÀO LỚP 10 MÔN TIẾNG ANH 2022-2023Blue.Sky Blue.Sky
 
Đề luyện tập môn tiếng Anh thi đánh giá năng lực vào lớp 10 Chuyên Ngoại Ngữ.pdf
Đề luyện tập môn tiếng Anh thi đánh giá năng lực vào lớp 10 Chuyên Ngoại Ngữ.pdfĐề luyện tập môn tiếng Anh thi đánh giá năng lực vào lớp 10 Chuyên Ngoại Ngữ.pdf
Đề luyện tập môn tiếng Anh thi đánh giá năng lực vào lớp 10 Chuyên Ngoại Ngữ.pdfBlue.Sky Blue.Sky
 
TOM TAT CONG THUC TOAN ON THI THPT QUOC GIA MON TOAN.pdf
TOM TAT CONG THUC TOAN ON THI THPT QUOC GIA MON TOAN.pdfTOM TAT CONG THUC TOAN ON THI THPT QUOC GIA MON TOAN.pdf
TOM TAT CONG THUC TOAN ON THI THPT QUOC GIA MON TOAN.pdfBlue.Sky Blue.Sky
 
Bài tập có đáp án chi tiết về kỹ thuật liên hợp trong giải phương trình môn t...
Bài tập có đáp án chi tiết về kỹ thuật liên hợp trong giải phương trình môn t...Bài tập có đáp án chi tiết về kỹ thuật liên hợp trong giải phương trình môn t...
Bài tập có đáp án chi tiết về kỹ thuật liên hợp trong giải phương trình môn t...Blue.Sky Blue.Sky
 
43 ĐỀ TUYỂN SINH LỚP 10 CÁC TỈNH CỰC HAY
43 ĐỀ TUYỂN SINH LỚP  10 CÁC TỈNH CỰC HAY43 ĐỀ TUYỂN SINH LỚP  10 CÁC TỈNH CỰC HAY
43 ĐỀ TUYỂN SINH LỚP 10 CÁC TỈNH CỰC HAYBlue.Sky Blue.Sky
 
45 ĐỀ TUYỂN SINH VÀO LỚP 10 NĂM 2012
45 ĐỀ TUYỂN SINH VÀO LỚP 10 NĂM 201245 ĐỀ TUYỂN SINH VÀO LỚP 10 NĂM 2012
45 ĐỀ TUYỂN SINH VÀO LỚP 10 NĂM 2012Blue.Sky Blue.Sky
 
40 DE THI VAO 10 HỆ CHUYÊN TỈNH BÌNH ĐỊNH
40 DE THI VAO 10 HỆ CHUYÊN TỈNH BÌNH ĐỊNH40 DE THI VAO 10 HỆ CHUYÊN TỈNH BÌNH ĐỊNH
40 DE THI VAO 10 HỆ CHUYÊN TỈNH BÌNH ĐỊNHBlue.Sky Blue.Sky
 
TUYỂN TẬP 16 ĐỀ THI THỬ MỚI NHẤT CÁC TRƯỜNG HÀ NỘI 2021-2022.pdf
TUYỂN TẬP 16 ĐỀ THI THỬ MỚI NHẤT CÁC TRƯỜNG HÀ NỘI 2021-2022.pdfTUYỂN TẬP 16 ĐỀ THI THỬ MỚI NHẤT CÁC TRƯỜNG HÀ NỘI 2021-2022.pdf
TUYỂN TẬP 16 ĐỀ THI THỬ MỚI NHẤT CÁC TRƯỜNG HÀ NỘI 2021-2022.pdfBlue.Sky Blue.Sky
 

More from Blue.Sky Blue.Sky (19)

NẮM TRỌN CHUYÊN ĐỀ HÌNH HỌC TUYỂN SINH 9 VÀO 10
NẮM TRỌN CHUYÊN ĐỀ HÌNH HỌC TUYỂN SINH 9 VÀO 10NẮM TRỌN CHUYÊN ĐỀ HÌNH HỌC TUYỂN SINH 9 VÀO 10
NẮM TRỌN CHUYÊN ĐỀ HÌNH HỌC TUYỂN SINH 9 VÀO 10
 
TOÁN 9-CHUYÊN ĐỀ HÀM SỐ BẬC NHẤT VÀ PARABOL.pdf
TOÁN 9-CHUYÊN ĐỀ HÀM SỐ BẬC NHẤT VÀ PARABOL.pdfTOÁN 9-CHUYÊN ĐỀ HÀM SỐ BẬC NHẤT VÀ PARABOL.pdf
TOÁN 9-CHUYÊN ĐỀ HÀM SỐ BẬC NHẤT VÀ PARABOL.pdf
 
TOÁN 9-CHUYÊN ĐỀ HỆ THỨC VIET VÀ ỨNG DỤNG
TOÁN 9-CHUYÊN ĐỀ HỆ THỨC VIET VÀ ỨNG DỤNGTOÁN 9-CHUYÊN ĐỀ HỆ THỨC VIET VÀ ỨNG DỤNG
TOÁN 9-CHUYÊN ĐỀ HỆ THỨC VIET VÀ ỨNG DỤNG
 
ĐỀ THI THỬ MỚI NHẤT CÁC TRƯỜNG HÀ NỘI 2021-2022.
ĐỀ THI THỬ MỚI NHẤT CÁC TRƯỜNG HÀ NỘI 2021-2022.ĐỀ THI THỬ MỚI NHẤT CÁC TRƯỜNG HÀ NỘI 2021-2022.
ĐỀ THI THỬ MỚI NHẤT CÁC TRƯỜNG HÀ NỘI 2021-2022.
 
đề toán 6 học kì 2 các trường Hà Nội
đề toán 6 học kì 2 các trường Hà Nộiđề toán 6 học kì 2 các trường Hà Nội
đề toán 6 học kì 2 các trường Hà Nội
 
BỘ đề kiểm tra học kỳ 2 môn toán lớp 5 năm học 2021-2022.
BỘ đề kiểm tra học kỳ 2 môn toán lớp 5 năm học 2021-2022.BỘ đề kiểm tra học kỳ 2 môn toán lớp 5 năm học 2021-2022.
BỘ đề kiểm tra học kỳ 2 môn toán lớp 5 năm học 2021-2022.
 
chuyên đề hàm số và parabol toán 9 ôn thi vào 10.
chuyên đề hàm số và parabol toán 9 ôn thi vào 10.chuyên đề hàm số và parabol toán 9 ôn thi vào 10.
chuyên đề hàm số và parabol toán 9 ôn thi vào 10.
 
de-cuong-on-thi-hoc-ki-2-mon-toan-lop-1.
de-cuong-on-thi-hoc-ki-2-mon-toan-lop-1.de-cuong-on-thi-hoc-ki-2-mon-toan-lop-1.
de-cuong-on-thi-hoc-ki-2-mon-toan-lop-1.
 
de-cuong-on-tap-hoc-ki-2-mon-toan-lop-4.
de-cuong-on-tap-hoc-ki-2-mon-toan-lop-4.de-cuong-on-tap-hoc-ki-2-mon-toan-lop-4.
de-cuong-on-tap-hoc-ki-2-mon-toan-lop-4.
 
De-thi-hoc-ki-2-toan-6-sach-chan-troi-sang-tao.
De-thi-hoc-ki-2-toan-6-sach-chan-troi-sang-tao.De-thi-hoc-ki-2-toan-6-sach-chan-troi-sang-tao.
De-thi-hoc-ki-2-toan-6-sach-chan-troi-sang-tao.
 
ĐỀ THI TOÁN 6 HỌC KỲ 2
ĐỀ THI TOÁN 6 HỌC KỲ 2ĐỀ THI TOÁN 6 HỌC KỲ 2
ĐỀ THI TOÁN 6 HỌC KỲ 2
 
TÀI LIỆU ÔN THI VÀO LỚP 10 MÔN TIẾNG ANH 2022-2023
TÀI LIỆU ÔN THI VÀO LỚP 10 MÔN TIẾNG ANH 2022-2023TÀI LIỆU ÔN THI VÀO LỚP 10 MÔN TIẾNG ANH 2022-2023
TÀI LIỆU ÔN THI VÀO LỚP 10 MÔN TIẾNG ANH 2022-2023
 
Đề luyện tập môn tiếng Anh thi đánh giá năng lực vào lớp 10 Chuyên Ngoại Ngữ.pdf
Đề luyện tập môn tiếng Anh thi đánh giá năng lực vào lớp 10 Chuyên Ngoại Ngữ.pdfĐề luyện tập môn tiếng Anh thi đánh giá năng lực vào lớp 10 Chuyên Ngoại Ngữ.pdf
Đề luyện tập môn tiếng Anh thi đánh giá năng lực vào lớp 10 Chuyên Ngoại Ngữ.pdf
 
TOM TAT CONG THUC TOAN ON THI THPT QUOC GIA MON TOAN.pdf
TOM TAT CONG THUC TOAN ON THI THPT QUOC GIA MON TOAN.pdfTOM TAT CONG THUC TOAN ON THI THPT QUOC GIA MON TOAN.pdf
TOM TAT CONG THUC TOAN ON THI THPT QUOC GIA MON TOAN.pdf
 
Bài tập có đáp án chi tiết về kỹ thuật liên hợp trong giải phương trình môn t...
Bài tập có đáp án chi tiết về kỹ thuật liên hợp trong giải phương trình môn t...Bài tập có đáp án chi tiết về kỹ thuật liên hợp trong giải phương trình môn t...
Bài tập có đáp án chi tiết về kỹ thuật liên hợp trong giải phương trình môn t...
 
43 ĐỀ TUYỂN SINH LỚP 10 CÁC TỈNH CỰC HAY
43 ĐỀ TUYỂN SINH LỚP  10 CÁC TỈNH CỰC HAY43 ĐỀ TUYỂN SINH LỚP  10 CÁC TỈNH CỰC HAY
43 ĐỀ TUYỂN SINH LỚP 10 CÁC TỈNH CỰC HAY
 
45 ĐỀ TUYỂN SINH VÀO LỚP 10 NĂM 2012
45 ĐỀ TUYỂN SINH VÀO LỚP 10 NĂM 201245 ĐỀ TUYỂN SINH VÀO LỚP 10 NĂM 2012
45 ĐỀ TUYỂN SINH VÀO LỚP 10 NĂM 2012
 
40 DE THI VAO 10 HỆ CHUYÊN TỈNH BÌNH ĐỊNH
40 DE THI VAO 10 HỆ CHUYÊN TỈNH BÌNH ĐỊNH40 DE THI VAO 10 HỆ CHUYÊN TỈNH BÌNH ĐỊNH
40 DE THI VAO 10 HỆ CHUYÊN TỈNH BÌNH ĐỊNH
 
TUYỂN TẬP 16 ĐỀ THI THỬ MỚI NHẤT CÁC TRƯỜNG HÀ NỘI 2021-2022.pdf
TUYỂN TẬP 16 ĐỀ THI THỬ MỚI NHẤT CÁC TRƯỜNG HÀ NỘI 2021-2022.pdfTUYỂN TẬP 16 ĐỀ THI THỬ MỚI NHẤT CÁC TRƯỜNG HÀ NỘI 2021-2022.pdf
TUYỂN TẬP 16 ĐỀ THI THỬ MỚI NHẤT CÁC TRƯỜNG HÀ NỘI 2021-2022.pdf
 

Recently uploaded

Đề cương môn giải phẫu......................
Đề cương môn giải phẫu......................Đề cương môn giải phẫu......................
Đề cương môn giải phẫu......................TrnHoa46
 
TÀI LIỆU BỒI DƯỠNG HỌC SINH GIỎI LÝ LUẬN VĂN HỌC NĂM HỌC 2023-2024 - MÔN NGỮ ...
TÀI LIỆU BỒI DƯỠNG HỌC SINH GIỎI LÝ LUẬN VĂN HỌC NĂM HỌC 2023-2024 - MÔN NGỮ ...TÀI LIỆU BỒI DƯỠNG HỌC SINH GIỎI LÝ LUẬN VĂN HỌC NĂM HỌC 2023-2024 - MÔN NGỮ ...
TÀI LIỆU BỒI DƯỠNG HỌC SINH GIỎI LÝ LUẬN VĂN HỌC NĂM HỌC 2023-2024 - MÔN NGỮ ...Nguyen Thanh Tu Collection
 
BỘ LUYỆN NGHE VÀO 10 TIẾNG ANH DẠNG TRẮC NGHIỆM 4 CÂU TRẢ LỜI - CÓ FILE NGHE.pdf
BỘ LUYỆN NGHE VÀO 10 TIẾNG ANH DẠNG TRẮC NGHIỆM 4 CÂU TRẢ LỜI - CÓ FILE NGHE.pdfBỘ LUYỆN NGHE VÀO 10 TIẾNG ANH DẠNG TRẮC NGHIỆM 4 CÂU TRẢ LỜI - CÓ FILE NGHE.pdf
BỘ LUYỆN NGHE VÀO 10 TIẾNG ANH DẠNG TRẮC NGHIỆM 4 CÂU TRẢ LỜI - CÓ FILE NGHE.pdfNguyen Thanh Tu Collection
 
GIÁO TRÌNH KHỐI NGUỒN CÁC LOẠI - ĐIỆN LẠNH BÁCH KHOA HÀ NỘI
GIÁO TRÌNH  KHỐI NGUỒN CÁC LOẠI - ĐIỆN LẠNH BÁCH KHOA HÀ NỘIGIÁO TRÌNH  KHỐI NGUỒN CÁC LOẠI - ĐIỆN LẠNH BÁCH KHOA HÀ NỘI
GIÁO TRÌNH KHỐI NGUỒN CÁC LOẠI - ĐIỆN LẠNH BÁCH KHOA HÀ NỘIĐiện Lạnh Bách Khoa Hà Nội
 
1.DOANNGOCPHUONGTHAO-APDUNGSTEMTHIETKEBTHHHGIUPHSHOCHIEUQUA (1).docx
1.DOANNGOCPHUONGTHAO-APDUNGSTEMTHIETKEBTHHHGIUPHSHOCHIEUQUA (1).docx1.DOANNGOCPHUONGTHAO-APDUNGSTEMTHIETKEBTHHHGIUPHSHOCHIEUQUA (1).docx
1.DOANNGOCPHUONGTHAO-APDUNGSTEMTHIETKEBTHHHGIUPHSHOCHIEUQUA (1).docxTHAO316680
 
GIÁO ÁN DẠY THÊM (KẾ HOẠCH BÀI DẠY BUỔI 2) - TIẾNG ANH 7 GLOBAL SUCCESS (2 CỘ...
GIÁO ÁN DẠY THÊM (KẾ HOẠCH BÀI DẠY BUỔI 2) - TIẾNG ANH 7 GLOBAL SUCCESS (2 CỘ...GIÁO ÁN DẠY THÊM (KẾ HOẠCH BÀI DẠY BUỔI 2) - TIẾNG ANH 7 GLOBAL SUCCESS (2 CỘ...
GIÁO ÁN DẠY THÊM (KẾ HOẠCH BÀI DẠY BUỔI 2) - TIẾNG ANH 7 GLOBAL SUCCESS (2 CỘ...Nguyen Thanh Tu Collection
 
sách sinh học đại cương - Textbook.pdf
sách sinh học đại cương   -   Textbook.pdfsách sinh học đại cương   -   Textbook.pdf
sách sinh học đại cương - Textbook.pdfTrnHoa46
 
3-BẢNG MÃ LỖI CỦA CÁC HÃNG ĐIỀU HÒA .pdf - ĐIỆN LẠNH BÁCH KHOA HÀ NỘI
3-BẢNG MÃ LỖI CỦA CÁC HÃNG ĐIỀU HÒA .pdf - ĐIỆN LẠNH BÁCH KHOA HÀ NỘI3-BẢNG MÃ LỖI CỦA CÁC HÃNG ĐIỀU HÒA .pdf - ĐIỆN LẠNH BÁCH KHOA HÀ NỘI
3-BẢNG MÃ LỖI CỦA CÁC HÃNG ĐIỀU HÒA .pdf - ĐIỆN LẠNH BÁCH KHOA HÀ NỘIĐiện Lạnh Bách Khoa Hà Nội
 
SÁNG KIẾN ÁP DỤNG CLT (COMMUNICATIVE LANGUAGE TEACHING) VÀO QUÁ TRÌNH DẠY - H...
SÁNG KIẾN ÁP DỤNG CLT (COMMUNICATIVE LANGUAGE TEACHING) VÀO QUÁ TRÌNH DẠY - H...SÁNG KIẾN ÁP DỤNG CLT (COMMUNICATIVE LANGUAGE TEACHING) VÀO QUÁ TRÌNH DẠY - H...
SÁNG KIẾN ÁP DỤNG CLT (COMMUNICATIVE LANGUAGE TEACHING) VÀO QUÁ TRÌNH DẠY - H...Nguyen Thanh Tu Collection
 
ĐỀ CHÍNH THỨC KỲ THI TUYỂN SINH VÀO LỚP 10 THPT CÁC TỈNH THÀNH NĂM HỌC 2020 –...
ĐỀ CHÍNH THỨC KỲ THI TUYỂN SINH VÀO LỚP 10 THPT CÁC TỈNH THÀNH NĂM HỌC 2020 –...ĐỀ CHÍNH THỨC KỲ THI TUYỂN SINH VÀO LỚP 10 THPT CÁC TỈNH THÀNH NĂM HỌC 2020 –...
ĐỀ CHÍNH THỨC KỲ THI TUYỂN SINH VÀO LỚP 10 THPT CÁC TỈNH THÀNH NĂM HỌC 2020 –...Nguyen Thanh Tu Collection
 
PHƯƠNG THỨC VẬN TẢI ĐƯỜNG SẮT TRONG VẬN TẢI
PHƯƠNG THỨC VẬN TẢI ĐƯỜNG SẮT TRONG VẬN TẢIPHƯƠNG THỨC VẬN TẢI ĐƯỜNG SẮT TRONG VẬN TẢI
PHƯƠNG THỨC VẬN TẢI ĐƯỜNG SẮT TRONG VẬN TẢImyvh40253
 
30 ĐỀ PHÁT TRIỂN THEO CẤU TRÚC ĐỀ MINH HỌA BGD NGÀY 22-3-2024 KỲ THI TỐT NGHI...
30 ĐỀ PHÁT TRIỂN THEO CẤU TRÚC ĐỀ MINH HỌA BGD NGÀY 22-3-2024 KỲ THI TỐT NGHI...30 ĐỀ PHÁT TRIỂN THEO CẤU TRÚC ĐỀ MINH HỌA BGD NGÀY 22-3-2024 KỲ THI TỐT NGHI...
30 ĐỀ PHÁT TRIỂN THEO CẤU TRÚC ĐỀ MINH HỌA BGD NGÀY 22-3-2024 KỲ THI TỐT NGHI...Nguyen Thanh Tu Collection
 
kinh tế chính trị mác lênin chương hai và hàng hoá và sxxhh
kinh tế chính trị mác lênin chương hai và hàng hoá và sxxhhkinh tế chính trị mác lênin chương hai và hàng hoá và sxxhh
kinh tế chính trị mác lênin chương hai và hàng hoá và sxxhhdtlnnm
 
TÀI LIỆU BỒI DƯỠNG HỌC SINH GIỎI KỸ NĂNG VIẾT ĐOẠN VĂN NGHỊ LUẬN XÃ HỘI 200 C...
TÀI LIỆU BỒI DƯỠNG HỌC SINH GIỎI KỸ NĂNG VIẾT ĐOẠN VĂN NGHỊ LUẬN XÃ HỘI 200 C...TÀI LIỆU BỒI DƯỠNG HỌC SINH GIỎI KỸ NĂNG VIẾT ĐOẠN VĂN NGHỊ LUẬN XÃ HỘI 200 C...
TÀI LIỆU BỒI DƯỠNG HỌC SINH GIỎI KỸ NĂNG VIẾT ĐOẠN VĂN NGHỊ LUẬN XÃ HỘI 200 C...Nguyen Thanh Tu Collection
 
Campbell _2011_ - Sinh học - Tế bào - Ref.pdf
Campbell _2011_ - Sinh học - Tế bào - Ref.pdfCampbell _2011_ - Sinh học - Tế bào - Ref.pdf
Campbell _2011_ - Sinh học - Tế bào - Ref.pdfTrnHoa46
 
30 ĐỀ PHÁT TRIỂN THEO CẤU TRÚC ĐỀ MINH HỌA BGD NGÀY 22-3-2024 KỲ THI TỐT NGHI...
30 ĐỀ PHÁT TRIỂN THEO CẤU TRÚC ĐỀ MINH HỌA BGD NGÀY 22-3-2024 KỲ THI TỐT NGHI...30 ĐỀ PHÁT TRIỂN THEO CẤU TRÚC ĐỀ MINH HỌA BGD NGÀY 22-3-2024 KỲ THI TỐT NGHI...
30 ĐỀ PHÁT TRIỂN THEO CẤU TRÚC ĐỀ MINH HỌA BGD NGÀY 22-3-2024 KỲ THI TỐT NGHI...Nguyen Thanh Tu Collection
 
powerpoint mẫu họp phụ huynh cuối kì 2 học sinh lớp 7 bgs
powerpoint mẫu họp phụ huynh cuối kì 2 học sinh lớp 7 bgspowerpoint mẫu họp phụ huynh cuối kì 2 học sinh lớp 7 bgs
powerpoint mẫu họp phụ huynh cuối kì 2 học sinh lớp 7 bgsNmmeomeo
 
30 ĐỀ PHÁT TRIỂN THEO CẤU TRÚC ĐỀ MINH HỌA BGD NGÀY 22-3-2024 KỲ THI TỐT NGHI...
30 ĐỀ PHÁT TRIỂN THEO CẤU TRÚC ĐỀ MINH HỌA BGD NGÀY 22-3-2024 KỲ THI TỐT NGHI...30 ĐỀ PHÁT TRIỂN THEO CẤU TRÚC ĐỀ MINH HỌA BGD NGÀY 22-3-2024 KỲ THI TỐT NGHI...
30 ĐỀ PHÁT TRIỂN THEO CẤU TRÚC ĐỀ MINH HỌA BGD NGÀY 22-3-2024 KỲ THI TỐT NGHI...Nguyen Thanh Tu Collection
 
Các điều kiện bảo hiểm trong bảo hiểm hàng hoá
Các điều kiện bảo hiểm trong bảo hiểm hàng hoáCác điều kiện bảo hiểm trong bảo hiểm hàng hoá
Các điều kiện bảo hiểm trong bảo hiểm hàng hoámyvh40253
 

Recently uploaded (20)

Đề cương môn giải phẫu......................
Đề cương môn giải phẫu......................Đề cương môn giải phẫu......................
Đề cương môn giải phẫu......................
 
TÀI LIỆU BỒI DƯỠNG HỌC SINH GIỎI LÝ LUẬN VĂN HỌC NĂM HỌC 2023-2024 - MÔN NGỮ ...
TÀI LIỆU BỒI DƯỠNG HỌC SINH GIỎI LÝ LUẬN VĂN HỌC NĂM HỌC 2023-2024 - MÔN NGỮ ...TÀI LIỆU BỒI DƯỠNG HỌC SINH GIỎI LÝ LUẬN VĂN HỌC NĂM HỌC 2023-2024 - MÔN NGỮ ...
TÀI LIỆU BỒI DƯỠNG HỌC SINH GIỎI LÝ LUẬN VĂN HỌC NĂM HỌC 2023-2024 - MÔN NGỮ ...
 
BỘ LUYỆN NGHE VÀO 10 TIẾNG ANH DẠNG TRẮC NGHIỆM 4 CÂU TRẢ LỜI - CÓ FILE NGHE.pdf
BỘ LUYỆN NGHE VÀO 10 TIẾNG ANH DẠNG TRẮC NGHIỆM 4 CÂU TRẢ LỜI - CÓ FILE NGHE.pdfBỘ LUYỆN NGHE VÀO 10 TIẾNG ANH DẠNG TRẮC NGHIỆM 4 CÂU TRẢ LỜI - CÓ FILE NGHE.pdf
BỘ LUYỆN NGHE VÀO 10 TIẾNG ANH DẠNG TRẮC NGHIỆM 4 CÂU TRẢ LỜI - CÓ FILE NGHE.pdf
 
GIÁO TRÌNH KHỐI NGUỒN CÁC LOẠI - ĐIỆN LẠNH BÁCH KHOA HÀ NỘI
GIÁO TRÌNH  KHỐI NGUỒN CÁC LOẠI - ĐIỆN LẠNH BÁCH KHOA HÀ NỘIGIÁO TRÌNH  KHỐI NGUỒN CÁC LOẠI - ĐIỆN LẠNH BÁCH KHOA HÀ NỘI
GIÁO TRÌNH KHỐI NGUỒN CÁC LOẠI - ĐIỆN LẠNH BÁCH KHOA HÀ NỘI
 
1.DOANNGOCPHUONGTHAO-APDUNGSTEMTHIETKEBTHHHGIUPHSHOCHIEUQUA (1).docx
1.DOANNGOCPHUONGTHAO-APDUNGSTEMTHIETKEBTHHHGIUPHSHOCHIEUQUA (1).docx1.DOANNGOCPHUONGTHAO-APDUNGSTEMTHIETKEBTHHHGIUPHSHOCHIEUQUA (1).docx
1.DOANNGOCPHUONGTHAO-APDUNGSTEMTHIETKEBTHHHGIUPHSHOCHIEUQUA (1).docx
 
GIÁO ÁN DẠY THÊM (KẾ HOẠCH BÀI DẠY BUỔI 2) - TIẾNG ANH 7 GLOBAL SUCCESS (2 CỘ...
GIÁO ÁN DẠY THÊM (KẾ HOẠCH BÀI DẠY BUỔI 2) - TIẾNG ANH 7 GLOBAL SUCCESS (2 CỘ...GIÁO ÁN DẠY THÊM (KẾ HOẠCH BÀI DẠY BUỔI 2) - TIẾNG ANH 7 GLOBAL SUCCESS (2 CỘ...
GIÁO ÁN DẠY THÊM (KẾ HOẠCH BÀI DẠY BUỔI 2) - TIẾNG ANH 7 GLOBAL SUCCESS (2 CỘ...
 
sách sinh học đại cương - Textbook.pdf
sách sinh học đại cương   -   Textbook.pdfsách sinh học đại cương   -   Textbook.pdf
sách sinh học đại cương - Textbook.pdf
 
3-BẢNG MÃ LỖI CỦA CÁC HÃNG ĐIỀU HÒA .pdf - ĐIỆN LẠNH BÁCH KHOA HÀ NỘI
3-BẢNG MÃ LỖI CỦA CÁC HÃNG ĐIỀU HÒA .pdf - ĐIỆN LẠNH BÁCH KHOA HÀ NỘI3-BẢNG MÃ LỖI CỦA CÁC HÃNG ĐIỀU HÒA .pdf - ĐIỆN LẠNH BÁCH KHOA HÀ NỘI
3-BẢNG MÃ LỖI CỦA CÁC HÃNG ĐIỀU HÒA .pdf - ĐIỆN LẠNH BÁCH KHOA HÀ NỘI
 
SÁNG KIẾN ÁP DỤNG CLT (COMMUNICATIVE LANGUAGE TEACHING) VÀO QUÁ TRÌNH DẠY - H...
SÁNG KIẾN ÁP DỤNG CLT (COMMUNICATIVE LANGUAGE TEACHING) VÀO QUÁ TRÌNH DẠY - H...SÁNG KIẾN ÁP DỤNG CLT (COMMUNICATIVE LANGUAGE TEACHING) VÀO QUÁ TRÌNH DẠY - H...
SÁNG KIẾN ÁP DỤNG CLT (COMMUNICATIVE LANGUAGE TEACHING) VÀO QUÁ TRÌNH DẠY - H...
 
ĐỀ CHÍNH THỨC KỲ THI TUYỂN SINH VÀO LỚP 10 THPT CÁC TỈNH THÀNH NĂM HỌC 2020 –...
ĐỀ CHÍNH THỨC KỲ THI TUYỂN SINH VÀO LỚP 10 THPT CÁC TỈNH THÀNH NĂM HỌC 2020 –...ĐỀ CHÍNH THỨC KỲ THI TUYỂN SINH VÀO LỚP 10 THPT CÁC TỈNH THÀNH NĂM HỌC 2020 –...
ĐỀ CHÍNH THỨC KỲ THI TUYỂN SINH VÀO LỚP 10 THPT CÁC TỈNH THÀNH NĂM HỌC 2020 –...
 
PHƯƠNG THỨC VẬN TẢI ĐƯỜNG SẮT TRONG VẬN TẢI
PHƯƠNG THỨC VẬN TẢI ĐƯỜNG SẮT TRONG VẬN TẢIPHƯƠNG THỨC VẬN TẢI ĐƯỜNG SẮT TRONG VẬN TẢI
PHƯƠNG THỨC VẬN TẢI ĐƯỜNG SẮT TRONG VẬN TẢI
 
30 ĐỀ PHÁT TRIỂN THEO CẤU TRÚC ĐỀ MINH HỌA BGD NGÀY 22-3-2024 KỲ THI TỐT NGHI...
30 ĐỀ PHÁT TRIỂN THEO CẤU TRÚC ĐỀ MINH HỌA BGD NGÀY 22-3-2024 KỲ THI TỐT NGHI...30 ĐỀ PHÁT TRIỂN THEO CẤU TRÚC ĐỀ MINH HỌA BGD NGÀY 22-3-2024 KỲ THI TỐT NGHI...
30 ĐỀ PHÁT TRIỂN THEO CẤU TRÚC ĐỀ MINH HỌA BGD NGÀY 22-3-2024 KỲ THI TỐT NGHI...
 
kinh tế chính trị mác lênin chương hai và hàng hoá và sxxhh
kinh tế chính trị mác lênin chương hai và hàng hoá và sxxhhkinh tế chính trị mác lênin chương hai và hàng hoá và sxxhh
kinh tế chính trị mác lênin chương hai và hàng hoá và sxxhh
 
TÀI LIỆU BỒI DƯỠNG HỌC SINH GIỎI KỸ NĂNG VIẾT ĐOẠN VĂN NGHỊ LUẬN XÃ HỘI 200 C...
TÀI LIỆU BỒI DƯỠNG HỌC SINH GIỎI KỸ NĂNG VIẾT ĐOẠN VĂN NGHỊ LUẬN XÃ HỘI 200 C...TÀI LIỆU BỒI DƯỠNG HỌC SINH GIỎI KỸ NĂNG VIẾT ĐOẠN VĂN NGHỊ LUẬN XÃ HỘI 200 C...
TÀI LIỆU BỒI DƯỠNG HỌC SINH GIỎI KỸ NĂNG VIẾT ĐOẠN VĂN NGHỊ LUẬN XÃ HỘI 200 C...
 
1 - MÃ LỖI SỬA CHỮA BOARD MẠCH BẾP TỪ.pdf
1 - MÃ LỖI SỬA CHỮA BOARD MẠCH BẾP TỪ.pdf1 - MÃ LỖI SỬA CHỮA BOARD MẠCH BẾP TỪ.pdf
1 - MÃ LỖI SỬA CHỮA BOARD MẠCH BẾP TỪ.pdf
 
Campbell _2011_ - Sinh học - Tế bào - Ref.pdf
Campbell _2011_ - Sinh học - Tế bào - Ref.pdfCampbell _2011_ - Sinh học - Tế bào - Ref.pdf
Campbell _2011_ - Sinh học - Tế bào - Ref.pdf
 
30 ĐỀ PHÁT TRIỂN THEO CẤU TRÚC ĐỀ MINH HỌA BGD NGÀY 22-3-2024 KỲ THI TỐT NGHI...
30 ĐỀ PHÁT TRIỂN THEO CẤU TRÚC ĐỀ MINH HỌA BGD NGÀY 22-3-2024 KỲ THI TỐT NGHI...30 ĐỀ PHÁT TRIỂN THEO CẤU TRÚC ĐỀ MINH HỌA BGD NGÀY 22-3-2024 KỲ THI TỐT NGHI...
30 ĐỀ PHÁT TRIỂN THEO CẤU TRÚC ĐỀ MINH HỌA BGD NGÀY 22-3-2024 KỲ THI TỐT NGHI...
 
powerpoint mẫu họp phụ huynh cuối kì 2 học sinh lớp 7 bgs
powerpoint mẫu họp phụ huynh cuối kì 2 học sinh lớp 7 bgspowerpoint mẫu họp phụ huynh cuối kì 2 học sinh lớp 7 bgs
powerpoint mẫu họp phụ huynh cuối kì 2 học sinh lớp 7 bgs
 
30 ĐỀ PHÁT TRIỂN THEO CẤU TRÚC ĐỀ MINH HỌA BGD NGÀY 22-3-2024 KỲ THI TỐT NGHI...
30 ĐỀ PHÁT TRIỂN THEO CẤU TRÚC ĐỀ MINH HỌA BGD NGÀY 22-3-2024 KỲ THI TỐT NGHI...30 ĐỀ PHÁT TRIỂN THEO CẤU TRÚC ĐỀ MINH HỌA BGD NGÀY 22-3-2024 KỲ THI TỐT NGHI...
30 ĐỀ PHÁT TRIỂN THEO CẤU TRÚC ĐỀ MINH HỌA BGD NGÀY 22-3-2024 KỲ THI TỐT NGHI...
 
Các điều kiện bảo hiểm trong bảo hiểm hàng hoá
Các điều kiện bảo hiểm trong bảo hiểm hàng hoáCác điều kiện bảo hiểm trong bảo hiểm hàng hoá
Các điều kiện bảo hiểm trong bảo hiểm hàng hoá
 

BỘ ĐỀ TUYỂN SINH VÀO 10 THÀNH PHỐ HỒ CHÍ MINH

  • 1. TÀI LIỆU LUYỆN THI TUYỂN SINH LỚP 10 – MÔN TOÁN [lưu hành nội bộ] TP HCM – 2012
  • 2. 1 MỤC LỤC I. MỘT SỐ THÔNG TIN CẦN THAM KHẢO....................................................................... 4 1. Điểm chuẩn năm học 2009-2010...................................................................................... 4 2. Điểm chuẩn năm học 2010-2011...................................................................................... 7 3. Điểm chuẩn năm học 2011-2012.................................................................................... 10 4. Bí quyết đạt "điểm rơi phong độ" ngay trong ngày thi.................................................... 13 II. Đề thi tuyển sinh lớp 10 thành phố Hồ Chí Minh .............................................................. 15 1. Đề thi tuyển sinh lớp 10 tp Hồ Chí Minh Năm học 2011-2012....................................... 15 2. Đề thi tuyển sinh lớp 10 tp Hồ Chí Minh Năm học 2010-2011....................................... 15 3. Đề thi tuyển sinh lớp 10 tp Hồ Chí Minh Năm học 2009-2010....................................... 16 4. Đề thi tuyển sinh lớp 10 tp Hồ Chí Minh Năm học 2008-2009....................................... 17 5. Đề thi tuyển sinh lớp 10 tp Hồ Chí Minh Năm học 2007-2008....................................... 18 6. Đề thi tuyển sinh lớp 10 tp Hồ Chí Minh Năm học 2006-2007....................................... 18 III. Đề thi tuyển sinh lớp 10 Chuyên thành phố Hồ Chí Minh................................................ 19 1. Chuyên tp Hồ Chí Minh, năm học 2011-2012(N/A)....................................................... 19 2. Chuyên tp Hồ Chí Minh, năm học 2010-2011(N/A)....................................................... 19 3. Chuyên tp Hồ Chí Minh, năm học 2009-2010................................................................ 19 4. Chuyên tp Hồ Chí Minh, năm học 2008-2009(N/A)....................................................... 21 5. Chuyên tp Hồ Chí Minh, năm học 2007-2008................................................................ 21 6. Chuyên tp Hồ Chí Minh, năm học 2006-2007................................................................ 21 7. Chuyên Lê Hồng Phong tp Hồ Chí Minh, năm học 2001-2002....................................... 22 8. Chuyên Lê Hồng Phong tp Hồ Chí Minh, năm học 2002-2003....................................... 23 9. Chuyên Lê Hồng Phong tp Hồ Chí Minh, năm học 2003-2004....................................... 24 10. Chuyên Lê Hồng Phong tp Hồ Chí Minh, năm học 2004-2005..................................... 25 11. Chuyên Đại học Sư Phạm tp Hồ Chí Minh, năm học 2006-2007(N/A)......................... 26 12. Chuyên Trần Đại Nghĩa tp Hồ Chí Minh, năm học....................................................... 26
  • 3. 2 IV. Đề thi tuyển sinh lớp 10 trường Phổ Thông Năng Khiếu.................................................. 26 1. Toán không chuyên trường Phổ Thông Năng Khiếu, năm học 2011-2012...................... 26 2. Toán không chuyên trường Phổ Thông Năng Khiếu, năm học 2010-2011...................... 27 3. Toán không chuyên trường Phổ Thông Năng Khiếu, năm học 2009-2010...................... 28 4. Toán AB trường Phổ Thông Năng Khiếu, năm học 2008-2009 ...................................... 29 5. Toán CD trường Phổ Thông Năng Khiếu, năm học 2008-2009 ...................................... 30 6. Toán AB trường Phổ Thông Năng Khiếu, năm học 2007-2008 ...................................... 31 7. Toán AB trường Phổ Thông Năng Khiếu, năm học 2006-2007(N/A)............................. 32 8. Toán CD trường Phổ Thông Năng Khiếu, năm học 2006-2007(N/A)............................. 32 9. Toán AB trường Phổ Thông Năng Khiếu, năm học 2005-2006(N/A)............................. 32 10. Toán CD trường Phổ Thông Năng Khiếu, năm học 2005-2006(N/A)........................... 32 11. Toán AB trường Phổ Thông Năng Khiếu, năm học 2004-2005 .................................... 33 12. Toán CD trường Phổ Thông Năng Khiếu, năm học 2004-2005 .................................... 34 V. Đề thi tuyển sinh lớp 10 trên toàn quốc............................................................................. 35 1. Tuyển sinh lớp 10 thành phố Đà Nẵng năm học 2011-2012 ........................................... 35 2. Tuyển sinh lớp 10 thành phố Đà Nẵng năm học 2010-2011 ........................................... 36 3. Tuyển sinh lớp 10 thành phố Đà Nẵng năm học 2009-2010 ........................................... 36 4. Tuyển sinh lớp 10 thành phố Hà Nội năm học 2010-2011.............................................. 37 5. Tuyển sinh lớp 10 thành phố Hà Nội năm học 2011-2012.............................................. 38 6. Tuyển sinh lớp 10 tỉnh Hải Dương năm học 2011-2012 ................................................. 39 7. Tuyển sinh lớp 10 tỉnh Hà Tĩnh năm học 2010-2011...................................................... 40 8. Tuyển sinh lớp 10 tỉnh Nghệ An năm học 2011-2012 .................................................... 40 9. Tuyển sinh lớp 10 tỉnh Hà Tĩnh năm học 2011-2012...................................................... 41 10. Tuyển sinh lớp 10 tỉnh Bình Định năm học 2011-2012 ................................................ 42 11. Tuyển sinh lớp 10 tỉnh Quảng Nam năm học 2011-2012.............................................. 42 12. Tuyển sinh lớp 10 tỉnh Quảng Ngãi năm học 2011-2012.............................................. 43
  • 4. 3 13. Tuyển sinh lớp 10 tỉnh Thanh Hoá năm học 2011-2012 ............................................... 44 14. Tuyển sinh lớp 10 tỉnh Khánh Hoà năm học 2011-2012............................................... 44 15. Tuyển sinh lớp 10 tỉnh Bình Dương năm học 2010-2011 ............................................. 45 16. Tuyển sinh lớp 10 tỉnh Bến Tre năm học 2011-2012.................................................... 46 VI. Đề tham khảo ôn tập tuyển sinh....................................................................................... 48 1. ĐỀ SỐ 01....................................................................................................................... 48 2. ĐỀ SỐ 02....................................................................................................................... 48 3. ĐỀ SỐ 03....................................................................................................................... 49 4. ĐỀ SỐ 04....................................................................................................................... 50 5. ĐỀ SỐ 05....................................................................................................................... 51 6. ĐỀ SỐ 06....................................................................................................................... 52 7. ĐỀ SỐ 07....................................................................................................................... 53 8. ĐỀ SỐ 08....................................................................................................................... 54 9. ĐỀ SỐ 09....................................................................................................................... 55 10. ĐỀ SỐ 10..................................................................................................................... 56 11. ĐỀ SỐ 11..................................................................................................................... 57 12. ĐỀ SỐ 12..................................................................................................................... 57 13. ĐỀ SỐ 13..................................................................................................................... 58 14. ĐỀ SỐ 14..................................................................................................................... 59 15. ĐỀ SỐ 15..................................................................................................................... 60 16. ĐỀ SỐ 16..................................................................................................................... 61 17. ĐỀ SỐ 17..................................................................................................................... 62 18. ĐỀ SỐ 18..................................................................................................................... 63 19. ĐỀ SỐ 19..................................................................................................................... 63 20. ĐỀ SỐ 20..................................................................................................................... 64 21. ĐỀ SỐ 21..................................................................................................................... 65
  • 5. 4 I. MỘT SỐ THÔNG TIN CẦN THAM KHẢO 1. Điểm chuẩn năm học 2009-2010 Tên trường Điểm chuẩn NV1 NV2 NV3 THPT Trưng Vương 36.75 37.25 37.25 THPT Bùi Thị Xuân 40.00 41.00 42.00 THPT Năng khiếu TDTT 21.00 21.00 21.50 THPT Ten Lơ Man 26.25 26.75 27.50 THPT Lương Thế Vinh Q1 32.25 32.25 32.25 THPT Giồng Ông Tố 21.25 21.50 22.50 THPT Thủ Thiêm 17.00 17.50 17.75 THPT Lê Quý Đôn 37.50 38.00 38.00 THPT Nguyễn Thị Minh Khai 40.50 41.50 42.50 THPT Lê Thị Hồng Gấm 29.75 30.50 30.50 THPT Marie Curie 32.25 32.25 32.50 THPT Nguyễn Thị Diệu 29.50 29.50 29.50 THPT Nguyễn Trãi 32.00 32.25 32.25 THPT Nguyễn Hữu Thọ 24.50 25.00 25.25 THPT Hùng Vương 36.50 37.00 37.00 TH Thực Hành ĐHSP 38.50 38.75 39.00 THPT Trần Khai Nguyên 31.25 31.25 31.25 THPT Trần Hữu Trang 25.75 26.25 26.50
  • 6. 5 THPT Mạc Đĩnh Chi 35.75 35.75 35.75 THPT Bình Phú 33.00 33.00 33.00 THPT Lê Thánh Tôn 25.50 25.75 26.00 THPT Ngô Quyền 24.00 24.50 24.75 THPT Tân Phong 20.00 20.50 20.50 THPT Nam Sài Gòn 18.50 19.50 20.50 THPT Lương Văn Can 28.25 28.50 29.50 THPT Ngô Gia Tự 17.50 18.50 19.25 THPT Tạ Quang Bửu 25.75 26.00 26.00 THPT Chuyên Năng khiếu TDTT Nguyễn Thị Định 18.75 19.75 20.25 THPT Nguyễn Khuyến 35.00 35.00 35.00 THPT Nguyễn Du 36.00 36.00 36.00 THPT Nguyễn An Ninh 23.00 24.00 24.25 THPT Diên Hồng 21.25 22.25 23.00 THPT Sương Nguyệt Anh 23.75 24.50 24.50 THPT Nguyễn Hiền 34.00 34.75 35.75 THPT Trần Quang Khải Q11 30.00 30.00 30.00 THPT Nam Kỳ Khởi Nghĩa 26.00 26.00 26.00 THPT Võ Trường Toản 30.50 30.75 30.75 THPT Trường Chinh 28.25 28.50 28.50 THPT Thạnh Lộc 22.50 22.50 22.50 THPT Thanh Đa 26.50 26.75 27.00
  • 7. 6 THPT Võ Thị Sáu 35.75 36.00 36.00 THPT Gia Định 37.75 37.75 37.75 THPT Phan Đăng Lưu 30.50 30.75 30.75 THPT Hoàng Hoa Thám 31.00 31.50 31.50 THPT Gò Vấp 33.25 34.00 34.75 THPT Nguyễn Công Trứ 38.00 39.00 39.00 THPT Trần Hưng Đạo 33.50 33.50 33.50 THPT Nguyễn Trung Trực 27.50 27.50 28.25 THPT Phú Nhuận 38.50 38.50 39.00 THPT Hàn Thuyên 21.00 21.50 22.00 THPT Nguyễn Chí Thanh 33.50 34.50 34.75 THPT Nguyễn Thượng Hiền 41.75 42.75 43.75 THPT Lý Tự Trọng 25.00 25.50 26.25 THPT Nguyễn Thái Bình 29.75 30.00 30.00 THPT Nguyễn Hữu Huân 36.00 36.00 36.00 THPT Thủ Đức 32.50 33.50 33.50 THPT Tam Phú 29.75 30.00 30.00 THPT Hiệp Bình 24.75 25.25 25.25 THPT Long Thới 13.00 13.00 13.00 THPT Tân Bình 32.00 32.00 32.00 THPT Trần Phú 36.75 37.25 37.50 THPT Tây Thạnh 27.25 27.25 27.50
  • 8. 7 THPT Vĩnh Lộc 23.25 24.00 24.50 THPT An Lạc 26.25 26.50 26.50 2. Điểm chuẩn năm học 2010-2011 Teân Tröôøng NV 1 NV 2 NV 3 THPT Tröng Vöông 36.25 37.25 38.25 THPT Buøi Thò Xuaân 40.25 41.25 42.25 THPT Naêng Khieáu TDTT 15.00 15.00 15.00 THPT Ten Lô Man 26.50 27.00 27.50 THPT Löông Theá Vinh 32.75 33.50 34.50 THPT Leâ Quyù Ñoân 37.50 38.50 38.50 THPT Nguyeãn Thò Minh Khai 40.00 41.00 42.00 THPT Leâ Thò Hoàng Gaám 28.75 29.75 30.00 THPT Marie Curie 31.00 31.75 32.00 THPT Nguyeãn Thò Dieäu 27.75 28.50 28.75 THPT Nguyeãn Traõi 32.25 33.00 33.00 THPT Nguyeãn Höõu Thoï 22.50 22.75 23.75 THPT Huøng Vöông 35.50 36.50 36.50 TH Thöïc Haønh ÑHSP 40.50 41.00 42.00 TH Thöïc Haønh Saøi Goøn 34.75 35.25 35.50 THPT Traàn Khai Nguyeân 32.00 32.25 33.00
  • 9. 8 THPT Traàn Höõu Trang 25.75 26.00 27.00 THPT Maïc Ñónh Chi 36.25 37.25 38.25 THPT Bình Phuù 32.50 33.25 33.75 THPT Leâ Thaùnh Toân 26.75 27.25 27.25 THPT Ngoâ Quyeàn 25.50 26.50 26.50 THPT Taân Phong 20.00 20.25 20.75 THPT Nam Saøi Goøn 24.00 24.75 25.25 THPT Löông Vaên Can 26.50 27.00 27.00 THPT Ngoâ Gia Töï 19.00 19.50 20.50 THPT Taï Quang Böûu 23.50 24.25 25.00 THPT Phöôøng 7 15.00 15.00 15.50 THPT Chuyeân TDTT Ng.Thò Ñònh 25.00 25.50 25.75 THPT Nguyeãn Khuyeán 34.25 34.50 34.50 THPT Nguyeãn Du 36.25 36.50 36.50 THPT Nguyeãn An Ninh 25.75 26.75 27.00 THPT Dieân Hoàng 26.00 26.25 26.25 THPT Söông Nguyeät Anh 24.00 24.50 25.50 THPT Nguyeãn Hieàn 33.75 34.25 34.25 THPT Traàn Quang Khaûi 31.00 31.75 31.75 THPT Nam Kyø Khôûi Nghóa 29.25 29.75 29.75 THPT Voõ Tröôøng Toaûn 32.25 32.50 32.50
  • 10. 9 THPT Tröôøng Chinh 28.50 29.50 30.00 THPT Thaïnh Loäc 25.00 25.50 25.75 THPT Thanh Ña 25.75 26.00 27.00 THPT Voõ Thò Saùu 34.50 35.50 35.50 THPT Gia Ñònh 37.75 38.50 39.50 THPT Phan Ñaêng Löu 29.75 30.50 30.75 THPT Hoaøng Hoa Thaùm 31.00 31.50 31.50 THPT Goø Vaáp 32.50 32.75 32.75 THPT Nguyeãn Coâng Tröù 36.75 36.75 36.75 THPT Traàn Höng Ñaïo 33.00 33.50 33.50 THPT Nguyeãn Trung Tröïc 26.50 27.00 27.00 THPT Phuù Nhuaän 38.75 39.25 40.25 THPT Haøn Thuyeân 23.50 24.25 24.50 THPT Nguyeãn Chí Thanh 33.50 34.50 34.50 THPT Nguyeãn Thöôïng Hieàn 41.75 42.75 43.75 THPT Lyù Töï Troïng 26.00 26.50 27.50 THPT Nguyeãn Thaùi Bình 29.50 30.00 30.00 THPT Long Thôùi 15.00 15.00 15.00 THPT Phöôùc Kieån 15.00 15.00 15.00 THPT Taân Bình 31.50 31.75 31.75 THPT Traàn Phuù 37.00 38.00 38.00
  • 11. 10 THPT Taây Thaïnh 27.50 28.50 28.75 THPT Vónh Loäc 23.25 23.75 24.50 THPT Bình Trò Ñoâng A 21.50 22.00 22.00 THPT An Laïc 28.00 28.50 29.00 3. Điểm chuẩn năm học 2011-2012 Tên Trường Q/H NV 1 NV 2 NV 3 THPT Trưng Vương 01 33.50 34.50 35.50 THPT Bùi Thị Xuân 01 37.00 38.00 39.00 THPT Năng Khiếu TDTT 01 14.00 14.50 15.00 THPT Ten Lơ Man 01 24.75 25.50 26.50 THPT Lương Thế Vinh 01 31.25 31.50 32.00 THPT Lê Quý Đôn 03 35.00 35.25 35.25 THPT Nguyễn Thị Minh Khai 03 37.25 37.25 38.25 THPT Lê Thị Hồng Gấm 03 25.50 26.00 26.50 THPT Marie Curie 03 29.25 30.25 31.00 THPT Nguyễn Thị Diệu 03 26.25 26.50 27.50 THPT Nguyễn Trãi 04 28.50 29.25 30.00 THPT Nguyễn Hữu Thọ 04 20.00 20.50 21.25 THPT Hùng Vương 05 31.75 32.75 33.75 TH Thực Hành ĐHSP 05 37.25 38.25 39.25 TH Thực Hành Sài Gòn 05 33.00 34.00 34.00
  • 12. 11 THPT Trần Khai Nguyên 05 30.00 30.25 31.00 THPT Trần Hữu Trang 05 24.25 25.25 26.25 THPT Lê Thánh Tôn 07 22.50 23.00 23.00 THPT Ngô Quyền 07 24.50 24.50 25.00 THPT Tân Phong 07 18.25 19.00 19.75 THPT Nam Sài Gòn 07 23.75 24.50 25.00 THPT Lương Văn Can 08 25.00 25.50 26.00 THPT Ngô Gia Tự 08 19.00 20.00 21.00 THPT Tạ Quang Bửu 08 23.00 24.00 24.50 THPT Nguyễn Văn Linh 08 15.50 16.25 17.25 THPT chuyên NKTDTT Nguyễn Thị Định 08 13.00 13.00 13.00 THPT Nguyễn Khuyến 10 32.50 33.25 33.25 THPT Nguyễn Du 10 35.50 36.25 37.25 THPT Nguyễn An Ninh 10 23.75 24.75 25.25 THPT Diên Hồng 10 23.50 24.00 25.00 THPT Sương Nguyệt Anh 10 22.75 23.75 24.50 THPT Nguyễn Hiền 11 32.00 32.75 32.75 THPT Trần Quang Khải 11 27.75 28.75 28.75 THPT Nam Kỳ Khởi Nghĩa 11 25.75 26.75 26.75 THPT Võ Trường Toản 12 28.75 29.75 30.75 THPT Trường Chinh 12 26.75 27.50 28.00 THPT Thạnh Lộc 12 24.25 24.50 24.50
  • 13. 12 THPT Thanh Đa Bình Thạnh 23.50 24.50 25.50 THPT Võ Thị Sáu Bình Thạnh 31.75 32.50 32.50 THPT Gia Định Bình Thạnh 35.00 35.25 35.75 THPT Phan Đăng Lưu Bình Thạnh 26.50 27.25 27.50 THPT Hoàng Hoa Thám Bình Thạnh 28.00 29.00 29.50 THPT Gò Vấp Gò Vấp 29.25 30.00 30.00 THPT Nguyễn Công Trứ Gò Vấp 35.25 36.25 37.25 THPT Trần Hưng Đạo Gò Vấp 30.75 31.50 32.25 THPT Nguyễn Trung Trực Gò Vấp 24.25 25.25 26.00 THPT Phú Nhuận Phú Nhuận 35.75 36.00 37.00 THPT Hàn Thuyên Phú Nhuận 23.25 24.25 24.75 THPT Nguyễn Chí Thanh Tân Bình 32.00 32.25 32.25 THPT Nguyễn Thượng Hiền Tân Bình 39.00 40.00 41.00 THPT Lý Tự Trọng Tân Bình 25.25 25.75 26.50 THPT Nguyễn Thái Bình Tân Bình 27.25 28.00 28.00 THPT Long Thới Nhà Bè 13.00 13.00 13.00 THPT Phước Kiển Nhà Bè 13.00 13.00 13.00 THPT Tân Bình Tân Phú 29.75 30.50 30.50 THPT Trần Phú Tân Phú 35.25 35.25 36.25 THPT Tây Thạnh Tân Phú 26.75 27.50 28.50
  • 14. 13 4. Bí quyết đạt "điểm rơi phong độ" ngay trong ngày thi TTO - Làm thế nào để đạt kết quả tốt nhất có thể tương ứng với khả năng của mình khi tham gia những kỳ thi quan trọng? Để làm được điều đó, các bạn cần quan tâm đến một khái niệm được gọi là “điểm rơi phong độ”. Phong độ học tập của các bạn sẽ có lúc lên “cao chót vót” (những lúc tập trung cao độ hoặc những lúc các bạn dành nhiều thời gian đầu tư học tập) nhưng cũng có lúc phong độ “rơi tự do” (vì bận xem những bộ phim hay, những chuyến đi chơi...). "Điểm rơi phong độ" là thời điểm “phong độ” của mình đạt ở đỉnh cao nhất, điều đó có nghĩa là các bạn cần “canh” làm sao mà ngay ngày thi, “phong độ” học tập của các bạn đạt ở mức “thượng thừa”. Chắc hẳn các bạn tự hỏi làm sao để phong độ đạt ở đỉnh điểm cao nhất khi kỳ thi diễn ra. Các bạn cần quan tâm các yếu tố sau: Tâm lý phải vững vàng Trong kết quả khảo sát bỏ túi gần đây do Trung tâm đào tạo kỹ năng sống Ý Tưởng Việt thực hiện với hơn 2.000 học sinh lớp 12 tại TP.HCM với câu hỏi: “Hai tuần trước kỳ thi đại học, bạn làm gì?” thì có hơn 90% học sinh trả lời: “Em sẽ nghỉ ngơi, thư giãn”. Nhưng khi khảo sát thực tế lại có hơn 85% trong số các bạn đó “cắm đầu cắm cổ” học ngày học đêm đến nỗi mắt thâm quầng, mặt mày xanh như... tàu là chuối và không còn chút tâm lý nào để bước vào kỳ thi. Hai yếu tố chính ảnh hưởng đến tâm lý thí sinh trước kỳ thi. Thứ nhất: áp lực từ gia đình, bắt buộc thí sinh phải thi đậu để nở mày nở mặt với gia đình, với bà con chòm xóm. Thứ hai: áp lực do chính thí sinh tạo ra vì ganh đua bạn bè, với suy nghĩ: "Bạn mình đậu mà mình không đậu thì quê lắm". Chính những áp lực này khiến thí sinh như mang trên vai mình một tảng đá lớn và mang tâm lý nặng nề vào phòng thi, khiến chất lượng bài thi thấp hơn rất nhiều so với khả năng thực tế. Bí quyết có tâm lý thoải mái trước ngày "xung trận" Thứ nhất: tư duy tích cực. Thay vì lo sợ: "Chết rồi, mai là ngày thi, phải làm sao đây?” thì hãy nghĩ: “Mình rất tự tin chào đón kỳ thi! Vũ khí của mình là kiến thức. Đề có thế nào thì mình cũng có thể đối phó được”. Thứ hai: thí sinh hãy dẹp bỏ tư tưởng: "Đã thi là phải đậu, không đậu thì mình sẽ... chết". Thay vào đó, hãy suy nghĩ sẽ cố gắng hết khả năng. Như vậy, tâm lý bạn sẽ nhẹ nhàng và thoải mái hơn khi đối mặt với kỳ thi. Những bí quyết nho nhỏ cho sức khỏe ngày thi Thứ nhất: để đảm bảo dạ dày và hệ tiêu hóa hoạt động tốt, ba ngày trước ngày thi thí sinh cần chọn ăn đúng một món ăn sáng bạn thích nhất như: cơm chiên, phở, hủ tiếu… nhưng chỉ nên ăn đúng một loại thức ăn và ở đúng một quán để cơ thể thích nghi tốt nhất. Vào ngày thi, nhiều thí sinh ra quán ăn gần điểm thi ăn đại món nào đó lót bụng, khiến không ít bạn vào phòng thi mà mặt mày nhăn nhó, tay ôm bụng vì cơn đau quằn quại.
  • 15. 14 Thứ hai: thí sinh cần tập thói quen dậy sớm trước ngày thi ít nhất ba ngày để đồng hồ sinh học tự điều chỉnh thích nghi với thời gian đó. Có những trường hợp các bạn thí sinh đành ngậm ngùi tiếc nuối chỉ vì đến hội đồng thi trễ 15 phút. Các bí quyết hữu ích trong phòng thi Khi đi thi, ngoài những vật dụng cần thiết phục vụ việc làm bài thì thí sinh còn cần mang theo những vật gì? Thứ nhất: thời gian làm bài thi khá dài dễ gây mất năng lượng, chính vì vậy việc mang theo một thanh chocolate để bổ sung năng lượng có thể là ý tưởng nghe khá lạ nhưng rất thực tế. Chocolate có nhiều chất đường, cung cấp năng lượng cấp thời cho não, bên cạnh đó còn có chất cafein giúp bạn giữ tỉnh táo. Xem xét kỹ quy chế thi hiện nay thì không có khoản nào cấm thí sinh mang kẹo vào phòng thi. Thứ hai: thí sinh cần mang theo đủ nước để uống. Theo các chuyên gia tâm lý, rất có thể nước làm tăng tốc độ truyền thông tin giữa các tế bào não. Một khả năng khác là những người uống đủ nước sẽ không bị phân tán tư tưởng bởi cảm giác khát nước mà nhờ đó có thể tập trung giải quyết bài thi. Nhiều nghiên cứu cũng chứng minh rằng não người trưởng thành hoạt động hiệu quả hơn sau khi họ uống nước đầy đủ. Thứ ba: khi nào nên xin tiếp giấy thi? Hầu hết thí sinh trả lời đó là khi viết hết tờ giấy thi hiện có. Nếu làm vậy thì dòng suy nghĩ sẽ bị cắt đứt trong khi chờ giám thị ký tên rồi mang giấy thi đến. Thay vào đó, khi viết đến trang thứ 4 của tờ giấy thi, bạn hãy nhanh chóng giơ tay xin tờ khác để có thể sử dụng ngay khi vừa viết xong và dòng suy nghĩ không bị ngắt quãng. Thứ tư: bạn nhớ mang theo đồng hồ để luôn kiểm soát sát sao lượng thời gian đã trôi qua, đồng thời tránh quay qua hỏi thí sinh khác hay hỏi giám thị. Một ít khăn giấy cũng sẽ rất cần thiết nếu bạn ra mồ hôi nhiều hay nhằm lúc trời mưa, làm đổ nước khi uống... Chuyên viên tâm lý ĐÀO LÊ HÒA AN (theo www.tuoitre.vn) Moïi yù kieán ñoùng goùp xin vui loøng lieân heä ban bieân taäp: DÑ: 098 9829820 – 0953 474 474 Email: chuyenluyenthi@yahoo.com
  • 16. 15 Đề thi tuyển sinh lớp 10 thành phố Hồ Chí Minh 5. Đề thi tuyển sinh lớp 10 tp Hồ Chí Minh Năm học 2011-2012 Bài 1: (2 điểm) Giải các phương trình và hệ phương trình sau: a) 2 3 2 1 0 x x − − = b) 5 7 3 5 4 8 x y x y + =   − = −  c) 4 2 5 36 0 x x + − = d) 2 3 5 3 3 0 x x + + − = Bài 2: (1,5 điểm) a) Vẽ đồ thị (P) của hàm số 2 y x = − và đường thẳng (D): 2 3 y x = − − trên cùng một hệ trục toạ độ. b) Tìm toạ độ các giao điểm của (P) và (D) ở câu trên bằng phép tính. Bài 3: (1,5 điểm) Thu gọn các biểu thức sau: 3 3 4 3 4 2 3 1 5 2 3 A − + = − + − 2 28 4 8 3 4 1 4 x x x x x B x x x x − + − + = − + − − + − ( 0, 16) x x ≥ ≠ Bài 4: (1,5 điểm) Cho phương trình 2 2 4 5 0 x mx m − − − = (x là ẩn số) a) Chứng minh rằng phương trình luôn luôn có nghiệm với mọi m. b) Gọi x1, x2 là các nghiệm của phương trình. Tìm m để A = 2 2 1 2 1 2 x x x x + − đạt giá trị nhỏ nhất. Bài 5: (3,5 điểm) Cho đường tròn (O) có tâm O, đường kính BC. Lấy một điểm A trên đường tròn (O) sao cho AB > AC. Từ A, vẽ AH vuông góc với BC (H thuộc BC). Từ H, vẽ HE vuông góc với AB và HF vuông góc với AC (E thuộc AB, F thuộc AC). a) Chứng minh rằng AEHF là hình chữ nhật và OA vuông góc với EF. b) Đường thẳng EF cắt đường tròn (O) tại P và Q (E nằm giữa P và F). Chứng minh AP2 = AE.AB. Suy ra APH là tam giác cân. c) Gọi D là giao điểm của PQ và BC; K là giao điểm cùa AD và đường tròn (O) (K khác A). Chứng minh AEFK là một tứ giác nội tiếp. Gọi I là giao điểm của KF và BC. Chứng minh IH2 = IC.ID. 6. Đề thi tuyển sinh lớp 10 tp Hồ Chí Minh Năm học 2010-2011 Bài 1: (2 điểm) Giải các phương trình và hệ phương trình sau: a) 2 2 3 2 0 x x − − = b) 4 1 6 2 9 x y x y + = −   − =  c) 4 2 4 13 3 0 x x − + = d) 2 2 2 2 1 0 x x − − = Bài 2: (1,5 điểm) a) Vẽ đồ thị (P) của hàm số 2 2 x y = − và đường thẳng (D): 1 1 2 y x = − trên cùng một hệ trục toạ độ. b) Tìm toạ độ các giao điểm của (P) và (D) bằng phép tính.
  • 17. 16 Bài 3: (1,5 điểm) Thu gọn các biểu thức sau: 12 6 3 21 12 3 A = − + − 2 2 5 3 5 2 3 3 5 2 3 3 5 2 2 B     = + + − − + − + + −             Bài 4: (1,5 điểm) Cho phương trình 2 2 (3 1) 2 1 0 x m x m m − + + + − = (x là ẩn số) a) Chứng minh rằng phương trình luôn luôn có 2 nghiệm phân biệt với mọi giá trị của m. b) Gọi x1, x2 là các nghiệm của phương trình. Tìm m để biểu thức sau đạt giá trị lớn nhất: A = 2 2 1 2 1 2 3 x x x x + − . Bài 5: (3,5 điểm) Cho đường tròn tâm O đường kính AB=2R. Gọi M là một điểm bất kỳ thuộc đường tròn (O) khác Avà B.Các tiếp tuyến của (O) tại A và M cắt nhau tại E. Vẽ MP vuông góc với AB (P thuộc AB), vẽ MQ vuông góc với AE (Q thuộc AE). a) Chứng minh rằng AEMO là tứ giác nội tiếp đường tròn và APMQ là hình chữ nhật. b) Gọi I là trung điểm của PQ. Chứng minh O, I, E thẳng hàng. c) Gọi K là giao điểm của EB và MP. Chứng minh hai tam giác EAO và MPB đồng dạng. Suy ra K là trung điểm của MP. d) Đặt AP = x. Tính MP theo R và x. Tìm vị trí của M trên (O) để hình chữ nhật APMQ có diện tích lớn nhất. 7. Đề thi tuyển sinh lớp 10 tp Hồ Chí Minh Năm học 2009-2010 Câu 1: Giải các phương trình và hệ phương trình sau: a) 2 8 2 1 0 x x − − = ; b) 2 3 3 5 6 12 x y x y + =   − =  ; c) 4 2 2 3 0 x x − − = ; d) 2 3 2 6 2 0 x x − + = . Câu 2: a) Vẽ đồ thị (P) của hàm số 2 2 x y = và đường thẳng (D): y = x + 4 trên cùng một hệ trục toạ độ. b) Tìm toạ độ giao điểm của (P) và (D) bằng phép tính. Câu 3: Thu gọn biểu thức sau: 4 8 15 3 5 1 5 5 A = − + + + : 1 1 1 x y x y x xy B xy xy xy   + −   + = +       − − +     Câu 4: Cho phương trình ( ) 2 2 5 1 6 2 0 x m x m m − − + − = (m là tham số) a) Chứng minh phương trình luôn có nghiệm với mọi m;
  • 18. 17 b) Gọi 1 2 ; x x là nghiệm của phương trình. Tìm m để 2 2 1 2 1 x x + = . Câu 5: Cho tam giác ABC (AB < AC) có ba góc nhọn nội tiếp đường tròn (O) có tâm O, bán kính R. Gọi H là giao điểm của ba đường cao AD, BE, CF của tam giác ABC. Gọi S là diện tích tam giác ABC. a) Chứng minh rằng AEHF và AEDB là các tứ giác nội tiếp đường tròn. b) Vẽ đường kính AK của đường tròn (O). Chứng minh tam giác ABD và tam giác AKC đồng dạng với nhau. Suy ra AB.AC = 2R.AD và S = AB.BC.CA 4R . c) Gọi M là trung điểm của BC. Chứng minh EFDM là tứ giác nội tiếp đường tròn. d) Chứng minh rằng OC vuông góc với DE và (DE + EF + FD).R = 2S. 8. Đề thi tuyển sinh lớp 10 tp Hồ Chí Minh Năm học 2008-2009 Bài 1: (2 điểm) Giải các phương trình và hệ phương trình sau: a) 2 2 3 5 0 x x + − = b) 4 2 3 4 0 x x − − = c) 2 1 3 4 1 x y x y + =   + = −  Bài 2: (2 điểm) a) Vẽ đồ thị (P) của hàm số 2 y x = − và đường thẳng (D): 2 y x = − trên cùng một hệ trục. b) Tìm tọa độ các giao điểm của (P) và (D) ở câu trên bằng phép tính. Bài 3: (1 điểm) Thu gọn các biểu thức sau: a) 7 4 3 7 4 3 A = − + + b) 1 1 2 4 8 . 4 4 4 x x x x x x B x x x x   + − + − − = −     − + +   với 0; 4 x x > ≠ Bài 4: (1, 5 điểm) Cho phương trình 2 2 1 0 x mx − − = a) Chứng minh phương trình trên luôn có 2 nghiệm phân biệt. b) Gọi 1 2 , x x là hai nghiệm của phương trình trên. Tìm m để 2 2 1 2 1 2 7 x x x x + − = . Bài 5: (3,5 điểm) Từ điểm M bên ngoài đường tròn (O) vẽ cát tuyến MCD không đi qua tâm O và hai tiếp tuyến MA, MB đến đường tròn (O), A, B là các tiếp điểm và C nằm giữa M, D. a) Chứng minh 2 . MA MC MD = b) Gọi I là trung điểm của CD. Chứng minh rằng 5 điểm M, A, O, I, B cùng nằm trên một đường tròn.
  • 19. 18 c) Gọi H là giao điểm của AB và MO. Chứng minh tứ giác CHOD nội tiếp được đường tròn. Suy ra AB là đường phân giác của góc CHD. d) Gọi K là giao điểm của các tiếp tuyến tại C và D của đường tròn (O). Chứng minh A, B, K thẳng hàng. 9. Đề thi tuyển sinh lớp 10 tp Hồ Chí Minh Năm học 2007-2008 Bài 1.(1.5 điểm) Giải các phương trình và hệ phương trình sau: a) 2 2 5 4 0 x x − + = b) 4 2 29 100 0 x x − + = c) 5 6 17 9 7 x y x y + =   − =  Bài 2.(1.5 điểm) Thu gọn các biểu thức sau a) 4 2 3 6 2 A − = − b) ( ) 3 2 6 6 3 3 B = + − Bài 3.(1 điểm) Một khu vườn hình chữ nhật có diện tích bằng 675m2 và có chu vi bằng 120m. Tìm chiều dài và chiều rộng của khu vườn đó. Bài 4.(2 điểm) Cho phương trình 2 2 2 1 0 x mx m m − + − + = với m là tham số và x là ẩn số. a)Giải phương trình với m = 1. b)Tìm m để phương trình có hai nghiệm phân biệt 1 2 , x x . c)Với điều kiện của câu b) hãy tìm m để biểu thức 1 2 1 2 A x x x x = − − đạt giá trị nhỏ nhất. Bài 5.(4 điểm) Cho tam giác ABC có ba góc nhọn (AB < AC). Đường tròn đường kính BC cắt AB, AC theo thứ tự tại E và F. Biết EF cắt CE tại H và AH cắt BC tại D. a)Chứng minh tứ giác BEFC nội tiếp và AH vuông góc với BC. b)Chứng minh AE.AB = AF.AC c)Gọi O là tâm đường tròn ngoại tiếp tam giác ABC và K là trung điểm của BC. Tính tỉ số OK BC khi tứ giác BHOC nội tiếp. d) Cho HF = 3cm, HB = 4cm, CE = 8cm và HC > HE. Tính HC. 10. Đề thi tuyển sinh lớp 10 tp Hồ Chí Minh Năm học 2006-2007 Bài 1. Giải phương trình và hệ phương trình sau:
  • 20. 19 a) 3 2 1 5 3 4 x y x y + =   + = −  b) 2 2 2 3 3 0 x x + − = c) 4 2 9 8 1 0 x x + − = Bài 2. Thu gọn biểu thức 15 12 1 5 2 2 3 A − = − − − ; 2 2 4 . 2 2 a a B a a a a   − +   = − −       + −     , với 0; 4 a a > ≠ . Bài 3. Cho mãnh đất hình chữ nhật có diện tích 360m2 . Nếu tăng chiều rộng 2m và giảm chiều dài đi 6m thì diện tích mãnh đất không đổi. Tính chu vi mãnh đất lúc ban đầu. Bài 4. a)Viết phương trình đường thẳng (d) song song với đường thẳng y = 3x + 1 và cắt trục tung tại điểm có tung độ bằng 4. b) Vẽ đồ thị hàm số y = 3x + 4 và 2 2 x y = − trên cùng một hệ trục toạ độ. Tìm toạ độ giao điểm của hai đồ thị ấy bằng phép tính. Bài 5. Cho tam giác ABC có ba góc nhọn và AB < AC. Đường tròn tâm O đường kính BC cắt cạnh AB, AC theo thứ tự tại E và D. a)Chứng minh rằng AD. AC = AE. AB. b)Gọi H là giao điểm của BD và CE, gọi K là giao điểm của AH và BC. Chứng minh rằng AH vuông góc với BC. c)Từ A kẻ tiếp tuyến AM, AN đến (O) với M, N là các tiếp điểm. Chứng minh ANM AKN = . d)Chứng minh rằng M, H, N thẳng hàng. II. Đề thi tuyển sinh lớp 10 Chuyên thành phố Hồ Chí Minh 1. Chuyên tp Hồ Chí Minh, năm học 2011-2012(N/A) 2. Chuyên tp Hồ Chí Minh, năm học 2010-2011(N/A) 3. Chuyên tp Hồ Chí Minh, năm học 2009-2010 Bài 1:(4 điểm) 1)Giải hệ phương trình 2 2 1 2 x y xy x y xy − − = −   − =  .
  • 21. 20 2)Cho phương trình 2 2 2 16 5 0 x mx m − − + = ( x là ẩn số). a)Tìm m để phương trình có nghiệm. b)Gọi 1 2 , x x là các nghiệm của phương trình. Tìm giá trị lớn nhất và giá trị nhỏ nhất của biểu thức ( ) ( ) 1 1 2 2 2 1 5 3 17 5 3 17 A x x x x x x = + − + + − . Bài 2:(4 điểm) 1)Thu gọn biểu thức 45 27 2 45 27 2 3 2 3 2 5 3 2 5 3 2 3 2 3 2 A + + − + + − = − + − − + − − 2)Cho , , x y z là ba số dương thỏa điều kiện 2 xyz = . Tính giá trị biểu thức 2 2 1 2 2 x y x B xy x yz y zx z = + + + + + + + + Câu 3:(2 điểm) 1) Cho ba số thực , , a b c . Chứng minh rằng ( ) ( ) ( ) 2 2 2 2 2 2 26 6 2009 a b b c c a a b c ab bc ca − − − + + ≥ + + + − + 2)Cho 0 a và 0 b . Chứng minh 1 2 8 2 a b a b ≥ + − . Câu 4:(2 điểm) 1)Cho hệ phương trình 5 5 ax by bx ay + =   + =  (a,b là nguyên dương và a khác b). Tìm a, b để hệ có nghiệm (x; y) với x, y là các số nguyên dương. 2)Chứng minh rằng không tồn tại các số nguyên x, y, z thỏa hệ: 2 2 2 2 2 3 3 31 8 100 x xy y z x xy z  − + − =   + + =   . Câu 5:(3 điểm) Cho ABCD là hình thoi có cạnh bằng 1. Giả sử tồn tại điểm M thuộc cạnh BC và N thuộc cạnh CD sao cho tam giác CMN có chu vi bằng 2 và 2 BAD MAN = . Tính các góc của hình thoi ABCD. Câu 7:(2 điểm) Cho a, b là các số dương thỏa 2 1 1 1 a b a b + = + + . Chứng minh 2 1 8 ab ≤ .
  • 22. 21 4. Chuyên tp Hồ Chí Minh, năm học 2008-2009(N/A) 5. Chuyên tp Hồ Chí Minh, năm học 2007-2008 Câu 1:(4 điểm) a)Chứng minh với mọi số thực x, y, z, t ta luôn có bất đẳng thức sau: ( ) 2 2 2 2 x y z t x y z t + + + ≥ + + . Đẳng thức xảy ra khi nào? b)Chứng minh với mọi số thực a, b khác không ta luôn có bất đẳng thức sau: 2 2 2 2 4 3 a b a b b a b a   + + ≥ +     Câu 2:(2 điểm) Tìm nghiệm nguyên của phương trình sau: 2 6 5 8 x xy x y − = − − Câu 3:(4 điểm) Cho hệ phương trình ( )( ) 2 2 2 2 11 2 2 x y x y xy x y m  + + + =   + + =   a)Giải hệ phương trình khi 24 m = . b)Tìm m để hệ phương trình có nghiệm. Câu 4: (2điểm) Cho ( )( ) 2 2 2007 2007 2007 x x y y + + + + = . Tính S x y = + . Câu 5:(2 điểm) Cho a, b là các số nguyên dương sao cho 1 1 a b b b + + + cũng là số nguyên. Gọi d là ước số chung của a và b. Chứng minh d a b ≤ + . Câu 6:(6 điểm) Cho tam giác ABC có ba góc nhọn nội tiếp trong đường tròn (O) (AB AC). Các tiếp tuyến với (O) tại B và C cắt nhau tại N. Vẽ dây AM song song với BC. Đường thẳng MN cắt đường tròn (O) tại M và P. a)Cho biết 2 2 1 1 1 16 OB NC + = , tính độ dài đoạn BC. b)Chứng minh BP CP AC AB = . c)Chứng minh BC, ON và AP đồng qui tại một điểm. 6. Chuyên tp Hồ Chí Minh, năm học 2006-2007 Câu 1:(2 điểm) Thu gọn các biểu thức sau:
  • 23. 22 ( ) 2 4 6 2 5 10 2 A   = + − −     2 1 1 2 1 , 0, 1 1 1 1 a a B a a a a a   − +   = + − ≠      + + −     Câu 2:(1 điểm) Với giá trị nào của m thì đường thẳng ( ) 3 : 2 2 d y x m = + cắt Parabol ( ) 2 3 : 4 P y x = − tại hai điểm phân biệt? Câu 3:(1,5 điểm) Giải các phương trình và hệ phương trình: a) 2 5 1 x x − = − b) 3 4 2 4 5 3 x y x y  − =     − =   c) 2 4 2 2 8 5 2 3 x x x x − + − + − + − = + Câu 5:(4 điểm) a)Cho hai số dương x, y thoả 3 x y xy + = . Tính x y . b)Tìm các số nguyên dương , x y thoả 1 1 1 2 x y + = . Câu 5:(4 điểm) Cho tam giác ABC có ba góc nhọn (AB AC), có đường cao AH. Gọi D, E lần lượt là trung điểm AB và AC. a)Chứng minh DE là tiếp tuyến chung của hai đường tròn ngoại tiếp hai tam giác DBH và ECH. b)Gọi F là giao điểm thứ nhì của hai đường tròn ngoại tiếp hai tam giác DBH và ECH. Chứng minh HF đi qua trung điểm của DE. c)Chứng minh rằng đường tròn ngoại tiếp tam giác ADE đi qua điểm F. 7. Chuyên Lê Hồng Phong tp Hồ Chí Minh, năm học 2001-2002 Bài 1: Cho phương trình ( ) ( ) 2 1 2 2 3 0 m x m x m + − + + − = .
  • 24. 23 a)Định m để phương trình có nghiệm. b)Định m để phương trình có hai nghiệm 1 2 , x x thỏa mãn ( )( ) 1 2 4 1 4 1 18 x x + + = . Bài 2: Chứng minh các bất đẳng thức sau a) 2 2 2 a b c ab bc ca + + ≥ + + , với mọi , , a b c . b) 8 8 8 3 3 3 1 1 1 a b c a b c a b c + + ≥ + + , với ( ) 0, 0, 0 a b c . c) ( ) 2 2 2 2 2 a b c d e a b c d e + + + + ≥ + + + , với mọi , , , , a b c d e. Bài 3: Giải các phương trình sau a) 2 2 8 1 x x x + = − b) 2 2 4 5 1 8 7 10 7 x x x x x x + = − − + − + Bài 4: Cho tam giác ABC có ba góc nhọn nội tiếp trong đường tròn tâm O và có trực tâm H. Lấy M thuộc cung nhỏ BC. a)Xác định vị trí điểm M sao cho BHCM là một hình bình hành. b)Với M lấy bất kỳ thuộc cung nhỏ BC, gọi N, E lần lượt là các điểm đối xứng của M qua AB, AC. Chứng minh rằng N, H, E thẳng hàng. c)Xác định vị trí của M thuộc cung nhỏ BC sao cho NE có độ dài lớn nhất. Bài 5: cho đường tròn cố định tâm O, bán kính bằng 1. Tam giác ABC thay đổi luôn ngoại tiếp đường tròn (O). Một đường thẳng đi qua tâm O và cắt các cạnh AB, AC lần lượt tại M, N. Xác định giá trị nhỏ nhất của diện tích tam giác AMN. 8. Chuyên Lê Hồng Phong tp Hồ Chí Minh, năm học 2002-2003 Bài 1: Rút gọn biểu thức a) 5 3 29 12 5 A = − − − b) 8 4 4 2 3 4 2 x x B x x + + = + + Bài 2: Cho phương trình ( ) 2 2 1 2 4 0 x m x m − − + − = .
  • 25. 24 a)Chứng minh rằng phương trình có hai nghiệm phân biệt. b)Gọi 1 2 , x x là hai nghiệm của phương trình. Tìm giá trị nhỏ nhất của biểu thức 2 2 1 2 y x x = + . Bài 3: a)Chứng minh: ( ) 2 2 2 2 x y x y + + ≥ b)Chứng minh: ( ) 4 4 4 8 x y x y + + ≥ c)Cho x, y 0 và x + y =1. Chứng minh rằng ( ) 4 4 1 8 5 x y xy + + ≥ . Bài 4: Giải các phương trình sau a) 3 4 1 8 6 1 5 x x x x + + − + + − − = b) ( )( )( )( ) 4 1 12 1 3 2 1 4 x x x x + − + + = Bài 5: Cho đường tròn (O; R) và đường thẳng (d) không qua O cắt đường tròn (O) tại hai điểm A, B. Từ một điểm di động trên đường thẳng (d) và ở ngoài (O), ta vẽ hai tiếp tuyến MN, MP với đường tròn (O) (N, P là hai tiếp điểm) a)Chứng minh rằng NMO NPO = . b)Chứng minh rằng đường tròn ngoại tiếp tam giác MNP đi qua một điểm cố định khi M lưu động trên đường thẳng (d). c)Xác định vị trí điểm M trên đường thẳng (d) sao cho tứ giác MNOP là hình vuông. d)Chứng minh rằng tâm I của đường tròn nội tiếp tam giác MNP lưu động trên một đường cố định khi M lưu động trên (d). 9. Chuyên Lê Hồng Phong tp Hồ Chí Minh, năm học 2003-2004 Bài 1: Cho phương trình 2 2 6 9 0 x mx m − − − = . a)Tìm m để phương trình có hai nghiệm phân biệt đều âm. b)Gọi 1 2 , x x là hai nghiệm của phương trình. Tìm m để có 2 2 1 2 13 x x + = . Bài 2:
  • 26. 25 a)Cho 0, 0 x y và 1 x y + ≤ . Chứng minh rằng 2 2 1 1 4 x xy y xy + ≥ + + . b)Tìm giá trị nhỏ nhất của biểu thức 2 3 2 2 7 A x x = + − + . Bài 3: Giải các hệ phương trình sau a) 2 2 11 30 x y xy x y xy + + =   + =  b) 64 1 1 1 4 xy x y = −    − =   Bài 4: Chứng minh rằng nếu 2 a b + thì ít nhất một trong hai phương trình sau có nghiệm 2 2 2 0; 2 0 x ax b x bx a + + = + + = Bài 5: Cho đường tròn tâm O đường kính AB. Gọi K là trung điểm của cung AB, M là điểm lưu động trên cung nhỏ AK (M khác A và K). Lấy điểm N trên đoạn BM sao cho BM = AM. a)Chứng minh rằng AMK BNK = . b)Chứng minh rằng tam giác MNK vuông cân. c)Hai đường thẳng AM và OK cắt nhau tại D. Chứng minh MK là đường phân giác DMN . Bài 6: Cho tam giác ABC có BC = a, CA = b, AB = c và có R là bán kính đường tròn ngoại tiếp thoả mãn hệ thức ( ) R b c a bc + = . Hãy định dạng tam giác ABC. 10. Chuyên Lê Hồng Phong tp Hồ Chí Minh, năm học 2004-2005 I.Phần tự chọn: học sinh chọn một trong hai bài sau đây Bài 1a: Cho phương trình ( ) 2 3 1 2 18 0 x m x m − + + − = . a)Tìm m để phương trình có hai nghiệm phân biệt đều âm. b)Gọi 1 2 , x x là hai nghiệm của phương trình. Tìm m để có 1 2 5 x x − ≤ . Bài 1b: Rút gọn các biểu thức sau: a) 2 2 1 1 1 x x x x A x x x x x − + = − + + + + − + b) 2 2 1 1 2 1 x x x x x x B x x x x    + − + − − = −       − + +    II.Phần bắt buộc: Bài 2: Giải các phương trình sau
  • 27. 26 a) 2 3 4 2 2 x x x + − = − b) ( ) 2 2 2 9 3 9 2 x x x = + − + Bài 3: a)Cho 1, 1 x y ≥ ≥ . Chứng minh rằng 1 1 x y y x xy − + − ≤ . b) 0, 0 x y và 1 x y + = . Tìm giá trị nhỏ nhất của biểu thức 2 2 1 1 1 1 A x y     = − −        . Bài 4: Tìm các số nguyên x, y thoả hệ 2 1 0 2 1 1 0 y x x y x  − − − ≥   − + + − ≤   . Bài 5: Cho đường tròn tâm O. Từ M nằm ngoài đường tròn (O) vẽ các tiếp tuyến MC, MD với (O) (C, D là các tiếp điểm). Vẽ các tiếp tuyến MAB không đi qua tâm O, A nằm giữa M và B. Tia phân giác của góc ACB cắt AB tại E. a)Chứng minh rằng MC = ME. b)Chứng minh DE là phân giác của góc ABD. c)Gọi I là trung điểm của đoạn AB. Chứng minh 5 điểm O, I, C, M, D cùng nằm trên một đường tròn. d)Chứng minh IM là phân giác của CID . Bài 6: Cho hình thang ABCD có hai cạnh đáy là BC và AD (BC AD). Trên tia đối của tia CA lấy điểm P tùy ý. Đường thẳng qua P và trung điểm I của BC cắt AB tại M, đường thẳng qua P và trung điểm J của AD cắt CD tại N. Chứng minh MN song song với AD. 11. Chuyên Đại học Sư Phạm tp Hồ Chí Minh, năm học 2006-2007(N/A) 12. Chuyên Trần Đại Nghĩa tp Hồ Chí Minh, năm học III. Đề thi tuyển sinh lớp 10 trường Phổ Thông Năng Khiếu 1. Toán không chuyên trường Phổ Thông Năng Khiếu, năm học 2011-2012 Bài 1:(2,5 điểm) Cho phương trình ( ) ( ) 2 2 2 3 0 1 x mx m x − − − = . a)Giải phương trình (1) khi m = 2. b)Tìm m để 2 2 2 0 x mx m − − = có hai nghiệm phân biệt 1 2 , x x thoả 2 2 2 1 2 2 7 2 x x m + = + .
  • 28. 27 c)Chứng minh phương trình (1) luông có không quá hai nghiệm phân biệt. Bài 2:(2 điểm) a)Giải phương trình 2 5 2 1 6 x x x + + − = + − . b)Giải hệ phương trình 2 2 2 1 1 x y y xy x  + = +  = +  . Bài 3:(1,5 điểm) a)Rút gọn biểu thức ( ) 2 3 1 1 1 : 1 1 1 x x x x R x x x   − +   − −   = +       − − +       với 0 x ≥ và 1 x ≠ . b)Chứng minh R 1. Bài 4:(1 điểm) Một tổ mua nguyên liệu để tổ chức thuyết trình tại lớp hết 72.000 đồng, chi phí được chia đều cho mỗi thành viên của tổ. Nếu tổ giảm bớt 2 người thì mỗi người phải đóng thêm 3.000 đồng. Hỏi số người của tổ? Bài 5:(3 điểm) Cho tam giác ABC có 75o BAC = , 45o BCA = , 2 AC a = , AK vuông góc với BC (K thuộc BC). a)Tính độ dài các đoạn KC và AB theo a. b)Gọi H là trực tâm và O là tâm đường tròn ngoại tiếp tam giác ABC. Tính góc HOC . c)Đường tròn tâm I nội tiếp tam giác ABC. Tính bán kính đường tròn ngoại tiếp tam giác HIO theo a. 2. Toán không chuyên trường Phổ Thông Năng Khiếu, năm học 2010-2011 Bài 1.(2,5 điểm) a)Tìm m để phương trình 2 2 3 0 x x m + + − = có hai nghiệm phân biệt 1 2 , x x thoả ( ) 2 2 2 1 2 1 2 1 2 2 7 x x x x x x + + = b)Giải phương trình ( )( ) 2 9 4 6 9 2 5 9 4 x x x x x − − + = − + − Bài 2.(2 điểm) a)Giải hệ phương trình 2 2 2 0 1 x y xy xy xy x y  + + =  + − = 
  • 29. 28 b)Rút gọn biểu thức ( ) 1 2 1 : 1 , 1 1 1 1 a a a a a A a a a a a   + + + = − + −     − + +   . Bài 3.(1,5 điểm) Cho tam giác ABC vuông tại A, có chu vi bằng 30cm và diện tích bằng 30cm2 . Tính độ dài các cạnh của tam giác. Bài 4.(1 điểm) Cho số tự nhiên n có hai chữ số, chữ số hàng chục là x, chữ số hàng đơn vị là y (nghĩa là 0 x ≠ và 10 n x y = + . Gọi n M x y = + . a)Tìm n để M = 2. b)Tìm n để M đạt giá trị nhỏ nhất. Bài 5.(3 điểm)Cho hình chữ nhật ABCD có tâm O, cạnh AB = 3a, 30o ABD = . Gọi G là trọng tâm của tam giác AOD, AG cắt CD tại E. a)Chứng minh tứ giác AOED nội tiếp được trong một đường tròn. b)Cho DG cắt AB tại F. Tính diện tích tứ giác AFOE. c)Đường tròn tâm J nội tiếp trong tam giác BCD tiếp với DB, CD tại I và K. Gọi H là giao điểm của IK và AC. Tính IOJ và độ dài đoạn HE. 3. Toán không chuyên trường Phổ Thông Năng Khiếu, năm học 2009-2010 Bài 1.(2 điểm) a)Giải phương trình bằng cách đặt ẩn số 5 4 t x x   = −     : 2 2 400 5 35 24 4 x x x x   + = + −     b)Cho phương trình ( ) 2 3 1 2 3 0 mx m x m + + − + = . Tìm m để phương trình có hai nghiệm phân biệt 1 2 , x x thoả mãn 2 2 1 2 34 x x + = . Bài 2.(2.5 điểm) Xét biểu thức 2 3 3 4 5 1 5 4 5 x x x x R x x x x + + + − = − − + − − − . a)Rút gọn R. b)Tìm số thực x để R -2. Tìm số tự nhiên x là số chính phương sao cho R là số nguyên. Bài 3.(2 điểm)
  • 30. 29 a)Giải hệ phương trình 2 2 0 8 x xy y x y + + =   + =  b)Cho a, b, c là độ dài ba cạnh của tam giác ABC. Giả sử phương trình ( )( ) ( )( ) ( )( ) 0 x a x b x b x c x c x a − − + − − + − − = có nghiệm kép. Tính số đo các góc tam giác ABC. Bài 4.(1 điểm) cho tam giác ABC có 60o ABC = , 45o ACB = . Dựng AH vuông góc với BC (H thuộc BC), và dựng HK vuông góc với AB (K thuộc AB). Gọi M là trung điểm của AC. Biết 3 AH = , tính BC. Chứng minh rằng BKMC là tứ giác nội tiếp. Bài 5.(1 điểm) Trong kỳ kiểm tra môn Toán một lớp gồm 3 tổ A, B, C điểm trung bình của học sinh các tổ được thống kê ở bảng sau: Tổ A B C A và B B và C Điểm trung bình 9.0 8.8 7.8 8.9 8.2 Biết tổ A gồm 10 học sinh, hãy xác định số học sinh và điểm trung bình của toàn lớp. Bài 6.(1 điểm) Cho tứ giác lồi ABCD nội tiếp trong (O), có đỉnh A cố định và các đỉnh B, C, D di chuyển trên (O) sao cho 90o BAD . Kẻ tia Ax vuông góc với AD cắt BC tại E, kẻ tia Ay vuông góc với AB cắt CD tại F. Gọi K là điểm đối xứng của A qua EF. Chứng minh tứ giác EFCK nội tiếp được và đường thẳng EF luôn đi qua một điểm cố định. 4. Toán AB trường Phổ Thông Năng Khiếu, năm học 2008-2009 Bài 1: Cho phương trình ( ) 2 2 2 2 1 6 2 x mx m m x x m + − = − + + (1) a)Giải phương trình khi m = 1. b)Tìm tất cả các giá trị của m để phương trình (1) có nghiệm. Bài 2: a)Giải phương trình 2 1 2 1 1 x x − − − = − b)Giải hệ phương trình 2 2 2 2 4 2 4 x x y xy x xy  − + =   + =  
  • 31. 30 Bài 3: a)Chứng minh rằng biểu thức sau không phụ thuộc vào biến x (x 1) ( )( ) ( )( )( ) 4 3 1 1 3 x x x x x x A x x x x x x + + − = − + + + . b)Cho a, b, c là các số thực thoả mãn a + 2b -3c = 0 và bc + 2ac – 3ab =0. Chứng minh rằng a = b = c. Bài 4: Cho tứ giác ABCD nội tiếp có góc A nhọn và hai đường chéo AC và BD vuông góc với nhau tại M. P là trung điểm của CD, H là trực tâm của tam giác ABD. a)Tính tỷ số PM DH . b)Gọi N, K lần lượt là chân đường cao hạ từ B và D của tam giác ABD, Q là giao điểm của MK và BC. Chứng minh MN = MQ. c)Chứng minh tứ giác BQNK nội tiếp. Bài 5: Một nhóm học sinh định chia một số kẹo thành các phần quà cho các em nhỏ tại một đơn vị trẻ em mồ côi. Nếu mỗi phần quà giảm đi 6 viên thì các em có thêm 5 phần quà, nếu giảm đi 10 viên thì các em có thêm 10 quà. Hỏi số kẹo mà nhóm học sinh này có. 5. Toán CD trường Phổ Thông Năng Khiếu, năm học 2008-2009 Bài 1:(2 điểm) Giả sử 1 2 , x x là hai nghiệm của phương trình ( ) 2 3 2 1 0 mx m x m − + + + = . a)Tính tổng S và tích P của hai nghiệm. Biết 1 S P − = , tính 1 2 , , m x x . b)Trong trường hợp tổng quát, không tính 1 2 , x x hãy tìm một hệ thức liên hệ giữa S và P độc lập với m. Bài 2:(2 điểm) a)Giải hệ phương trình 2 2 2 2 2 3 0,19 5 7 0,83 x y x y  − = −   + =   , với 0 xy . b)Giải phương trình ( ) 2 2 1 1 x x x + − = .
  • 32. 31 Bài 3:(1 điểm) Một phân số P có dạng 3 a P a = + với a là số nguyên dương. Nếu mẫu số tăng thêm một thì phân số giảm đi 11 325 . Tính P. Bài 4:(3 điểm) Cho tam giác ABC vuông tại A có AB = 6, AC =8. a)Đường tròn nội tiếp tam giác ABC có tâm I, bán kính r. Tính IA. b)Đường tròn ngoài tiếp tam giác ABC có tâm O, bán kính R. Tính 2 2 OI R Rr − . c)Dựng AH vuông góc với BC tại H và HK vuông góc với AC tại K. Đường tròn ngoại tiếp tam giác BKC cắt AB tại E (E khác B). Tính AEH . Bài 5:(2 điểm) Một sinh viên suýt bị tai nạn tàu hoả kể chuyệ với bạn: “Khi đang chạy trên cầu sắt (sử dụng cho tàu hoả) dài 120m thì tôi thấy một chiếc xe lửa tốc hành lao tới. Thay vì chạy cùng chiều với xe lửa tôi quyết định chạy nước rút nhược chiều xe lửa. Dù là vô chạy nước rút của trường tôi, tôi cũng chỉ chạy được với vận tốc 18km/h vì đang mang ba lô trên lưng. May mắn thay, khi tôi chạy đến đầu cầu bên kia thì 2 giây sau xe lửa mới đến nên tôi đã kịp tránh sang một bên. Khi hoàn hồn lại, tôi tính được rằng nếu tôi chạy cùng chiều với xe lửa thì ngay khi đến đầu cầu bên kia thì tôi sẽ bị xe lửa đụng. May thật!” Biết xe lửa chạy với vận tốc 72km/h, em hãy tính khi thấy xe lửa, anh sinh viên đang ở cách đầu cầu bên này bao nhiêu mét? 6. Toán AB trường Phổ Thông Năng Khiếu, năm học 2007-2008 Bài 1. Cho phương trình ( ) ( ) 2 2 2 1 3 0 1 1 x x m m m x − + + − = − a)Tìm m để x = -1 là một nghiệm của phương trình (1). b)Tìm m để phương trình (1) vô nghiệm. Bài 2. a)Giải bất phương trình ( )( ) 2 3 1 2 1 7 x x x x + − − − −
  • 33. 32 b)Giải hệ phương trình 2 3 2 1 2 3 2 1 x y y x x x y x x y y y  + = −   + = −   Bài 3. a)Cho a, b là hai số thực thoả mãn điều kiện 2 2 2 2 3 2 2 5 7 0 a ab b a b a ab b a b − − + − = − + − + = . Chứng tỏ rằng 12 15 0 ab a b − + = . b)Cho ( )( )( ) ( ) 2 2 4 2 1 4 2 2 1 1 x x x x x x A x x x + − + + + + − + = − . Hãy tìm tất cả các giá trị của x để 0 A ≥ . Bài 4. Cho tam giác ABC nhọn có trực tâm H và 60o BAC = . Gọi M, N, P lần lượt là chân các đường cao kẻ từ A, B, C của tam giác ABC và I là trung điểm của BC. a)Chứng minh rằng tam giác INP đều. b)Gọi E và K lần lượt là trung điểm PB và NC. Chứng minh rằng các điểm I, M, E, K cùng thuộc một đường tròn. c)Giả sử IA là phân giác của NIP . Hãy tính số đo của góc BCP . Bài 5.Một công ty may giao cho tổ may A may 16800 sản phẩm, tổ B may 16500 sản phẩm và bắt đầu thực hiện công việc cùng một lúc. Nếu sau 6 ngày, tổ A được hỗ trợ thêm 10 công nhân may thì họ hoàn thành công việc cùng lúc với tổ B. Nếu tổ A được hỗ trợ thêm 10 công nhân ngay từ đầu thì họ sẽ hoàn thành công việc sớm hơn tổ B một ngày. Hãy xác định số công nhân ban đầu của mỗi tổ. Biết rằng mỗi công nhân mỗi ngày may được 20 sản phẩm. 7. Toán AB trường Phổ Thông Năng Khiếu, năm học 2006-2007(N/A) 8. Toán CD trường Phổ Thông Năng Khiếu, năm học 2006-2007(N/A) 9. Toán AB trường Phổ Thông Năng Khiếu, năm học 2005-2006(N/A) 10. Toán CD trường Phổ Thông Năng Khiếu, năm học 2005-2006(N/A) Bài 1(2 điểm) a)Gọi (d) là đường thẳng qua hai điểm A(0; -1) và B(1; -m-1). Tìm giá trị m để Parabol (P) 2 4 y mx mx = + − tiếp xúc với đường thẳng (d).
  • 34. 33 b)Giải sử 1 2 , x x là nghiệm cuả phương trình 2 2 3 0 mx mx + − = , tính 2 2 1 2 A x x = + theo m. Bài 2(2 điểm) a)Với điều kiện xy 0, giải hệ phương trình 2 2 2 2 3 4 0,11 2 3 0,02 x y x y  − =   − =   b)Rút gọn biểu thức 3 5 3 5 2 3 5 2 3 5 R + − = + + + − − Bài 3(2 điểm) a)Giải phương trình 2 2 15 4 4 6 9 2 x x x x x − + + + + = b)Tìm bảy số nguyên liên tiếp sao cho tổng bình phương bốn số đầu bằng tổng bình phương ba số sau. Bài 4(2 điểm) Cho tam giác ABC có 45o ACB = và 2 ACB BAC ABC + = . Trung trực của AB cắt BC tại M. a)Tính MAC . b)Gọi I là tâm đường tròn ngoại tiếp tam giác AMC. Chứng minh tứ giác ABCI nội tiếp. Bài 5 (2 điểm) Một cuộc đua thuyền được tổ chức trên tuyến đường hình tam giác đều ABC (chạy từ A đến B, từ B đến C rồi từ C về A). Chiếc thuyền “Bảy cây sứ trắng” tham dự cuộc đua và được ghi nhận các thông tin sau đây: thuyền chạy từ 2 3 đoạn đường AB cho đến đích thì mất 3 giờ 15 phút; thuyền vượt đoạn BC nhanh hơn khi vượt đoạn CA 25 phút; thuyền chạy từ A đến 1 4 đoạn CA hết 2 giờ 40 phút. Giả sử rằng khi di chuyển trên mỗi cạnh, tốc độ của thuyền là không đổi và thuyền đi rất thẳng; ngoài ra, thời gian để thuyền đổi hướng là không đáng kể (khi vượt xong một cạnh nhưng chưa tới đích, thuyền đổi hướng chuyển sang cạnh kế). Tính thời gian thuyền vượt toàn bộ đường đua. 11. Toán AB trường Phổ Thông Năng Khiếu, năm học 2004-2005 Bài 1. a)Giải phương trình 4 3 2 x x − − =
  • 35. 34 b)Định m để phương trình ( ) 2 1 2 0 x m x m − + + = có hai nghiệm phân biệt 1 2 , x x sao cho 1 2 , x x là độ dài cạnh góc vuông của một tam giác vuông có cạnh huyền bằng 5. Bài 2. Cho a, b là các số thực dương thoả mãn điều kiện ( ) ( ) ( ) 2 2 2 2 2 2 a b c a b b c c a + + = − + − + − a)Tính a b c + + biết rằng 9 ab bc ca + + = . b)Chứng minh rằng nếu , c a c b ≥ ≥ thì c a b ≥ + . Bài 3.Cùng một thời điểm, một chiếc ôtô A X xuất phát từ thành phố A hướng về thành phố B và một chiếc khác B X xuất phát từ thành phố B hướng về thành phố A. Chúng chuyển động với vận tốc riêng không đổi và gặp nhau lần đầu tại một điểm cách A là 20km. Cả hai chiếc xe, sau khi đến B và A tương ứng, lập tức quay trở lại và chúng gặp nhau lần thứ hai tại một điểm C. Biết thời gian xe B X đi từ C đến B là 10 phút và thời gian giữa hai lần gặp nhau là 1 giờ, hãy tính vận tốc của từng chiếc ôtô. Bài 4.Gọi I, O lần lượt là tâm đường tròn nội tiếp và ngoại tiếp (C) của tam giác nhọn ABC. Tia AI cắt đường tròn (C) tại K (K khác A) và J là điểm đối xứng của I qua K. Gọi P và Q lần lượt là các điểm đối xứng của I và O qua BC. a)Chứng minh rằng tam giác IBJ vuông tại B. b)Tính góc BAC nếu Q thuộc đường tròn (C). c)Chứng minh rằng nếu Q thuộc (C) thì P cũng thuộc (C). Bài 5. Chứng minh rằng từ 8 số nguyên dương tuỳ ý không lớn hơn 20, luôn luôn chọn được ba số x, y, z là độ dài ba cạnh của một tam giác. 12. Toán CD trường Phổ Thông Năng Khiếu, năm học 2004-2005 Bài 1. a)Tìm m để Parabol (P) 2 2 2 y x mx m = + − + tiếp xúc với đường thẳng (d) y x m = + . b)Giả sử phương trình ( ) 2 2 2 1 1 0 mx m x m + + + − = có hai nghiệm phân biệt 1 2 , x x . Hãy tính tổng S và tích P của hai nghiệm, tìm một hệ thức liên hệ giữa S và P độc lập với m. Bài 2.
  • 36. 35 a)Giải hệ phương trình 3 3 1 21 x y x y + = −   + = −  b)Giải phương trình 20 3 2 2 3 x x − − = − Bài 3. a)Tìm k để đa thức ( ) 4 2 22 51 2 f x x x x k = − + + chia hết cho đa thức ( ) 2 3 2 g x x x = − + (nghĩa là có đa thức ( ) h x sao cho ( ) ( ) ( ) f x h x g x = . Giải phương trình ( ) 0 f x = với k vừa tìm được. b)Rút gọn biểu thức 2 2 2 2 2 2 2 2 3 2 3 4 : 2 3 2 a ab b a ab b R a ab b a ab b − − − + = + − + − . Bài 4. Cho tam giác ABC vuông ở đỉnh A và 75o ABC = . Đường trung trực của BC cắt các đường thẳng BC, AC và AB lần lượt tại các điểm M, N và P. a)Tính AN NC . b)Gọi I là giao điểm của các đường thẳng BN và PC. So sánh MA và MI. c)Lấy điểm Q trên đường thẳng vuông góc với mặt phẳng (ABC) tại B sao cho BQ = BI, QJ vuông góc với PC, J nằm trên PC. Tính QJ AB . IV. Đề thi tuyển sinh lớp 10 trên toàn quốc 1. Tuyển sinh lớp 10 thành phố Đà Nẵng năm học 2011-2012 Bài 1: (2,0 điểm) a)Giải phương trình: (2x + 1)(3-x) + 4 = 0 b)Giải hệ phương trình: 3 | | 1 5 3 11 x y x y − =   + =  Bài 2: (1,0 điểm) Rút gọn biểu thức 6 3 5 5 2 ( ): . 2 1 5 1 5 3 Q − − = + − − − Bài 3: (2,0 điểm) Cho phương trình x2 – 2x – 2m2 = 0 (m là tham số). a) Giải phương trình khi m = 0 b) Tìm m để phương trình có hai nghiệm x1, x2 khác 0 và thỏa điều kiện 2 2 1 2 4 x x = . Bài 4: (1,5 điểm) Một hình chữ nhật có chu vi bằng 28 cm và mỗi đường chéo của nó có độ dài 10 cm. Tìm độ dài các cạnh của hình chữ nhật đó. Bài 5: (3,5 điểm) Cho tam giác đều ABC nội tiếp đường tròn đường kính AD. Gọi M là một điểm di động trên cung nhỏ AB ( M không trùng với các điểm A và B). a) Chứng minh rằng MD là đường phân giác của góc BMC.
  • 37. 36 b) Cho AD = 2R. Tính diện tích của tứ giác ABDC theo R c) Gọi K là giao điểm của AB và MD, H là giao điểm của AD và MC. Chứng minh rằng ba đường thẳng AM, BD, HK đồng quy. 2. Tuyển sinh lớp 10 thành phố Đà Nẵng năm học 2010-2011 Bài 1 (2,0 điểm) a) Rút gọn biểu thức A ( 20 45 3 5). 5 = − + b) Tính 2 B ( 3 1) 3 = − − Bài 2 (2,0 điểm) a) Giải phương trình 4 2 x 13x 30 0 − − = b) Giải hệ phương trình 3 1 7 x y 2 1 8 x y  − =     − =   Bài 3 (2,5 điểm) Cho hai hàm số y = 2x2 có đồ thị (P) và y = x + 3 có đồ thị (d). a) Vẽ các đồ thị (P) và (d) trên cùng một mặt phẳng tọa độ Oxy. b) Gọi A là giao điểm của hai đồ thị (P) và (d) có hoành độ âm. Viết phương trình của đường thẳng (∆) đi qua A và có hệ số góc bằng - 1. c) Đường thẳng (∆) cắt trục tung tại C, cắt trục hoành tại D. Đường thẳng (d) cắt trục hoành tại B. Tính tỉ số diện tích của hai tam giác ABC và tam giác ABD. Bài 4 (3,5 điểm) Cho hai đường tròn (C) tâm O, bán kính R và đường tròn (C') tâm O', bán kính R' (R R') cắt nhau tại hai điểm A và B. Vẽ tiếp tuyến chung MN của hai đường tròn (M ∈ (C), N ∈ (C')). Đường thẳng AB cắt MN tại I (B nằm giữa A và I). a) Chứng minh rằng BMN MAB = b) Chứng minh rằng IN2 = IA.IB c) Đường thẳng MA cắt đường thẳng NB tại Q; đường thẳng NA cắt đường thẳng MB tại P. Chứng minh rằng MN song song với QP. 3. Tuyển sinh lớp 10 thành phố Đà Nẵng năm học 2009-2010 Bài 1. ( 3 điểm ) Cho biểu thức a 1 1 2 K : a 1 a 1 a a a 1     = − +     − − − +     a) Rút gọn biểu thức K. b) Tính giá trị của K khi a = 3 + 2 2
  • 38. 37 c) Tìm các giá trị của a sao cho K 0. Bài 2. ( 2 điểm ) Cho hệ phương trình: mx y 1 x y 334 2 3 − =    − =   a) Giải hệ phương trình khi cho m = 1. b) Tìm giá trị của m để phương trình vô nghiệm. Bài 3. ( 3,5 điểm ) Cho đường tròn (O), đường kính AB cố định, điểm I nằm giữa A và O sao cho AI = 2 3 AO. Kẻ dây MN vuông góc với AB tại I. Gọi C là điểm tùy ý thuộc cung lớn MN sao cho C không trùng với M, N và B. Nối AC cắt MN tại E. a) Chứng minh tứ giác IECB nội tiếp được trong một đường tròn. b) Chứng minh ∆AME ∆ACM và AM2 = AE.AC. c) Chứng minh AE.AC - AI.IB = AI2 . d) Hãy xác định vị trí của điểm C sao cho khoảng cách từ N đến tâm đường tròn ngoại tiếp tam giác CME là nhỏ nhất. Bài 4. ( 1,5 điểm ) Người ta rót đầy nước vào một chiếc ly hình nón thì được 8 cm3 . Sau đó người ta rót nước từ ly ra để chiều cao mực nước chỉ còn lại một nửa. Hãy tính thể tích lượng nước còn lại trong ly. 4. Tuyển sinh lớp 10 thành phố Hà Nội năm học 2010-2011 Bài I (2,5 điểm) Cho biểu thức : A = 2 3 9 9 3 3 x x x x x x + + − − + − , với x≥ 0 và x≠ 9. 1) Rút gọn biểu thức A. 2) Tìm giá trị của x để A = 1/3 3) Tìm giá trị lớn nhất của biểu thức A. Bài II (2,5 điểm)Giải bài toán sau bằng cách lập phương trình: Một mảnh đất hình chữ nhật có độ dài đường chéo là 13 m và chiều dài lớn hơn chiều rộng 7 m. Tính chiều dài và chiều rộng của mảnh đất đó. Bài III (1,0 điểm) Cho parabol (P): y = -x2 và đường thẳng (d): y = mx – 1. 1) Chứng minh với mọi giá trị của m thì đường thẳng (d) luôn cắt parabol (P) tại hai điểm phân biệt.
  • 39. 38 2) Gọi x1, x2 lần lượt là hoành độ các giao điểm của đường thẳng (d) và parabol (P). Tìm giá trị của m để: x1 2 x2 + x2 2 x1 – x1x2 = 3. Bài IV (3,5 điểm) Cho đường tròn (O) có đường kính AB = 2R và điểm C thuộc đường tròn đó (C khác A, B). Lấy điểm D thuộc dây BC (D khác B, C). Tia AD cắt cung nhỏ BC tại điểm E, tia AC cắt tia BE tại điểm F. 1) Chứng minh FCDE là tứ giác nội tiếp. 2) Chứng minh DA.DE = DB.DC. 3) Chứng minh CFD = OCB . Gọi I là tâm đường tròn ngoại tiếp tứ giác FCDE, chứng minh IC là tiếp tuyến của đường tròn (O). 4) Cho biết DF = R, chứng minh tg AFB = 2. Bài V ( 0,5 điểm) Giải phương trình: x2 + 4x + 7 = (x + 4) 2 7 x + 5. Tuyển sinh lớp 10 thành phố Hà Nội năm học 2011-2012 Bài I (2,5 điểm) Cho x 10 x 5 A x 25 x 5 x 5 = − − − − + Với x 0,x 25 ≥ ≠ . 1) Rút gọn biểu thức A. 2) Tính giá trị của A khi x = 9. 3) Tìm x để 1 A 3 . Bài II (2,5 điểm) Giải bài toán sau bằng cách lập phương trình hoặc hệ phương trình: Một đội xe theo kế hoạch chở hết 140 tấn hàng trong một số ngày quy định. Do mỗi ngày đội đó chở vượt mức 5 tấn nên đội đã hoàn thành kế hoạch sớm hơn thời gian quy định 1 ngày và chở thêm được 10 tấn. Hỏi theo kế hoạch đội xe chở hàng hết bao nhiêu ngày? Bài III (1,0 điểm) Cho Parabol (P): 2 y x = và đường thẳng (d): 2 y 2x m 9 = − + . 1) Tìm toạ độ các giao điểm của Parabol (P) và đường thẳng (d) khi m = 1. 2) Tìm m để đường thẳng (d) cắt Parabol (P) tại hai điểm nằm về hai phía của trục tung. Bài IV (3,5 điểm) Cho đường tròn tâm O, đường kính AB = 2R. Gọi d1 và d2 là hai tiếp tuyến của đường tròn (O) tại hai điểm A và B.Gọi I là trung điểm của OA và E là điểm thuộc đường tròn (O) (E
  • 40. 39 không trùng với A và B). Đường thẳng d đi qua điểm E và vuông góc với EI cắt hai đường thẳng d1 và d2 lần lượt tại M, N. 1) Chứng minh AMEI là tứ giác nội tiếp. 2) Chứng minh ENI EBI ∠ = ∠ và 0 MIN 90 ∠ = . 3) Chứng minh AM.BN = AI.BI . 4) Gọi F là điểm chính giữa của cung AB không chứa E của đường tròn (O). Hãy tính diện tích của tam giác MIN theo R khi ba điểm E, I, F thẳng hàng. Bài V (0,5 điểm) Với x 0, tìm giá trị nhỏ nhất của biểu thức: 2 1 M 4x 3x 2011 4x = − + + . 6. Tuyển sinh lớp 10 tỉnh Hải Dương năm học 2011-2012 Câu 1 (3,0 điểm). 1)Giải các phương trình: a)5( 1) 3 7 + = + x x b) 4 2 3 4 1 ( 1) + + = − − x x x x x 2)Cho hai đường thẳng (d1): 2 5 y x = + ; (d2): 4 1 y x = − − cắt nhau tại I. Tìm m để đường thẳng (d3): ( 1) 2 1 y m x m = + + − đi qua điểm I. Câu 2 (2,0 điểm). Cho phương trình: 2 2( 1) 2 0 x m x m − + + = (1) (với ẩn là x ). 1)Giải phương trình (1) khi m=1. 2)Chứng minh phương trình (1) luôn có hai nghiệm phân biệt với mọi m. 3)Gọi hai nghiệm của phương trình (1) là 1 x ; 2 x . Tìm giá trị của m để 1 x ; 2 x là độ dài hai cạnh của một tam giác vuông có cạnh huyền bằng 12 . Câu 3 (1,0 điểm). Một hình chữ nhật có chu vi là 52 m. Nếu giảm mỗi cạnh đi 4 m thì được một hình chữ nhật mới có diện tích 77 m2 . Tính các kích thước của hình chữ nhật ban đầu? Câu 4 (3,0 điểm). Cho tam giác ABC có Â 900 . Vẽ đường tròn (O) đường kính AB và đường tròn (O’) đường kính AC. Đường thẳng AB cắt đường tròn (O’) tại điểm thứ hai là D, đường thẳng AC cắt đường tròn (O) tại điểm thứ hai là E. 1)Chứng minh bốn điểm B, C, D, E cùng nằm trên một đường tròn. 2)Gọi F là giao điểm của hai đường tròn (O) và (O’) (F khác A). Chứng minh ba điểm B, F, C thẳng hàng và FA là phân giác của góc EFD. 3)Gọi H là giao điểm của AB và EF. Chứng minh BH.AD = AH.BD.
  • 41. 40 Câu 5 (1,0 điểm). Cho x, y, z là ba số dương thoả mãn x + y + z =3. Chứng minh rằng: 1 3 3 3 + + ≤ + + + + + + x y z x x yz y y zx z z xy . 7. Tuyển sinh lớp 10 tỉnh Hà Tĩnh năm học 2010-2011 Câu I (3 điểm). Cho biểu thức A = 2 2 1 1 1 x x x x − − − − + . 1. Nêu điều kiện xác định và rút gọn biểu thức A. 2. Tính giá trị của biểu thức A khi x = 9. 3. Khi x thoả mãn điều kiện xác định. Hãy tìm giá trị nhỏ nhất của biểu thức B, với B = A(x – 1). Câu II (2 điểm). Cho phương trình bậc hai sau, với tham số m: x2 – (m + 1)x + 2m – 2 = 0 (1) 1. Giải phương trình (1) khi m = 2. 2. Tìm giá trị của tham số m để x = -2 là một nghiệm của phương trình (1). Câu III (1,5 điểm). Hai người cùng làm chung một công việc thì sau 4 giờ 30 phút họ làm xong. Nếu một mình người thứ nhất làm trong 4 giờ, sau đó một mình người thứ hai làm trong 3 giờ thì cả hai người làm được 75% công việc. Hỏi nếu mỗi người làm một mình thì sau bao lâu sẽ xong công việc? (Biết rằng năng suất làm việc của mỗi người là không thay đổi). Câu IV (3,5 điểm). Cho nửa đường tròn tâm O đường kính AB. Điểm H cố định thuộc đoạn thẳng AO (H khác A và O). Đường thẳng đi qua điểm H và vuông góc với AO cắt nửa đường tròn (O) tại C. Trên cung BC lấy điểm D bất kỳ (D khác B và C). Tiếp tuyến của nửa đường tròn (O) tại D cắt đường thẳng HC tại E. Gọi I là giao điểm của AD và HC. 1. Chứng minh tứ giác HBDI nội tiếp đường tròn. 2. Chứng minh tam giác DEI là tam giác cân. 3. Gọi F là tâm đường tròn ngoại tiếp tam giác ICD. Chứng minh góc ABF có số đo không đổi khi D thay đổi trên cung BC (D khác B và C). 8. Tuyển sinh lớp 10 tỉnh Nghệ An năm học 2011-2012 Câu I (3,0 điểm) Cho biểu thức A = ( ) 2 1 1 1 : 1 1 x x x x x +   +   − −   − a) Nêu ĐKXĐ và rút gọn A. b) Tìm giá trị của x để A = 1 3 .
  • 42. 41 c) Tìm giá trị lớn nhất của biểu thức P = A - 9 x . Câu 2. (2,0 điểm) Cho phương trình bậc hai: x2 – 2(m + 2)x + m2 + 7 = 0 (1), (m là tham số) a) Giải phương trình (1) khi m = 1. b) Tìm m để phương trình (1) có hai nghiệm x1, x2 thỏa mãn: x1x2 – 2(x1 + x2) = 4. Câu 3(1,5 điểm) Quãng đường AB dài 120 km. Hai xe máy khởi hành cùng một lúc đi từ A đến B. Vận tốc của xe thứ nhất lớn hơn vận tốc của xe thứ hai là 10 km/h nên xe máy thứ nhất đến B trước xe thứ hai 1 giờ. Tính vận tốc của mỗi xe. Câu 4. (3,5 điểm) Cho A nằm ngoài đường tròn (O). Từ A kẻ hai tiếp tuyến AB, AC và cát tuyến ADE tới đường tròn đó (B, C là hai tiếp điểm; D nằm giữa A và E). Gọi H là giao điểm của AO và BC. a) Chứng minh rằng ABOC là tứ giác nội tiếp. b) Chứng minh rằng: AH. AO = AD. AE c) Tiếp tuyến tại D của đường tròn (O) cắt AB, AC theo thứ tự tại I và K. Qua điểm O kẻ đường thẳng vuông góc với OA cắt AB tại P và cắt AC tại Q. Chứng minh rằng: IP + KQ ≥ PQ 9. Tuyển sinh lớp 10 tỉnh Hà Tĩnh năm học 2011-2012 Câu 1 a)Tìm m để đường thẳng y = (2m – 1)x + 3 song song với đường thẳng y = 5x – 1. b)Giải hệ phương trình: 2 5 3 2 4 x y x y + =   − =  Câu 2 Cho biểu thức: 1 1 1 1 1 1 P a a a    = − +    − +    với a 0 và 1 a ≠ a)Rút gọn biểu thức P. b)Với những giá trị nào của a thì P 1 2 . Câu 3 a)Tìm tọa độ giao điểm của đồ thị các hàm số: y = x2 và y = - x + 2. b)Xác định các giá trị của m để phương trình x2 – x + 1 – m = 0 có 2 nghiệm x1, x2 thỏa mãn đẳng thức: 1 2 1 2 1 1 5 4 0 x x x x   + − + =     . Câu 4 Trên nửa đường tròn đường kính AB, lấy hai điểm P, Q sao cho P thuộc cung AQ. Gọi C là giao điểm của tia AP và tia BQ; H là giao điểm của hai dây cung AQ và BP. a)Chứng minh tứ giác CPHQ nội tiếp đường tròn. b)Chứng minh CBP ∆ HAP ∆ . c)Biết AB = 2R, tính theo R giá trị của biểu thức: S = AP.AC + BQ.BC. Câu 5 Cho các số a, b, c đều lớn hơn 25 4 . Tìm giá trị nhỏ nhất của biểu thức:
  • 43. 2 5 2 5 2 5 a b c Q b c a = + + − − − . 10. Tuyển sinh lớp 10 tỉnh Bình Định năm học 2011-2012 Bài 1: (2,0 điểm) a)Giải hệ phương trình  −   3x y = 7 2x + y = 8 . b) Cho hàm số y = ax + b . Tìm a và b biết rằng đồ thị của hàm số đã cho song song với đường thẳng 2 3 y x = − + và đi qua điểm M(2; 5). Bài 2: (2,0 điểm) Cho phương trình ( ) 2 2 1 4 0 x m x m + + + − = , với m là tham số. a) Giải phương trình đã cho khi . b) Chứng tỏ phương trình đã cho luôn có hai nghiệm phân biệt với mọi giá trị của tham số m. c) Tìm m để phương trình đã cho có nghiệm x1, x2 thõa mãn hệ thức 2 2 1 2 1 2 3 0 x x x x + + = . Bài 3: (2,0 điểm) Một mảnh đất hình chữ nhật có chiều dài hơn chiều rộng 6m và bình phương của số đo độ dài đường chéo gấp 5 lần số đo của chu vi. Tính diện tích của mảnh đất hình chữ nhật đã cho. Bài 4: (3,0 điểm) Cho đường tròn tâm O và BC là dây cung không đi qua tâm. Trên tia đối của tia BC lấy điểm M sao cho M không trùng với B. Đường thẳng đi qua M cắt đường tròn (O) đã cho tại N và P (N nằm giữa M và P) sao cho O nằm bên trong . Gọi A là điểm chính giữa của cung nhỏ NP. Các dây AB và AC lần lượt cắt NP tại D và E . a) Chứng minh tứ giác BDEC nội tiếp. b) Chứng tỏ MB.MC = MN.MP . c) OA cắt NP tại K. Chứng minh MK2 MB.MC . Bài 5: (1,0 điểm) Tìm giá trị nhỏ nhất của biểu thức 2 2 2 2011 x x A x − + = (với ) 11. Tuyển sinh lớp 10 tỉnh Quảng Nam năm học 2011-2012 Bài 1 (2,0 điểm) Rút gọn các biểu thức sau: a)A 2 5 3 45 500 = + − b) 1 15 12 B 5 2 3 2 − = − − + Bài 2 (2,5 điểm) 1) Giải hệ phương trình: 3x y 1 3x 8y 19    − = + = 2) Cho phương trình bậc hai: 2 x mx +m 1= 0 (1) − − a) Giải phương trình (1) khi m = 4. b) Tìm các giá trị của m để phương trình (1) có hai nghiệm 1 2 x ;x thỏa 1 2 1 2 x x 1 1 x x 2011 + + = . m 5 = − PMC x 0 ≠
  • 44. 43 Bài 3 (1,5 điểm) Cho hàm số y = 2 1 x 4 . 1) Vẽ đồ thị (P) của hàm số đó. 2) Xác định a, b để đường thẳng (d): y = ax + b cắt trục tung tại điểm có tung độ bằng –2 và cắt đồ thị (P) nói trên tại điểm có hoành độ bằng 2. Bài 4 (4,0 điểm) Cho nửa đường tròn (O; R) đường kính AB. Gọi C là điểm chính giữa của cung AB. Trên tia đối của tia CB lấy điểm D sao cho CD = CB. OD cắt AC tại M. Từ A, kẻ AH vuông góc với OD (H thuộc OD). AH cắt DB tại N và cắt nửa đường tròn (O; R) tại E. 1) Chứng minh MCNH là tứ giác nội tiếp và OD song song với EB. 2) Gọi K là giao điểm của EC và OD. Chứng minh rằng ∆CKD = ∆CEB. Suy ra C là trung điểm của KE. 3) Chứng minh tam giác EHK vuông cân và MN song song với AB. 4) Tính theo R diện tích hình tròn ngoại tiếp tứ giác MCNH. 12. Tuyển sinh lớp 10 tỉnh Quảng Ngãi năm học 2011-2012 Bài 1: (1.5 điểm) 1) Thực hiện phép tính: 2 9 3 16 + 2) Giải phương trình và hệ phương trình sau: a) x2 – 20x + 96 = 0 b) 4023 1 x y x y + =   − =  Bài 2: (2.5điểm) 1) Cho hàm số y = x2 có đồ thị là (P) và đường thẳng (d): y = x + 2 a) Vẽ ( P ) và ( d ) trên cùng một hệ toạ độ Oxy b) Bằng phép tính hãy tìm toạ độ giao điểm của ( P ) và ( d ) 2) Trong cùng một hệ toạ độ Oxy cho 3 điểm: A(2;4); B(-3;-1) và C(-2;1). Chứng minh 3 điểm A, B, C không thẳng hàng. 3) Rút gọn biểu thức: 2 1 x x x M x x x − = + − − với 0; 1 x x ≠ Bài 3: (1.5điểm) Hai bến sông cách nhau 15 km. Thơì gian một ca nô xuôi dòng từ bến A đến bến B, tại bến B nghỉ 20 phút rồi ngược dòng từ bến B trở về bến A tổng cộng là 3 giờ. Tính vận tốc của ca nô khi nước yên lặng, biết vận tốc của dòng nước là 3 km/h. Bài 4: (3.5 điểm) Cho nửa đường tròn tâm O đường kính AB. Một điểm C cố định thuộc đoạn thẳng AO ( C khác A và C khác O ). Đường thẳng đi qua điểm C và vuông góc với AO cắt nửa đường tròn đã cho tại D. Trên cung BD lấy điểm M ( với M khác B và M khác D). Tiếp tuyến của nửa đường tròn đã cho tại M cắt đường thẳng CD tại E. Gọi F là giao điểm của AM và CD. 1)Chứng minh : BCFM là tứ giác nội tiếp đường tròn. 2)Chứng minh EM = EF 3)Gọi I là tâm đường tròn ngoại tiếp tam giác FDM. Chứng minh D, I, B thẳng hàng; từ đó suy ra góc ABI có số đo không đổi khi M thay đổi trên cung BD.
  • 45. Bài 5:(1.0 điểm) Cho phương trình ( ẩn x ): ( ) 2 2 3 0 x m x m − + + = . Gọi x1 và x2 là hai nghiệm của phương trình đã cho. Tìm giá trị của m để biểu thức 2 2 1 2 x x + có giá trị nhỏ nhất. 13. Tuyển sinh lớp 10 tỉnh Thanh Hoá năm học 2011-2012 Bài 1: ( 1,5 điểm ) 1. Cho hai số : b1 = 1 + ; b2 = 1 - . Tính b1 + b2 2. Giải hệ phương trình Bài 2: ( 1,5 điểm ) Cho biểu thức B = với b và b 4 1. Rút gọn biểu thức B 2. Tính giá trị của B tại b = 6 + 4 Bài 3: ( 2,5 điểm ) Cho phương trình : x2 - ( 2n -1 )x + n (n - 1) = 0 ( 1 ) với n là tham số 1. Giải phương trình (1) với n = 2 2. Chứng minh rằng phương trình (1) luôn có hai nghiệm phân biệt với mọi n 3. Gọi x1, x2 là hai nghiệm của phương trình (1) (vơí x1x2). Chứng minh x1 2 - 2x2 + 3 0. Bài 4: ( 3 điểm ) Cho tam giác BCD có 3 góc nhọn. Các đường cao CE và DF cắt nhau tại H . 1. CM: Tứ giác BFHE nội tiếp được trong một đường tròn 2. Chứng minh BFE và BDC đồng dạng 3. Kẻ tiếp tuyến Ey của đường tròn tâm O đường kính CD cắt BH tại N. CMR: N là trung điểm của BH . Bài 5: ( 1 điểm ) Cho các số dương x, y, z . Chứng minh rằng 14. Tuyển sinh lớp 10 tỉnh Khánh Hoà năm học 2011-2012 Bài 1( 2 điểm) 1)Đơn giản biểu thức: A 2 3 6 8 4 2 3 4 + + + + = + + 2) Cho biểu thức: 1 1 ;( 1) 1 1 P a a a a a a   = − − ≥   − − + −   . Rút gọn P và chứng tỏ P ≥ 0. Bài 2( 2 điểm) 1)Cho phương trình bậc hai x2 + 5x + 3 = 0 có hai nghiệm x1; x2. Hãy lập một phương trình bậc hai có hai nghiệm (x1 2 + 1 ) và ( x2 2 + 1). 2)Giải hệ phương trình 2 3 4 2 4 1 1 2 x y x y  + =  −    − =  −  2 2    − = − = + 3 2 1 2 n m n m 2 1 : ) 4 1 4 2 2 ( + − − + − − + b b b b b b b 0 ≥ ≠ 2 ≥ ∆ ∆ ∆ 2 + + + + + y x z z x y z y x
  • 46. 45 Bài 3( 2 điểm) Quãng đường từ A đến B dài 50km. Một người dự định đi xe đạp từ A đến B với vận tốc không đổi. Khi đi được 2 giờ, người ấy dừng lại 30 phút để nghỉ. Muốn đến B đúng thời gian đã định, người đó phải tăng vận tốc thêm 2 km/h trên quãng đường còn lại. Tính vận tốc ban đầu của người đi xe đạp. Bài 4( 4 điểm) Cho tam giác ABC có ba góc nhọn và H là trực tâm. Vẽ hình bình hành BHCD. Đường thẳng đi qua D và song song BC cắt đường thẳng AH tại E. 1) Chứng minh A,B,C,D,E cùng thuộc một đường tròn 2) Chứng minh BAE DAC ∠ = ∠ 3) Gọi O là tâm đường tròn ngoại tiếp tam giác ABC và M là trung điểm của BC,đường thẳng AM cắt OH tại G.Chứng minh G là trọng tâm của tam giácABC. 4) Giả sử OD = a.Hãy tính độ dài đường tròn ngoại tiếp tam giác BHC theo a 15. Tuyển sinh lớp 10 tỉnh Bình Dương năm học 2010-2011 Bài 1 (1đ) Rút gọn 2 16 8 1 M x x = + + . Tính giá trị của M tại x = 2. Bài 2 (1đ5) 1) Vẽ đồ thị của các hàm số sau trên cùng một mặt phẳng tọa độ 2 ( ) : P y x = ; ( ) : 2 3 d y x = + 2) Tìm tọa độ giao điểm (nếu có) của (d) và (P). Bài 3(2đ) 1) Giải phương trình 2 5 6 0 x x + + = 2) Giải hệ phương trình 3 4 2 5 7 x y x y + =   + =  Bài 4 (2đ) 1) Một người dự định đi xe gắn máy từ địa điểm A đến địa điểm B cách nhau 90km. Vì có việc gấp phải đến B trước giờ dự định là 45 phút nên người ấy phải tăng vận tốc lên mỗi giờ 10 km . Hãy tính vận tốc mà người đó dự định đi . 2) Chứng minh rằng phương trình ( ) 2 2 2 1 4 8 0 x m x m − − + − = (m là tham số) luôn có 2 nghiệm phân biệt và khác 1 với mọi m ∈ R . Bài 5 (3đ5) Một hình vuông ABCD nội tiếp trong đường tròn Tâm O bán kính R . Một điểm M di động trên cung ABC , M không trùng với A,B và C, MD cắt AC tại H. 1) Chứng minh tứ giác MBOH nội tiếp được trong đường tròn và DH.DM = 2R2 . 2) Chứng minh tam giác MDC đồng dạng với tam giác MAH . 3) Hai tam giác MDC và MAH bằng nhau khi M ở một vị trí đặc biệt M’. Xác định điểm M’. Khi đó M’D cắt AC tại H’. Đường thẳng qua M’ và vuông góc với AC cắt AC tại I. Chứng minh rằng I là trung điểm của H’C .
  • 47. 46 16. Tuyển sinh lớp 10 tỉnh Bến Tre năm học 2011-2012 I. PHẦN TRẮC NGHIỆM: Thời gian làm bài 20 phút / 3,0 điểm (Chọn phương án đúng cho mỗi câu và ghi vào giấy làm bài . Ví dụ: câu 1 chọn A thì ghi 1.A) Câu 1. Biểu thức M = 4 2 3 3 + − có giá trị bằng: A. 2 3 1 − B. 1 2 3 − C. 1 D. -1 Câu 2. Với giá trị nào của m thì đường thẳng (d1): mx – 2y = 2 cắt đường thẳng (d2): x + y = 3? A. 2 m ≠ − B. 2 m ≠ C. 2 m = − D. 2 m = Câu 3. Hệ phương trình 2 4 2 x y x y    + = − = có nghiệm (x;y). Tổng x + y bằng: A.0 B. 2 C. 4 D. 6 Câu 4. Đồ thị hàm số y = f(x) = ax2 đi qua điểm A(-2; 4) có hệ số a bằng: A. -1 B. 1 C. 1 8 D. 1 8 − Câu 5. Cho hàm số y = f(x) = ax2 . Nếu f(2) = 1 th ì f(-2) + 2 bằng: A. 1 B. 2 C. 3 D. 4 Câu 6. Nếu 0 1 3 x = − là nghiệm của phương trình 2 1 x x m − + = thì m bằng: A. 4 3 − B. 4 3 + C. 4 3 12 − D. 4 3 2 + Câu 7. Với giá trị nào của m thì phương trình ( ) 2 2 1 2 0 mx m x m + − + + = có nghiệm? A. 1 12 m ≥ B. 1 12 m ≤ C. 1 12 m ≥ và 0 m ≠ D. 1 12 m và 0 m ≠ Câu 8. Phương trình nào sau đây nhận 1 2 2 3; 2 3 x x = − = + là nghiệm? A. 2 4 0 x x + + = B. 2 4 0 x x − − = C. 2 4 1 0 x x + + = D. 2 4 1 0 x x − + = Câu 9. Tam giác ABC cân tại A nội tiếp đường tròn (O) có 0 A 60 = , số đo của AOB bằng: A. 0 65 B. 0 120 C. 0 130 D. 0 135 Câu 10. Cho tam giác ABC cân tại B có 6 AC cm = , 0 120 B = . Độ dài đường tròn ngoại tiếp tam giác ABC tính bằng cm là:
  • 48. 47 A. 3 π B. 2 3 π C.4 3 π D. 5 3 π Câu 11. Một ngọn tháp cao 50, có bóng trên mặt đất dài 15m. Góc mà tia sáng mặt trời tạo với mặt đất (làm tròn đến độ) là: A. 0 71 B. 0 73 C. 0 75 D. 0 80 Câu 12. Cho tam giác ABC vuông tại A. Biết rằng 5 6 AB AC = , đường cao 30 . AH cm = Độ dài BH tính bằng cm là: A.18 B.20 C.25 D.36 II. PHẦN TỰ LUẬN: Thời gian làm bài 100 phút/7 điểm. Bài 1. (1,0 điểm) Cho biểu thức 1 1 1 2 : 1 2 1 x x A x x x x + +     = − −     − − −     . 1. Rút gọn A khi 0; 1; 2 x x x ≠ ≠ ≠ 2. Tìm x để giá trị của 3 3 A = − . Bài 2. (2,0 điểm) Cho hệ phương trình 2 3 5 2 x y m x y m + = +   + =  với m là tham số. 1. Giải hệ phương trình khi 1 m = − . 2. Xác định giá trị của m để hệ phương trình có nghiệm ( ) ; x y thoả mãn điều kiện: 1 x y + = Bài 3. (1,5 điểm) Cho phương trình ( ) 2 2 1 3 0 x m x m − + − − = với m là tham số. 1. Chứng minh rằng phương trình luôn có hai nghiệm phân biệt. 2. Gọi 1 2 , x x là hai nghiệm của phương trình. Tìm m để ( ) 2 1 2 x x − đạt giá trị nhỏ nhất. Bài 4. (2,5 điểm) Cho góc xOy và điểm P nằm trong góc đó. Gọi H và K lần lượt là hình chiếu của P lên Ox và Oy. Đường thẳng PK cắt Ox tại A, đường thẳng PH cắt Oy tại B. 1. Chứng minh tứ giác OKPH và tứ giác KHAB nội tiếp đường tròn. 2. Cho 0 60 xOy = và OP a = . Tính độ dài HK và AB theo a. 3. Gọi M và N lần lượt là trung điểm OP và AB. Chứng minh tứ giác MKNH nội tiếp đường tròn.
  • 49. 48 V. Đề tham khảo ôn tập tuyển sinh 1. ĐỀ SỐ 01 Bài 1. Giải phương trình và hệ phương trình: a) 4 2 4 2 1 0 x x − − = b) 3 2 0 2 5 5 x y x y − =   + = −  Bài 2. Rút gọn biểu thức: a) 2 2 3 5 3 5 + + − b) 17 12 2 2 2 2 1 3 2 2 2 2 3 2 − + − + − + − Bài 3. Cho hàm số ( ) 2 2 x y P = − và ( ) 2 y x d = . a)Vẽ (P) và (d) trên cùng một hệ trục toạ độ. b)Tìm toạ độ giao điểm của (P) và (d) bằng phép toán. c)Viết phương trình đường thẳng ( ) 1 d // ( ) d và tiếp xúc với ( ) P . Bài 4. Cho phương trình 2 2 2 1 0 x mx m − + − = . a)Tìm m để phương trình có 2 nghiệm cùng dương. b)Với m tìm được ở câu a hãy tính 1 2 x x + theo m. c)Với giá trị nào của m thì phương trình có 2 nghiệm đối nhau. Bài 5. Cho nửa đường tròn (O), đường kính AD. Trên nửa đường tròn lấy 2 điểm B và C sao cho cung AB nhỏ hơn cung AC, AC cắt BD tại E. Kẻ EF vuông góc với AD tại F. a)Chứng minh rằng ADEF nội tiếp. b)Chứng minh rằng BD là phân giác của CEF . c)Gọi M là trung điểm của ED. Chứng minh rằng BCMF nội tiếp. d)CF cắt BD tại I. Chứng minh rằng 2 . . BI BF BC IF IC = − . 2. ĐỀ SỐ 02 Bài 1. Rút gọn biểu thức
  • 50. 49 a) ( )( ) 4 7 2 14 4 7 A = − + + b) 2 2 2 2 2 2 2 2 B = + + + − + Bài 2. Giải các phương trình và hệ phương trình: a) ( ) 4 2 2 5 2 5 0 x x − − − = b) 3 4 2 3 2 3 3 x y x y  − =   + = −   Bài 3. Cho hàm số ( ) 2 1 2 P y x = − và ( ) 2 D y x = . a)Vẽ (P) và (D) trên cùng một hệ trục toạ độ Oxy. b)Bằng phép toán, tìm toạ độ giao điểm (P) và (D). c)Viết phương trình đường thẳng (d) đi qua A(-3; -4) và tiếp xúc với (P). Bài 4. Cho phương trình ( ) 2 2 1 4 0 x m x m − + + − = . a)Chứng minh rằng phương trình có hai nghiệm phân biệt 1 2 , x x với mọi m. b)Tìm m để phương trình có hai nghiệm trái dấu. c)Chứng minh rằng biểu thức ( ) ( ) 1 2 2 1 1 1 M x x x x = − + − không phụ thuộc vào m. Bài 5. Cho tam giác ABC vuông tại A (AB AC). D là điểm thuộc AC. Vẽ DE vuông góc BC tại E. a)Chứng minh rằng ADEB nội tiếp. Xác định tâm O của đường tròn ngoại tiếp này. b)Vẽ đường tròn tâm D bán kính DE cắt (O) tại F, BF cắt AD tại I, BD cắt AE tại K. Chứng minh rằng ABKI nội tiếp. c)Chứng minh rằng BI. BF = BK. BD d)Trung tuyến AM của tam giác ABC cắt tại N. Chứng minh rằng NA = NF. 3. ĐỀ SỐ 03 Bài 1. Cho biểu thức 2 2 2 : ; 0, 1 1 1 2 1 x x x A x x x x x x   − + = − ≠     − − + +   . a)Rút gọn biểu thức A. b)Tìm x để A có giá trị nguyên. Bài 2. Giải phương trình và hệ phương trình
  • 51. 50 a) 4 2 25 144 0 x x − + = b) ( ) 2 2 2 6 0 x x − − = c) 2 0 4 3 20 x y x y − =   − =  d) 2 2 3 2 0 x x − − = Bài 3. a)Vẽ đồ thị hàm số ( ) 2 y x P = − và ( ) 6 D y x = − trên cùng một hệ trục toạ độ. b)Tìm toạ độ giao điểm của (P) và (D) bằng phép toán. c)Viết phương trình đường thẳng (D’) vuông góc với (D) và tiếp xúc với (P). Bài 4. Cho phương trình ( ) 2 2 2 4 8 0 x m x m − + + − = . a)Định m để phương trình có nghiệm. b)Gọi 1 2 , x x là 2 nghiệm của. Lập hệ thức liên hệ giữa 1 2 , x x không phụ thuộc vào m. c)Tìm m để 2 2 1 2 1 2 A x x x x = − − đạt giá trị nhỏ nhất. Tìm giá trị đó. Bài 5. Từ đỉnh A của hình vuông ABCD, ta kẻ hai tia tạo với nhau một góc 45o . Một tia cắt BC tại E và cắt BD tại P, tia kia cắt CD tại F và cắt BD tại Q. a)Chứng minh rằng APED nội tiếp. b)Chứng minh rằng E, P, Q, F, C cùng thuộc một đường tròn. c)Chứng minh rằng 2 AEF APQ S S = . d)Kẻ đường trung trực của cạnh CD cắt AE tại M. Tính MAB khi tứ giác CPMD nội tiếp. 4. ĐỀ SỐ 04 Bài 1. Giải phương trình và hệ phương trình a) 4 2 3 10 8 0 x x + + = b) ( ) 2 2 3 2 3 0 x x − − − = c) 3 2 0 2 3 5 x y x y − =   + = −  Bài 2. Cho hai hàm số ( ) 2 4 x y P = và ( ) 1 y x d = − − . a)Vẽ (P) và (d) trên cùng một hệ trục toạ. b)Tìm toạ độ giao điểm của (P) và (d) bằng phép toán. c)Viết phương trình đường thẳng ( ) ( ) 1 / / d d và cắt (P) tại điểm có hoành độ bằng O. Bài 3. Cho phương trình ( ) 2 2 2 0 x m x m + − − = .
  • 52. 51 a)Chứng minh rằng phương trình có nghiệm với mọi m. b)Tìm m để phương trình có nghiệm số kép và tính nghiệm đó. c)Với giá trị nào của m thì phương trình có 2 nghiệm phân biệt là hai số nghịch đảo của nhau. Bài 4. Rút gọn a) 13 160 53 4 90 − − + b) 23 2 2 14 5 3 + + Bài 5. Cho tam giác ABC nhọn (AB AC) nội tiếp trong (O; R) có BE và CF là các đường cao cắt nhau tại H. a)Chứng minh rằng BFEC nội tiếp, xác định tâm I. b)Tia AH cắt BC tại D. Chứng minh rằng EB là phân giác DEF . c)Chứng minh rằng OA vuông góc với EF. d)Đường thẳng EF cắt (O) tại N và M (F nằm giữa N và E). Chứng minh rằng AN là tiếp tuyến của đường tròn ngoại tiếp tam giác NHD. e)Vẽ (H, HA) cắt AB và AC lần lượt tại P và Q. Chứng minh rằng đường thẳng kẻ từ A vuông góc với PQ đi qua điểm P cố định. 5. ĐỀ SỐ 05 Bài 1. Tính a) ( ) 3 5 10 2 A = − + b) 3 8 15 1 2 30 2 B − = − + − Bài 2. Giải phương trình và hệ phương trình a) 4 2 4 5 9 0 x x − − = b) ( ) 2 1 3 3 0 x x − + + = c) 2 2 3 5 9 3 2 3 2 x y x y  + =   − =   d) 4 4 20 3 5 9 45 6 3 x x x + − + + + = Bài 3. Cho hàm số ( ) 2 2 x y P = và ( ) 2 2 x y D = + . a)Vẽ (P) và (D) trên cùng một hệ trục toạ độ.
  • 53. 52 b)Tìm toạ độ giao điểm của (P) và (D). c)Viết phương trình đường thẳng (D’), biết (D’) // (D) và đi qua B(2; -3). Bài 4. Cho phương trình ( ) 2 2 1 2 1 0 x m x m − + + + = . a)Tìm điều kiện m để phương trình có nghiệm. b)Gọi 1 2 , x x là nghiệm của phương trình. Tìm giá trị nhỏ nhất của ( ) 2 1 2 1 2 8 M x x x x = + + . c)Tìm m để phương trình có hai nghiệm thoả 1 2 2 2 x x + = . Bài 5. Cho (O; R) lấy P ở ngoài đường tròn sao cho OP = 2R. Vẽ cát tuyến PAB, từ A và B vẽ 2 tiếp tuyến của (O) cắt nhau tại M, hạ MH vuông góc với OP tại H. Chứng minh rằng MO vuông góc với AB tại I và tứ giác MIHP nội tiếp. Chứng minh rằng OH luôn không đổi khi cát tuyến PAB quay quanh P. Cho 3 R OI = . Tính PHA S theo R. 6. ĐỀ SỐ 06 Bài 1. Rút gọn biểu thức a) 5 2 5 3 3 5 3 5 3 + + + − − b) ( ) 9 9 6 6 0; 9 3 3 x x x x x x x − − + − − ≥ ≠ + − Bài 2. Giải các phương trình và hệ phương trình sau: a) 2 2 7 3 0 x x − + = b) 4 2 5 4 0 x x − + = c) 2 5 3 2 8 x y x y − =   − =  Bài 3. Cho (P) 2 2 x y = và (D) 1 2 x y = + . Vẽ (P) và (D) trên cùng một hệ trục toạ độ. Xác định toạ độ giao điểm của (P) và (D) bằng đồ thị và kiểm tra lại bằng phép toán. Bài 4. Cho phương trình bậc hai 2 3 2 0 x x m − + − = , trong đó m là tham số, x là ẩn số. a)Tìm điều kiện của m để phương trình có nghiệm. b)Gọi 1 2 , x x là nghiệm của phương trình. Tính 2 3 1 2 1 2 3 3 2 A x x x x = + − theo m.
  • 54. 53 Bài 5. Cho tam giác nhọn ABC (AB AC) nội tiếp trong (O; R) có BE và CF là các đường cao cắt nhau tại H (E thuộc AC, F thuộc AB). a)Chứng minh rằng BFEC nội tiếp trong đường tròn có tâm là M, xác định vị trí điểm M. b)Tia AH cắt BC tại D. Chứng minh rằng EB là tia phân giác góc DEF . c)Vẽ tiếp tuyến xAy của đường tròn (O). Chứng minh rằng OA vuông góc với OF. d)Đường thẳng EF cắt đường tròn tại M, N (F nằm giữa N và E). Chứng minh rằng AN là tiếp tuyến của đường tròn ngoại tiếp tam giác NHD. 7. ĐỀ SỐ 07 Bài 1. Giải phương trình và hệ phương trình a) ( ) 4 2 2 2 2 2 0 x x − − − = b) 2 3 1 3 2 1 1 4 2 3 3 x y x y  − =     + =   Bài 2. Cho (P) 2 2 x y = − và (D) 2 y x = . a)Vẽ (P) và (D) trên cùng một hệ trục toạ độ. b)Tìm toạ độ giao điểm của (P) và (D) bằng phép toán. c)Viết phương trình đường thẳng (D’) // (D) và (D’) cắt (P) tại điểm A có hoành độ bằng -2. Bài 3. Cho phương trình 2 2 3 1 0 x x m − − − = . a)Chứng minh rằng phương trình luôn có nghiệm với mọi x. b)Tìm m để phương trình có hai nghiệm 1 2 , x x thoả 2 2 1 2 29 x x + = c)Tìm m để phương trình có hai nghiệm 1 2 , x x thoả 1 2 5 x x − = Bài 4. Cho 2 2 1 : 1 2 1 x x x Q x x x x   + − + = −     − + +   a)Tìm điều kiện để Q có nghĩa và rút gọn Q. b)Tìm các giá trị nguyên của x để Q có giá trị nguyên. Bài 5. Cho (O; R) đường kinh AB. Tiếp tuyến tại điểm M bất kỳ thuộc (O) (M khác A và B) cắt tiếp tuyến tại A và B của đường tròn này lần lượt tại C và D.
  • 55. 54 a)Chứng minh rằng ACMO và DBOM là các tứ giác nội tiếp. b)Chứng minh rằng AC + BD = CD và tam giác COD vuông. c)Gọi I và K lần lượt là tâm đường tròn nội tiếp của tam giác DBM và tam giác CAM. Chứng minh rằng các điểm I, K thuộc một đường tròn cố định và độ dài IK không đổi khi M di động trên (O). d)Tìm vị trí của điểm M trên (O) để tứ giác CKID có diện tích nhỏ nhất. 8. ĐỀ SỐ 08 Bài 1. Tính và rút gọn các biểu thức sau: a) ( ) 2 4 6 2 5 10 2 A   = + + −     b) ( ) 2 1 1 2 1 0; 1 1 1 1 a a B a a a a a   − +   = + − ≠      + + −     Bài 2. Giải phương trình và hệ phương trình sau: a) 4 2 2 2 3 2 4 2 1 2 3 x y x y x y x y  + − =  + +    − + =  + +  b) 2 2 4 2 2 8 5 2 3 x x x x − + − + − + − = + Bài 3. Một khu vườn hình chữ nhật, có chiều dài hơn chiều rộng 5m và diện tích bằng 300 m2. Tính kích thước và chu vi khu vườn. Bài 4. Cho phương trình 2 2 4 3 0 x x m − − = (m là tham số khác 0) Chứng minh rằng phương trình có 2 nghiệm trái dấu với mọi giá trị m. Định m để phương trình có hai nghiệm 1 2 , x x thoả 1 2 3 4 26 x x − = . Bài 5. Cho tam giác ABC vuông tại A nội tiếp trong (O; R) có đường cao AH. Gọi M, N, P lần lượt là tâm đường tròn nội tiếp các tam giác AHB, AHC, ABC và I là điểm chính giữa của cung BC không chứa A. a)HM cắt AB tại E, HN cắt AC tại F. Chứng minh rằng tam giác AEF vuông cân. b)Chứng minh rằng MN // EF. c)Chứng minh rằng BM vuông góc với AN và AI vuông góc với MN. d)Khi A di động trên nửa (O) thì P đi động trên đường cố định nào.